Jump to content

Wikipedia:Reference desk/Humanities

From Wikipedia, the free encyclopedia

This is an old revision of this page, as edited by 83.100.183.180 (talk) at 13:33, 20 March 2008 (→‎"quarried from the solar disk."). The present address (URL) is a permanent link to this revision, which may differ significantly from the current revision.

Welcome to the humanities section
of the Wikipedia reference desk.
Select a section:
Want a faster answer?

Main page: Help searching Wikipedia

   

How can I get my question answered?

  • Select the section of the desk that best fits the general topic of your question (see the navigation column to the right).
  • Post your question to only one section, providing a short header that gives the topic of your question.
  • Type '~~~~' (that is, four tilde characters) at the end – this signs and dates your contribution so we know who wrote what and when.
  • Don't post personal contact information – it will be removed. Any answers will be provided here.
  • Please be as specific as possible, and include all relevant context – the usefulness of answers may depend on the context.
  • Note:
    • We don't answer (and may remove) questions that require medical diagnosis or legal advice.
    • We don't answer requests for opinions, predictions or debate.
    • We don't do your homework for you, though we'll help you past the stuck point.
    • We don't conduct original research or provide a free source of ideas, but we'll help you find information you need.



How do I answer a question?

Main page: Wikipedia:Reference desk/Guidelines

  • The best answers address the question directly, and back up facts with wikilinks and links to sources. Do not edit others' comments and do not give any medical or legal advice.
See also:


March 14

Details about Columbia River

I have been trying to elevate Columbia River to Featured Article status for some time. A few facts, present in the article before I began editing it, remain elusive. Hoping somebody can help me find a citation for the following claims:

  • Roughly 85% of basin is in U.S.
  • final 300 miles of Columbia is Ore./Wash. border
  • how much of earth's hydropower capacity is on the Columbia?
  • Hanford Superfund cleanup expected to complete by when?

Any help would be greatly appreciated! -Pete (talk) 01:19, 14 March 2008 (UTC)[reply]

You could probably find a number for hydropower fraction from Bonneville Power Association literature -- they're responsible for operating the dams on the Columbia.
Completion dates for Hanford cleanup could be a bit tricky: you could probably find an official date from the Department of Energy or the Environmental Protection Agency, but nobody expects them to meet that date, and it's a reasonably common belief that cleanup will never be finished. --Carnildo (talk) 20:56, 14 March 2008 (UTC)[reply]

By "hydropower capacity" do you mean "potential capacity", whether developed or not. I can't tell which is meant in the Columbia River article. I'll post over there too. Pfly (talk) 20:05, 15 March 2008 (UTC)[reply]

Thanks for the input, to both -- and especially to Pfly for finding the cites! I'll post more detailed replies over there. -Pete (talk) 04:16, 16 March 2008 (UTC)[reply]

Service that is still missing on the internet

The only service that is still missing on the internet is baptism. Right? Mr.K. (talk) 03:23, 14 March 2008 (UTC)[reply]

I think doing surgery over the Internet would be a bit difficult. --Bowlhover 03:29, 14 March 2008 (UTC)
Difficult? yes. Unprecedented? No. HYENASTE 04:05, 14 March 2008 (UTC)[reply]
To be baptized on-line you will need two baptized Christians to stand beside you to witness and support your baptism event. E-mail Rev. Walker with your request for baptism and the names of your two witness/support persons. Rev. Walker will process your request and set up an on-line baptism service especially for you.[1]Keenan Pepper 05:56, 14 March 2008 (UTC)[reply]
So is an online baptism as valid as a real-life one? Our non-theological answer is that the symbolism makes a reasonable case to consider an online baptism as valid as a real-life immersion of your physical body.[2]Keenan Pepper 05:58, 14 March 2008 (UTC)[reply]
Of course, for at least some denominations, all you need is one human to perform the thing. Anything added to that (e.g., Rev. Walker's Internet participation) is window-dressing rather than an integral part of baptism. As far as Catholics are concerned, you need (1) flowing water (even if it just flows over the forehead) and (2) a Trinitarian formula ("I baptize you in the name of the Father and the Son and the Holy Spirit"), and you've got a valid baptism. The person performing the baptism need not be Christian, or baptised: he may be pagan, heretic, or schismatic without affecting the validity of the ceremony. - Nunh-huh 06:26, 14 March 2008 (UTC)[reply]
Online massage exists? Pfly (talk) 07:39, 14 March 2008 (UTC)[reply]
It depends what you mean by service. Most of the services listed on that disambiguation page are not available over the internet.--Shantavira|feed me 09:45, 14 March 2008 (UTC)[reply]
I have found that anything to do with car maintenance is virtually IMPOSSIBLE to find online. I have to either buy a Haynes manual (or equivilent) or take it to a garage. Considering the huge amount of other 'do it yourself' guides online the minor-car-repair part appears not to exist. ny156uk (talk) 17:46, 14 March 2008 (UTC)[reply]
You can get your rocks off by watching stuff online, or chatting to some anonymous stranger, but if you want to have sex with another person involving physical contact, it actually helps if you're in the same room. -- JackofOz (talk) 20:52, 14 March 2008 (UTC)[reply]
*cough* Teledildonics/Bluedildonics *cough* GeeJo (t)(c) • 14:20, 16 March 2008 (UTC)[reply]

Insurance: what sort do stunt-people purchase?

Hello. Which insurance do people who do stunts purchase? If the person rescued had to pay for his or her rescue, which type of insurance covers that (let's say Tori Murden)? Thanks in advance. --Mayfare (talk) 03:38, 14 March 2008 (UTC)[reply]

The short answers would be A expensive or can't get any, B special ie expensive.87.102.83.204 (talk) 09:35, 14 March 2008 (UTC)[reply]
The insurance company I work for operate on an 'occupation class' model. That is to say that they offer different rates based on the presumed dangers of different occupations. Often to reduce premium-costs for prospective clients we may include an exclusion - whereby the individual is not covered for injury/death caused by specific outlined pursuits. Also have a look at public liability insurance. This protects people for damage done to third parties during their performances (obviously within limits/boundaries). Hiscox for example are a British firm that offer specialist insurance policies such as 'Personal Accident' where they can cover people such as racing-drivers, sportspeople etc. Many insurance companies will offer similar insurance, both against death or (if desired) against damages such as loss of limbs/loss of sight/loss etc. This webpage (http://www.tmasiainsurance.com/aic/publish/html/accident.cfm) for instance includes stuntmen in 'special risk' category - which I suspect is a nice way of saying "You better have plenty of cash!!" ny156uk (talk) 17:43, 14 March 2008 (UTC)[reply]
In Australia, in the case of death, a life assurance policy would probably have conditions or exclusions surrounding occupations classified by the company as dangerous. But if we're talking about injury requiring health treatment (which includes ambulance transportation), private health insurance companies operate on a "Community Rating" principle. This means that when it comes to paying benefits for the cost of treatment, they're not permitted to discriminate purely on the basis of a person's occupation or lifestyle (amongst other things). So, an injury sustained by a stunt-person in the pursuit of his/her occupation would be treated in exactly the same way as an injury sustained by a little old granny who got it while picking up her knitting, or whatever. This does not involve the Pre-Existing Ailment rule, which denies benefits for treatment provided in the first 12 months of membership for medical conditions the signs or symptoms of which existed at the time the membership commenced. An occupation, however inherently dangerous, is not a sign or symptom of a medical condition. As long as the injury occurred after the membership commenced, the stunt-person would be covered up to their level of cover, without even the standard 2-month waiting period applying, because it would be classified as an accident. If some special rescue service was needed, not just an ambulance, and they charged for their services, the cover would depend on the company and you'd need to check. In general, I'd suggest that this would not be considered to be "health-related treatment", because a person who needs rescuing is not necessarily injured in any way, and you might need to investigate taking out separate cover for such costs. -- JackofOz (talk) 20:46, 14 March 2008 (UTC)[reply]
Rumor has it that Lloyd's of London will insure anyone for anything, if they can work out the odds. --Carnildo (talk) 21:04, 14 March 2008 (UTC)[reply]

Trans Holocaust victims: any details on transgender victims?

Is there any available information about transgender people who were affected (imprisoned, killed, experimented upon, etc) in the Nazi Holocaust? I can find brief mentions that say they were, but nothing more detailed than that. My suspicion is that from the Nazi perspective, they were just lumped with gays and lesbians which might make them harder to identify historically. --Ephilei (talk) 04:35, 14 March 2008 (UTC)[reply]

I looked, Ephilei, but have found nothing substantive. I think you are right to assume that transgenders were placed in the same pink triangle class as other sexual offenders. Clio the Muse (talk) 00:19, 15 March 2008 (UTC)[reply]
It may be controversial to claim this, but in the 1930s-40s, I doubt that what we consider as 'transsexual' even existed. Gender is, afterall, a completely cultural phenomena, and cultures have changed hugely over the last 70 years. Ninebucks (talk) 20:54, 15 March 2008 (UTC)[reply]
Not controversial at all. However, you're confusing transgender with transexual. Transgender have existed for thousands of years, specifically including in Germany during the rise of the Nazi party. See Institut für Sexualwissenschaft. It all gets fuzzy after that. --76.192.189.206 (talk) 04:41, 22 March 2008 (UTC)[reply]

Zita, death: 13th-century Saint Zita's clothes

Saint Zita died in 1272. Was she entombed in the clothes she wears this image? I have trouble believing that those clothes are over 500 years old. HYENASTE 05:17, 14 March 2008 (UTC)[reply]

Why? Adam Bishop (talk) 06:59, 14 March 2008 (UTC)[reply]
Zita died in 1272, so the clothes you see on her now would not have been those in which she was buried. There is some indirect evidence here that she was undressed in about the 1990s. It is an article about mummified saints' bodies, with a brief mention of the examination of St Vita's body, which revealed no marks of incisions on her body. The body of Bernadette of Lourdes is recorded to have been given a fresh habit on at least two occasions, so the re-dressing of a body is not in itself sacrilegious. The fact that Zita is dressed in a beautiful new dress is quite poignant when part of the story of her piety is that she was continually giving away the new clothes her employers gave her, so that poor people could be dressed. SaundersW (talk) 09:53, 14 March 2008 (UTC)[reply]
Aw, the clothes aren't part of the uncorruptedness? Adam Bishop (talk) 10:20, 14 March 2008 (UTC)[reply]
While Superman apparently transfers his invulnerability to his clothes by contact, saints don't seem to have that same kind of contagion. Anyway, why shouldn't a girl get new clothes from time to time so she looks her best, even when she's dead! SaundersW (talk) 10:30, 14 March 2008 (UTC)[reply]

Doing a history PhD in the UK

How much does it cost? How do I go about getting funding? How do I find the best university for my particular subject? Can I do it 'remotely' rather than having to live in the university town? Help, please! 81.159.89.104 (talk) 08:22, 14 March 2008 (UTC) Perhaps I should add that I graduated 23 years so am very out of touch about these things. 81.159.89.104 (talk) 08:32, 14 March 2008 (UTC)[reply]

Funding is distributed by the Arts and Humanities Research Council, but it may be difficult to get. The Open University doesn't appear to do a History PhD, but I recommend browsing it and other university sites for advice on applying for funding etc. AlmostReadytoFly (talk) 09:30, 14 March 2008 (UTC)[reply]
Thanks for that. But oh dear, I have more questions. Do I have to do an MA before the PhD or can I just go straight into doing a PhD? (There's certainly enough material and research potential to make a very meaty thesis) Is there a time limit on how long it takes to complete the PhD? What is a ballpark figure for doing a three year PhD, including university fees etc but excluding living expenses? 81.159.89.104 (talk) 09:57, 14 March 2008 (UTC)[reply]
Here are some facts and figures on postgraduate degree fees at Cambridge, as an example. One way to choose a university is to look at recent books on the area that interests you and see where the authors are based. That does not guarantee that they want to take on students but it is a first approximation. Studying remotely, and even in another country, is possible, but you will need to consult the rules of the particular university. It has disadvantages: you are remote from the library, which is a particularly important resource in studying history, you are remote from your supervisor, which may be a problem if he/she is an elusive individual, and you are remote from fellow students, a very important and often under-rated resource. As a mature student you will have the disadvantage of having been away from study for a while, and of feeling maybe out of place as a student. You will have the great advantage of being much more mature in your attitude to life and having a lot more life experience and general knowledge and wisdom on which to draw. If you decide to go for it, it could change your life. Good luck! SaundersW (talk) 10:11, 14 March 2008 (UTC)[reply]
Oh my! Lots of questions! The time limits are generally 3 years minimum to 7 years maximum. You don't have to have an MA, I believe that some funding bodies require you to have either a first or upper second class honours, or an MA. SaundersW (talk) 10:13, 14 March 2008 (UTC)[reply]
Thanks for all your answers, and bloody hell, they're expensive. Would a grant cover 100% of the fees, or would I have to stump up some? I have a first and an MA which probably isn't worth the paper it's written on as it's from Cambridge ...! Jasper33 (talk) 10:20, 14 March 2008 (UTC)[reply]
As for history PhDs, I have no idea what kind of funding you can get. But being an MA (Cantab) certainly opens doors to you. (Especially if it isn't the first thing you mention about yourself!) SaundersW (talk) 10:28, 14 March 2008 (UTC)[reply]
it wouldn't open any doors in academia as it's not a real qualification and would not count as a proper MA. --Fredrick day (talk) 11:21, 14 March 2008 (UTC)[reply]
I do not mean that it counts as an MA, though just that it impresses people disproportionately. (In my experience, thus original research warning applies) SaundersW (talk) 11:32, 14 March 2008 (UTC)[reply]
Haha, outed myself there, didn't I, dammit! So much for craftily unlogging on to ask the questions ...! Jasper33 (talk) 10:31, 14 March 2008 (UTC)[reply]
UK funding is pretty much all on a 1+3 basis now - in that you get funded to do a Masters and then if you show the talent for it, they will continue to fund you for another 3 years. If you are in the UK and doing the "right" sort of history, then look at Welcome Trust funding - it's a fantastic gravy train to get onto. --Fredrick day (talk) 11:15, 14 March 2008 (UTC)[reply]
Thanks. That first 'if' is a pretty big one. And sadly, no, not the 'right' sort of history for me. I think I'm going to have to start looking for a 'Kindly Gentleman', a la The Railway Children ... Jasper33 (talk) 12:02, 14 March 2008 (UTC)[reply]

I would be interested to know if any of the respondents here on Humanities actually have a UK history PhD? —Preceding unsigned comment added by 86.147.185.172 (talk) 11:06, 14 March 2008 (UTC)[reply]

:: I don't have a history PhD mine is in another subject *but* my significant other's PhD is a history PhD from one of the top rated centres for her particularly area of history and I proof-read it for her and a couple of our friends who are also history graduates. If you are doing a history PhD you have an even bigger problem than I first thought. Writing a PhD in history is a very specialist thing, in my area there is no subtext really, you have your argument, you present it, you present your results. However in history, you are writing some far more involving because of their use of footnotes - those can add up to almost as much text as the main body. In addition, without being attached to a university, I don't see how you'd have the document access required for a history PhD. --Fredrick day (talk) 11:13, 14 March 2008 (UTC) Sorry I'm confusing you with the problem in the section below. --Fredrick day (talk) 11:17, 14 March 2008 (UTC)[reply]

If it is really relevant, mine is from a School of Mathematics. I have close connections with academics (who have PhDs and supervise PhD students) in economics, management, engineering and ancient history. All this is of course utterly unverifiable, as are any other statements made by anonymous internet users. You'll see that I have made no comments which refer specifically to history, except for the importance of a good library. All the rest refers to UK PhDs in general. If there is anybody who has a UK history PhD who can give even more specific information who is awake, able and willing to contribute, I am sure that they will. SaundersW (talk) 11:24, 14 March 2008 (UTC)[reply]
Hi Frederick. I'm an archaeologist and part of my work regularly involves research, often historical, so I should be okay with the methodologies. I take your point about the document access - I've been interested in my proposed topic for a while now and have gathered most of the available published work already (there's not a lot about); luckily for me I have access to primary archives and people for taking oral histories. I meant 'remotely' in the sense of not having to have residence during a full-time course. Jasper33 (talk) 11:29, 14 March 2008 (UTC)[reply]
By residence, do you mean live by the campus? that's fine? it's not uncommon just to pop in every couple of months to see your supervisor and do the rest by email. --Fredrick day (talk) 11:32, 14 March 2008 (UTC)[reply]
Excellent - just the sort of thing I wanted to hear! Jasper33 (talk) 12:02, 14 March 2008 (UTC)[reply]

A (successful) contemporary PhD student of mine lived in the USA most of the time and spent spells of a fortnight or so on site for supervision.

For what it is worth, this is my analysis of the support you need for completing a PhD.

1. Somebody with the subject knowledge to tell you what is new, what is interesting, what is relevant, what is counted as evidence.

2. Somebody who can tell you what a PhD in your subject actually looks like (structure, arguments, references, etc)

3. Pastoral support. It can get very lonely, and there should be moments where you question what you know, how you know it, what reality is, etc. You need somebody to say that these will pass.

These do not need to be provided by the same person. SaundersW (talk) 11:46, 14 March 2008 (UTC)[reply]

Wise advice. Thank-you. Jasper33 (talk) 12:02, 14 March 2008 (UTC)[reply]

My PhD/Doctorate question

Can I just write a doctorate and present it to a university body for 'assessment'?. Does it cost money to present it.87.102.83.204 (talk) 09:33, 14 March 2008 (UTC)[reply]

I suppose you could try, but there are numerous bureaucratic hoops they make you jump through first. The usual process, at least here, is a 4-year BA, a one or two-year MA, and a 6-year PhD, 2 of which consist of classwork, one of preparing for a Major Field Exam, and the rest in writing a thesis. It's different everywhere but the basic point is that it takes A Very Long Time. So if you're not a student and you haven't suffered the previous six years (or whatever), you probably won't get very far. I imagine they would actually just toss it in the trash. (But naturally, you can publish whatever you want outside academia!) Adam Bishop (talk) 10:19, 14 March 2008 (UTC)[reply]
Generally you have to be registered as a student for the course of your thesis. The idea is that you carry out what is essentially an apprenticeship in research. Some Universities will grant an honorary degree for the achievement of a work of scholarship which they regard as equivalent to a degree, especially if the author is an alumnus/a of that university. SaundersW (talk) 10:23, 14 March 2008 (UTC)[reply]
Of course if you manage to do real ground breaking research you could be awarded an Honourary Ph.D. There are many examples of this, including Jack Horner (paleontologist), who had no formal degree but was given a Ph.D by the University of Montana. I don't think this would be a quick route though. -- Q Chris (talk) 16:09, 14 March 2008 (UTC)[reply]
In the UK is a thesis effectively sufficient if I can get it accepted?87.102.83.204 (talk) 10:53, 14 March 2008 (UTC)[reply]
yes but you'd have to do your viva (an oral defence of the work in front of 3 experts) to get the qualification. Having said that, getting a university to just "accept" your PhD is not going to be an easy process - first they have no idea of the quality and second are going to wonder why they didn't screw £3000 X 3 years of bench fees out of you. Moreover, if you are doing it "cold" on your own and with no guidance, the chances are that it's not going to be of an acceptable standard. --Fredrick day (talk) 11:01, 14 March 2008 (UTC)[reply]
We have a Doctor of Philosophy article that contains outline information on admission requirements and funding in various countries . Broadly speaking, to be awarded a Ph.D. you have to produce a "thesis or dissertation consisting of a suitable body of original academic research, which is in principle worthy of publication in a peer-refereed context". As has been said, you are very unlikely to be able to do this for the first time without some degree of guidance and input from an expert in the field, who acts as your advisor. But experts are busy people, so you have to convince a relevant expert and the establishment that employs them that it is worth spending their time on helping you. And part of the deal may be that you choose a research topic that is aligned with your advisor's current research. A relevant MA or Honours BA is one piece of evidence that you can present that shows you have the appropriate background and outlook to successfully complete a Ph.D.; personal recommendation is another piece of evidence that may be required. Your best way to get more information is to contact one or two prospective universeties, tell them about your background and ask some specific questions. Gandalf61 (talk) 11:14, 14 March 2008 (UTC)[reply]
Yes thanks both of you - you confirmed what I thought.87.102.83.204 (talk) 11:57, 14 March 2008 (UTC)[reply]
And it's not just that you won't be able to probably "do it right", it's also that the definition of "do it right" is a little different for every advisor. I have multiple advisors and each of them has a somewhat different notion of what the final product should look like to be a good thesis, and the final product is an amalgamation of their feedback. If I had never talked to them and never worked with them and never really got any feedback on it then it's not likely what I would produce would look like something they wanted. --98.217.18.109 (talk) 12:14, 14 March 2008 (UTC)[reply]
Kind of pisses on the idea of it being 'your PhD' when you just have to do what the advsiser tells you? Maybe I've just got a problem with authority figures? 87.102.83.204 (talk) 13:15, 14 March 2008 (UTC)[reply]
Well it still yours, because it's still you who writes it and drives it's development - but your supervisor is there to keep you on track and make sure the quality is there. Sure you can take no notice, but you are likely to turn a complete steamer that gets rejected and then you've just wasted 3 years of your life. --Fredrick day (talk) 13:19, 14 March 2008 (UTC)[reply]
Believe me, 3 years is nothing (smile).87.102.83.204 (talk) 15:37, 14 March 2008 (UTC)[reply]
It's all relative - if after working for 14 hours a day for three years, I didn't get my PhD, I would have considered it a complete waste of my time. --Fredrick day (talk) 15:45, 14 March 2008 (UTC)[reply]
What about the value of the work itself -wasn't that worth doing too - for no paper reward (just feeling chatty, not argumentative)87.102.83.204 (talk) 15:48, 14 March 2008 (UTC)[reply]
it has no inherent value, yes you know a lot more about the subject but so what? to what end? what's the point of doing a PhD unless you want to complete it? a complete headache with no payoff, you might as well just study it in your spare time and save your thousands of pounds and many many pointless meetings and discussions. --Fredrick day (talk) 15:53, 14 March 2008 (UTC)[reply]
I will disagree here. I loved writing my dissertation. It was the high point of my graduate career. It was its own reward for me. I did not go on to pursue an academic career, but still the dissertation was well worth doing. I hope that I will have a chance to write a book again, next time with more of an eye to publication. If you do take on such a project, you may want to be careful to pick one that will be published. It helps to know what kinds of topics are currently "hot" and how to market your idea in that context. You might approach an academic (or even trade) publisher in advance with a query letter. Marco polo (talk) 17:39, 14 March 2008 (UTC)[reply]
If your committee had done things right, you would have been conditioned such that you became physically ill when you came within sight of your department's building or when you heard any of their voices. Edison (talk) 19:23, 14 March 2008 (UTC)[reply]
(he he) And if they hadn't done it right, there's still every chance that contact with them would make you mentally ill. -- JackofOz (talk) 20:16, 14 March 2008 (UTC)[reply]
Your point about choosing a marketable target is well taken. I've seen a lot of people who chose REALLY narrow topics that more than a dozen people in the world couldn't have cared about. That's a pretty bad idea in most cases—it'll make it pretty hard to get a job, or to get something published, or to even get your friends to care at all. It's no fun doing scholarship that is only read by a dozen people in the world, especially since you'll end up hating at least half of them. I never take on a project unless it's the sort of thing I could explain to my family in a way that they could understand it and see why it was interesting to me (my family are not, to say the least, academics). --98.217.18.109 (talk) 21:08, 14 March 2008 (UTC)[reply]
Well, if you view experts as "authority figures" then why do you want a PhD in the first place? (To be "The Man" yourself?). A PhD is very similar to an apprenticeship. You are supposed to be guided by an expert so that you too can become an expert. (Some do better guiding than others, some people need less guiding than others. But it's part of the system.) Anyway, if you pick your advisor well, it's not a fascist regime, it's a concerned and interested person who will help you push your work as far as it can go. A great advisor is hard to find, but well worth it. There are, of course, a lot of bad advisors out there too. It's not the end of the world if you are stuck with one, but you've got to figure out mitigating strategies. --98.217.18.109 (talk) 21:08, 14 March 2008 (UTC)[reply]
It was a hypothetical question. Though I've often wondered if any of my work would with a little extra work pass muster as a Ph.D. thesis - I guess not - I doubt I've done anything truly original.87.102.21.171 (talk) 10:06, 15 March 2008 (UTC)[reply]
By itself, probably not, but that's a statement made by assuming your work is just average unguided work by someone who hasn't been looking much at academic style expectations, methodological issues, current concerns, etc. A (modern) PhD thesis is not a standard work—it is a specialized document. It is not a book. I could become a book, but that takes even more work. I don't think most PhD theses are meant to be read at all—they're meant to satisfy a group of prickly committee members, they're meant to convince a future employer that you know how to work within the academic norms and expectations. Most theses by themselves get read a total of four or five times. If they get turned into a book — which involves transforming them into quite a different document in most cases, for a different audience, with different expectations, and stripping away much of the scholasticism — then maybe they get read more widely. Of course, I speak from my own discipline's experience (History) but for most humanities/social sciences work I think it probably applies.
Which is all a way of saying that if you don't think your work looks like a PhD thesis, that's probably a good thing. It's not that you're probably not qualified to write one, but the document itself is a product of a particular educational and employment system, with particular expectations. It's like trying to submit a wonderful little essay on what you spent money on to the IRS, rather than filling out their tax forms. It might contain all of the same information, and be an entertaining read, but they're not going to accept it as a proper tax filing.
As an aside, one of my favorite non-fiction books, the award-winning City of Quartz, was once a rejected PhD thesis. The book is now a standard assignment in many different disciplines and the author has since won a MacArthur "Genius" fellowship. Approval by entrenched academics is not all there is to life, of course (but then again, most of us aren't really going to be MacArthur "Geniuses" either). --98.217.18.109 (talk) 17:14, 15 March 2008 (UTC)[reply]
Here is an example of a book which earned its author a doctorate: Divine Prerogative and Royal Pretension: Pragmatics, Poetics and Polemics in a Narrative Sequence About David (2 Samuel 5.17-7.29) (Journal for the Study of the Old Testament Supplement) by Donald F. Murray. I am trying to hunt down a reference to the event, but I'm 99% certain that it was from Cambridge. Don Murray has returned to Australia, maybe Brisbane, so I can't contact him to ask him. SaundersW (talk) 16:54, 16 March 2008 (UTC)[reply]

Beastorn report: see archives

I have completed my research on JackofOz's enigmatic Beastorn. Interested editors with sufficient security clearance may view it (eyes only) in the archives.

¡ɐɔıʇǝoNoetica!T09:58, 14 March 2008 (UTC)[reply]

You should take some rest. Perhaps a darkened room? --Tagishsimon (talk) 12:10, 14 March 2008 (UTC)[reply]
Perhaps man was never meant to know the truth of the Beastorn. 206.252.74.48 (talk) 12:50, 14 March 2008 (UTC)[reply]
Funny because the name rings a bell for me too. Perhaps some sort of genetic memory??87.102.83.204 (talk) 13:17, 14 March 2008 (UTC)[reply]
My effusive (but hopefully not fulsome) thanks to St Noetica of the Heatwave are to be found in the archives. -- JackofOz (talk) 20:10, 14 March 2008 (UTC)[reply]
O please! It was really no trouble. One has to dramatise for Apocalyptic effect, that's all.
My preferred hypothesis is still that it's a simple mistake for "Eastern", due to either a scanner or a scribbling. The fact that Beastorn looks plausible as a medieval twist on Byzantine would then account for the word's persistence and plausibility, and especially its initial resistance to correction at copyediting, or between editions. Still, we can't rule out that it's authentic.
Looks like we've got a meme on our hands, folks. I think Beastorn will be the next eggcorn. JackOO, I'll communicate with you soon about how to investigate this further, and other matters. :)
¡ɐɔıʇǝoNoetica!T21:32, 14 March 2008 (UTC)[reply]
Ah, the ever elusive Beastorn, a shy creature, much like the Loch Ness Monster. Or, perhaps, it is best ranged in the same class as the Phoenix and the Roc!
And, hast thou slain the Jabberwock?
Come to my arms, my bemish boy!
O frabjous day! Callooh! Callay!
She chortled in her joy. Clio the Muse (talk) 00:32, 15 March 2008 (UTC)[reply]
BEEEEEEEA-STORRRNNN! --98.217.18.109 (talk) 17:09, 15 March 2008 (UTC)[reply]
I've just realised why I kept getting this strange feeling of déja vu reading the news. The prophecy has been fulfilled! Skittle (talk) 14:13, 18 March 2008 (UTC)[reply]

Paul de Lagarde

I don't suppose too many people read the work of Paul de Lagarde now. I know him as a 'cultural pessimist', one of the roots of the Nazi movement. I must say that I find him and others of his kind quite intriguing and would like to know why he was so disenchanted with Bismarck's work in German unification? Why did he take such a negative view of the new Germany? Doctor Claude (talk) 11:56, 14 March 2008 (UTC)[reply]

There is a lot of good stuff on Paul de Lagarde in Fritz Stern's The Politics of Cultural Despair, Doctor Claude. He was one of those misty-minded right-wing intellectuals, ever in pursuit of nebulous notions of 'spiritual renewal'. This, he believed in the period before 1871, would come with German unification; that the creation of a new nation would also entail the creation of a new, more elevated type of human being. But the reality of the Second Reich came nowhere near his expectations, making his disappointment-and his reaction-all the greater. Indeed, he went so far as to argue that Bismarck's creation was the very antithesis of what he had looked for; that it was more likely to bring collapse rather than spiritual renewal. Quite simply, it was boring! The new Reich was stultifying; it crushed the individual beneath its stolid sobriety, and thus annihilated the only source of cultural vitality-"Everything depends on the human being, and Germany lacks nothing so much as men; there is nothing toward which Germany, with its adoration of the State, of public opinion, of Kultur, and of success, directs so much hostility as toward the individual, who alone can bring it life and honour." Clio the Muse (talk) 00:54, 15 March 2008 (UTC)[reply]

Latin Empire

I should like to know please how the Greek people of the east reacted to the setting up of the Latin Empire after the fourth crusade in 1204? —Preceding unsigned comment added by K L Orr (talkcontribs) 15:52, 14 March 2008 (UTC)[reply]

To begin with, K L Orr, most people seem to have accepted the change of power passively: some even welcomed it. Besides, all of the old leaders had left prior to the fall of Constantinople, including John Camaterus, the Patriarch, who could have been expected to be an obvious pole of resistance to the Latin conquerors. People were ready to hail Boniface of Montferrat as the new Basileus; and when Baldwin of Flanders was chosen in his place, many came to Hagia Sophia to cheer at his coronation.
This mood would appear in part to have been caused by anger with the perceived failings Byzantine authorities. Imperial officials fleeing from Constantinople were treated with hostility in the Thracian countryside. When Baldwin advanced towards Thessalonica in the summer of 1204, far from resisting, the local people came out to welcome him. But reactions also varied from place to place; welcoming in some, defiant in others. With the passage of time the mood of defiance grew ever greater. Clio the Muse (talk) 01:19, 15 March 2008 (UTC)[reply]
Some good places to look for this period are Nicetas Choniates, the Chronicle of Morea, and as a secondary source, "The Franks in the Aegean" by Peter Lock. After the initial shock of conquest things weren't all that much different in the Latin Empire proper; they weren't there long enough to have much of a lasting effect. The emperors were fairly weak, were constantly at war on numerous fronts, and they were far outnumbered by their Greek subjects. Elsewhere though, like in the Principality of Achaea or the Duchy of Athens or the territories ruled by Venice, Latin rule continued for centuries, so it was somewhat different. From a religious perspective, which was really the heart of the matter, some dioceses were willing to recognize a Latin bishop if the Orthodox hierarchy was allowed to co-exist; some did not accept it at all; and some essentially recognized the authority of Rome, or at least pretended to. There wasn't much they could do, since the emperor and the patriarch had left even before Constantinople fell, as Clio said. Adam Bishop (talk) 09:40, 15 March 2008 (UTC)[reply]

Sociology question: selective affinity?

Hi. can anyone please tell me what :"selective affinity" means? i really appreciate it. this is a term in sociology. thanks. --Steve, Sm8900 (talk) 15:58, 14 March 2008 (UTC)[reply]

Logically...an affinity can be a similarity/appreciation/relationship and selective can be discriminating/divisive. As a result i would expect given the context selective-affinity means social-relationships that are a result of choice/consideration (or active). Your best bet is to look for its use in a sentence and try to understand the context of what it is trying to say. Both are pretty every day words so i'd be surprised if they take on any meaning beyond their well-known meanings in this instance ny156uk (talk) 17:34, 14 March 2008 (UTC)[reply]

Do you mind using your logical analysis of language on the term "ecological feminism"? This isn't meant to be an asshole, but just that many disciplines are not very good in using terms which make sense. To answer the OP, it probably means roughly the same thing it means in chemistry (where it's well defined as, some substrates used in chemistry have selective affinity, which means they only bind to certain molecules). It's just the typical post-modernism coopting technical words and using them in non-technical situations.--droptone (talk) 19:24, 14 March 2008 (UTC)[reply]

Advice on moving to Belfast

I'm moving to Belfast, Northern Ireland in a few months. I am Canadian and have lived only in Canada and the United States. I have read the articles Belfast, Northern Ireland, The Troubles, Politics in Northern Ireland and some related linked articles, Etiquette in Europe#United Kingdom, Etiquette in Europe#Ireland (and am not sure which of the latter two is more relevant), Mid Ulster English, Hiberno-English and some other linked linguistics articles (which I have some trouble reading, being unfamiliar with the IPA). What else (on Wikipedia and elsewhere) should I read?

I also have some specific questions. What do the residents/natives of Northern Ireland tend to call themselves? Irish, British, Northern Irish, one or all of the above and will I insult them if I get it wrong? Are there other ways of insulting people/starting fights that I might inadvertently stumble into? What is the standard day-to-day term for the Republic of Ireland? Do residents of Northern Ireland wear shoes in their homes? (That last one might seem weird, but it's a difference between Canada and the U.S. and one I'm curious about).

Thanks, moink (talk) 16:05, 14 March 2008 (UTC)[reply]

Slightly off-topic, but that article Etiquette in Europe is very strange. I would note that almost everything mentioned in the section on Ireland (except the things which specifically refer to Irish politics) apply in the UK. And the article is almost entirely uncited. And written in a familiar tone. Hmmm. Skittle (talk) 17:16, 17 March 2008 (UTC)[reply]
I think there was a very similar question a few months ago - if anyone remembers it they could dig it out of the archives.87.102.83.204 (talk) 17:13, 14 March 2008 (UTC)[reply]
Residents of Northern Ireland have different ways of defining their identity depending on their ethnicity/religion. Catholics generally consider themselves "Irish"; Protestants generally consider themselves "British". To be safe, I would avoid using any term or referring to identities whenever possible. Just tiptoe around the topic. When you absolutely need a term to refer to all residents of the region, without knowing their religion, then I would use the fairly neutral and accurate "Northern Irish". Not so many people in the region self-identify as Northern Irish, but most will acknowledge that it is accurate. Probably the most neutral way to refer to the Republic of Ireland is "the Republic of Ireland" or "the republic" for short. Again, you may blend in better by referring to specific places in the republic rather than the republic as a whole when possible. For example, "I will be in Dublin for the weekend" instead of "I will be in the republic for the weekend." The foregoing is based on a summer I spent mostly in the republic but also partly in the north some years ago and reading I've done since. Others will probably have a deeper knowledge than I. As for shoes, I'm not sure. (However, I have lived in the United States most of my life, and in my experience, whether shoes are worn indoors varies from household to household here. Most do wear shoes indoors, but a substantial minority don't in my experience. I tend to ask before entering.) Marco polo (talk) 17:23, 14 March 2008 (UTC)[reply]
Thanks, just reading your answer and what you use helped too, "the republic," "the north." I was there for a job interview and people were asking about my familiarity with the area etc.; it sounds like I did ok saying things like "Well, my sister lived in Dublin for a year, but I've never been to the UK or Ireland at all before now myself." moink (talk) 18:47, 14 March 2008 (UTC)[reply]
Yes, talking about towns and cities (or counties, if you're off to the countryside) rather than countries is a wise idea; that fails only for Derry/Londonderry. -- Finlay McWalter | Talk 17:57, 14 March 2008 (UTC)[reply]
There's an article linked from Derry/Londonderry name dispute telling the story of a Canadian tourist who was told by a train ticket vendor that "No such place exists" when she asked for a ticket to Derry. Just the kind of thing I can see getting myself into unknowingly. moink (talk) 18:47, 14 March 2008 (UTC)[reply]
(ec)The language of nationality is a somewhat thorny issue, one you might wish to avoid at first (see caveat later). Loyalists will typically call themselves British, Republicans will typically call themselves Irish - but being a foreigner you'll get a lot of slack, providing you don't start expressing strong views about NI politics (which is, after all, probably unwise when visiting any foreign country). ROI is generally called "the republic". Caveat: while the sectarian fissure runs through all strata of NI society, the gulf was always greatest at the poorest parts of society - as I imagine you'll be living in some nice twee wee place and living and working with middle-class professional people (not a council house on the Falls Road) the sectarian divide will not rear its head all that much, and you won't know (and it's probaby impolite to ask) from which "side" someone comes. The Troubles for NI are a bit like WW2 for Germany - people might get a bit hacked off if you go on the subject too much, as they'll feel there's so much more to NI than that. My advice - buy a tshirt with a maple leaf on it, as you'll probably spend much more time telling people you're Canadian (and not American) than worring about the intricacies of sectarian strife. -- Finlay McWalter | Talk 17:43, 14 March 2008 (UTC)[reply]
I've met enough Belgians tired of being mistaken for French, Austrians tired of being mistaken for Germans, and New Zealanders tired of being mistaken for Australians (and one Austrian who had to explain to her roommate that Austria is a different country than Australia), that I'm ok with people making that error and I'm not overly offended. moink (talk) 18:47, 14 March 2008 (UTC)[reply]
One other piece of advice I'd offer is to learn the broad outlines of the history of Ireland (including Northern Ireland) since the Middle Ages. I agree with Finlay McWalter that you definitely do not want to take sides, but knowing the history, as nearly all locals do, will help you to understand the different perspectives. Marco polo (talk) 18:01, 14 March 2008 (UTC)[reply]
That's why I'm reading a lot on Wikipedia these days. However, I'm bad with dates and details, and I expect to see murals saying things like "Remember 1916" and having to go home and look it up in Wikipedia, again, because I don't know which of many many incidents they're talking about or which side that mural supports. moink (talk) 18:47, 14 March 2008 (UTC)[reply]
As for Derry/Londonderry, you can feign ignorance as a foreigner. You can say something like "I am going to Londonderry...or is it Derry?" Marco polo (talk) 18:01, 14 March 2008 (UTC)[reply]

And no one has yet told me if they wear their shoes in their homes. Perhaps a native of Belfast visits this desk and can eventually help me out on that one. moink (talk) 18:47, 14 March 2008 (UTC)[reply]

You might like to listen to this series about Northern Ireland, and the Good Friday agreement in particular, on BBC Radio 4, entitled The Price of Peace. The Radio 4 website has a 'listen again' facility where you should be able to hear all three episodes wherever you are. [3]. 86.133.55.238 (talk) 20:03, 16 March 2008 (UTC)[reply]

Tertullian and the definition of heresy

How does Tertullian define heresy in his "Preemptive Objection against Heresy"?86.151.241.198 (talk) 16:29, 14 March 2008 (UTC)[reply]

Assuming you mean De praescriptione hereticorum, you can read it yourself. Algebraist 16:40, 14 March 2008 (UTC)[reply]

For Tertullian-at least at the time of writing-the Church alone bore the apostolic rule of faith, revered the canon of Scriptures, and bore through its ecclesiastical hierarchy the sanction of apostolic succession. Heretics were those who challenged any of these precepts. They were people who refused to accept the rule of faith, as others did. Instead they challenged people to raise theological questions to which there was no answer, "...being ready to say, and sincerely of certain points of their belief, 'That is not so' and 'I take this in a different sense' and 'I do not admit that'". A bit like Wikipedia, then!

Tertullian has it that all such unnecessary questioning automatically leads to heresy-"This rule was raised by Christ, and raises among ourselves no other questions than those which the heresies introduce and which make men heretics!" Heretics, moreover, are those who do not restrict themselves to the Scriptures, but bring in other writings or challenge orthodox interpretations. Heretics are, quite simply, rebels, in theological and in practical terms. Clio the Muse (talk) 01:38, 15 March 2008 (UTC)[reply]

Trajan and the First Dacian War

Precisely what forces, legions and auxiliaries, did Trajan deploy in the first Dacian War? I cannot find the information I seek in your article.Marcus Trajanus (talk) 20:46, 14 March 2008 (UTC)[reply]

I was unable to find anything precise. This source [4] tells that two new legions were recruited and that overall 11 legions were involved. However I have read that large detachments (called vextallions) were the norm at these times. To transfer whole legions from one point of the Roman empire to another was simply dangerous and usually foolish as local threats like invaders, raiders, brigands, and rebels would take all too quickly advantage of the absence of Roman garrisons. It was better only to sent a part of a legion (usually the best units). Flamarande (talk) 23:59, 14 March 2008 (UTC)[reply]
Found this: [5] (through a Wiki-article). It gives the names of some of the involved legions as well as some original sources. Flamarande (talk) 00:09, 15 March 2008 (UTC)[reply]

From the information I have you (it is you, is it not?) had a total force of 90,000 men by May 101AD. At the core of this there were seven legions, as well as vexillaries, special veteran legionary detachments drawn from at least five others. There were also the Praetorian cohorts, commanded by Claudius Livianus. Added to this there were 24 cohorts of auxiliary cavalry, and more than 70 cohorts of auxiliary infantry, including slingers and archers. And, to match the stealth of Decebalus' forces, you also organised the symmachiarii, troops of barbarians armed in native fashion but led by Roman officers. Ave, Imperator! Clio the Muse (talk) 01:54, 15 March 2008 (UTC)[reply]

I've now given you a place in the Pantheon, O Muse. Marcus Trajanus (talk) 19:51, 15 March 2008 (UTC)[reply]
My thanks, mighty Caesar! Clio the Muse (talk) 03:21, 16 March 2008 (UTC)[reply]

Computer as a Judge?

Has there been any work done on eleminating human arbitrariness of the judge in law so that everyone really is treated equal, eg by using a computer that judges based on input parameters, the current law and precedents? If such a project is running, what is it's name? If considered pointless, why? (and is there an articel about is?) Thank, --Gnorkel (talk) 21:18, 14 March 2008 (UTC)[reply]

I have never heard of such. The closest you may find (in USA) is something like the Federal (or State) Sentencing Guidelines ... which is not a computer program. And, even that, ... they are "guidelines" rather than strictly applied rigid formulas. I will note that the premise of your question is rather flawed. Most would feel that the judge's role should indeed have discretion (what you call "arbitrariness") in treating people equally. Remember that "equal" does not always or necessarily mean "the same". For example, for a given offense, to fine Joe Blow $100 and to fine Donald Trump $100 may not be fair, even though the amount is equal. The $100 represents a huge percentage of Joe Blow's discretionary income and merely a drop in the bucket for Donald Trump. Hence, such a fine / punishment would be tantamount to saying to Trump, "Go ahead and break this law all you want because our fines are trivial and you can simply pay your way out of your infractions a million times over without feeling the pinch". So, the dollar amount is equal ... but the effect of that is clearly unequal / imbalanced / unfair. Generally speaking, I think that most would say (at least I would say) that such a program as you cite would indeed be pointless. The bottom line being that every case is different, every person (defendant) is different, each case/person brings a unique set of facts to the table. It would be hard, if not impossible, to "quantify" all of these unique differences into a computer program series of calculations. That's why you sometimes see "three strikes and you're out laws" sending a man to prison for the rest of his life for stealing a donut or a candy bar. That's clearly not the intent or spirit of the law -- but, sometimes, it is the effect of the letter of the law. There is typically some outrage when these odd results occur (rightly so) ... which makes the case for individual judicial discretion in how to treat different people "equally". Thanks. (Joseph A. Spadaro (talk) 22:52, 14 March 2008 (UTC))[reply]
Oregon Ballot Measure 11 (1994). This replaced sentencing guidelines, similar to those described above, with mandatory minimums. IMHO, it's been an absolute failure. -Pete (talk) 22:58, 14 March 2008 (UTC)[reply]
That may be true. But even a "mandatory minimum" is not a rigid, set, mathematical formula -- as the original poster questioned (of the type: If a person of X Characteristics commits the Crime Y, then his sentence calculates to be Z Years in prison). A mandatory minimum merely sets a minimum floor -- yet, there is still judicial discretion to exceed that minimum (upward). So, as such, it partially is relevent ... but not fully. Thanks. (Joseph A. Spadaro (talk) 23:44, 14 March 2008 (UTC))[reply]
There has been a lot of work done on combining different evidence in decisions as part of artificial intelligence. As far as I recall there are various ways of combining two or more probabilities (not just Baysian) but unfortunately cannot remember enough about it to point you to an article. 80.2.196.223 (talk) 23:24, 14 March 2008 (UTC)[reply]
Joseph, your point is well taken, and theoretically true. However, a sufficiently high floor effectively eliminates discretion as well (if the floor is above all reasonable sentencing levels.) It's my understanding that that's the case in a large number of Measure 11 cases; it would be interesting to learn how often judges exceed Measure 11 sentences. I am pretty sure it's very rare, but don't have any sources to back that up. -Pete (talk) 00:40, 15 March 2008 (UTC)[reply]
What you are thinking of is an expert system relating to law. Fuzzy logic is one way of combining probabilities or certainty factors. I have not been able to remember or find the more appropriate Somebody-Somebody theory or law of combining probabilities. Edit: Its the Dempster-Shafer theory. There are other similar theories also. 80.2.196.223 (talk) 00:59, 15 March 2008 (UTC)[reply]
Are you thinking specifically of one of criminal law court cases and private law cases, or both? And are you considering only civil law or also common law? In general, the interpretation of the law, which as any expression in natural language is vague and ambiguous, is created and refined by judges. I don't see how this task could be taken over by a computer program; we are nowhere near automating and codifying the creativity required for this; nor can we construct a program that displays an acceptable understanding of natural language like even a six-year old has. In many jurisdictions, the system has a jury, which would also need to be replaced by computers if the aim is to "eliminate human arbitrariness". Examples of vagueness are legal notions like due diligence – no investigation will be fully diligent, but where is the threshold between due and not due? Likewise for gross negligence, and beyond a reasonable doubt. To judge this, the trier of fact has to weigh the evidence, which can involve the weather, known predilections of the defendant, conflicting testimony of expert witnesses, and so on. How are you going to input that into the computer? And in Dutch law, for example, the law requires the judge, in sentencing, to consider "the person of the defendant", meaning that a repentant otherwise exemplary member of society should get off more lightly than an aloof cad for otherwise the same crime. Again the same problem.
Some work has been done on formalizing fragments of law code in formal logic, but (as far as I know) the scope of this has been extremely limited, and the results have not been promising. For a form of logic that has been used for this, see Deontic logic.  --Lambiam 01:05, 15 March 2008 (UTC)[reply]
There have been several science-fiction stories about computers as judges (including at least one by Isaac Asimov, I seem to remember). AnonMoos (talk) 03:37, 15 March 2008 (UTC)[reply]
You might be interested in the book Objectivity by Peter Galison and Lorraine Daston. One of the things it talks about is how the 19th century fad of "mechanical objectivity" (the belief that mechanized ways of getting data would someone produce something more "objective" and free of human influence) really fell out of favor among scientists in the 20th century, when it was clear that "judgment" and "experience" were often much more able to deal with the wide variety of difference in natural phenomena than was the mechanical means. I think you'd find it relevant to your query—if even the hard sciences don't think that pure automation by expert systems is a great reflection of objectivity and accuracy, surely something as plastic as legal judgment is not going to be able to do that any better by introducing automation. --98.217.18.109 (talk) 17:07, 15 March 2008 (UTC)[reply]
It's no accident that "code" can refer both to software and to organized collections of laws. Judges exist in the system precisely to administer the part of the law that we don't know how to codify. Programming a computer to do the task of a judge is equivalent to codifying every aspect of the law that remains uncodified, and if we could do that then the computer's program would be the legal code and the notion of a judge would become meaningless. All of the responses so far are just roundabout ways of saying "we don't know how to do AI". -- BenRG (talk) 13:26, 16 March 2008 (UTC)[reply]

British Monarchy - Line of Succession

I must admit that I am always averse to posting on the Miscellaneous Help Desk. I mean, the name itself is so trivializing ... and Lord knows what type of jetsom and flotsom congregate over there. That being said, it seems to me that the Humanities folk have their finger on the pulse of this topic. Hence, I post here. Is there any "real" reason why they calculate the heir to the throne of the British monarchy way out to like 800+ lines of succession or so? I mean, is there any real, practical, legal, cultural, logistic, logical reason that this has any importance? Or is it just name-dropping and ego-building. I mean, to say that "I am 857th in line to the British throne" sounds both impressive and laughable at the same time. We all know that we will only ever go out to maybe the 2nd, 3rd, 4th, 5th line perhaps, practically speaking. So, why on earth do "they" (whoever that is) calculate such a long line? Thanks. (Joseph A. Spadaro (talk) 23:04, 14 March 2008 (UTC))[reply]

I've never heard of anyone saying they are the XXXth or even XXth. or Xth. in line. Perhaps you misunderstood a joke. 80.2.196.223 (talk) 23:12, 14 March 2008 (UTC)[reply]
Are you kidding me? They calculated it down to Number 1,389 in Britain. They got it down to a science, it seems. See: Line of succession to the British Throne. (Joseph A. Spadaro (talk) 23:17, 14 March 2008 (UTC))[reply]
Someone might have calculated them, but it is an extremely obscure part of British life. What you are supposing would be like expecting most Americans to have an extremely detailed knowledge of for example baseball statistics - although detailed baseball statistics might be published in some obscure book, its a big leap to imagine that therefore many Americans take an interest in them or can quote them to you. 80.2.196.223 (talk) 23:32, 14 March 2008 (UTC)[reply]
I don't understand your reply to my question. Yes, I know that someone calculated them. Yes, I can imagine it's rather obscure at best. My question was -- is there any real reason why this is done ... legal, financial, etc. ... or is it just ego building and trivial? How does my question suppose that (analygously) Americans would be able to quote or be interested in detailed baseball stats? I don't follow your post at all. (Joseph A. Spadaro (talk) 23:37, 14 March 2008 (UTC))[reply]
Ahh, well there isn't any real and logical reason for these calculations. However it is a unquestionable fact that several persons of this world are simply interested in such topics (royalty and nobility, the lives of the rich and famous) and in the British monarchy in particular. This creates a large market and there are plenty of woman's magazines that are simply full with such garba... heeeh specialized information. I honestly suspect that the overwhelming majority of the persons of this particular list couldn't care less and probably will not introduce themselves as "I am 857th in line to the British throne". Flamarande (talk) 23:36, 14 March 2008 (UTC)[reply]
It’s important to know who’s in the immediate line, but I agree that calculating it down to the nth degree is absurd, and nobody does it to that extent except Wikipedia. Which is why I and others have asked for explanations as how what Wikipedia has done doesn’t constitute original research. I haven’t been satisfied with the answers given to date, but it’s a low priority for me so I’ve let it lie. -- JackofOz (talk) 00:08, 15 March 2008 (UTC)[reply]
Of course there's a good reason for this list. If you're on it, you need to know which and how many people you need to assassinate. (And your hobbies might include collaborating at Wikipedia to come up with the list. Hmmm.) See Kind Hearts and Coronets and the history of any royal dynasty.4.234.99.243 (talk) 00:47, 15 March 2008 (UTC)[reply]
Perhaps Number 857 dreams of becoming King Ralph if the higher Numbers have Rather a Nasty Accident. Edison (talk) 02:38, 15 March 2008 (UTC)[reply]

The list at Line of succession to the British Throne is NOT any kind of official UK royal family or UK government production -- beyond the first 40 or so names on the list, it's based pretty much purely on efforts of private unofficial individual personal initiative (not really original to Wikipedia, however -- similar lists have been circulating on the Internet since before Wikipedia existed, as seen in some of the external links at the bottom of the article Line of succession to the British Throne). AnonMoos (talk) 03:33, 15 March 2008 (UTC)[reply]

Of course there's no practical significance to knowing one's exact place in the line of succession (once you get two or three heartbeats away), but I imagine there might be a certain frisson in knowing that one is the last person in the line of succession (who seems to be one "Karin Vogel", born 1973, unless she's had children....) - Nunh-huh 03:42, 15 March 2008 (UTC)[reply]
Why are you saying that Karin Vogel is "last"? Isn't the # 1389 person on the list "last"? At least until a new birth occurs? (Joseph A. Spadaro (talk) 05:05, 15 March 2008 (UTC))[reply]
There are far more than 1389 people in the line of succession. Karin Vogel is 4900+. The last person is the youngest daughter of the youngest daughter, etc... that is, to find the youngest person you start at the Electress Sophia and reverse the usual rule of male preference first born, and take the youngest born of each family, females before males. And that works out to Karin Vogel (or her youngest daughter, if she's had one yet :)... In that way, you can find the last without knowing all the intervening people's positions. - Nunh-huh 05:28, 15 March 2008 (UTC)[reply]
There you go Joseph, you started out asking why anybody would calculate the line of succession because it seems so pointless and now you are so interested in why Karin Vogel is not the 'last in line'. It is so interesting isn't it. I would estimate that 99.9% of British people haven't the slightest idea or interest about the line of succession beyond Prince William. Richard Avery (talk) 16:54, 15 March 2008 (UTC)[reply]
Agreed. Very good point, Richard Avery. As my original post states ... it is both impressive and laughable at the same time. I can't help but to be laughing and smirking at how ridiculous it is (to be # 726,354 in line) ... at the same time, I am oddly impressed and slightly envious. Fascinating, to say the least. Thanks. (Joseph A. Spadaro (talk) 07:58, 17 March 2008 (UTC))[reply]
But now that you're initiated into the mysteries, you must be careful to demonstrate your knowledge... You know there are only roughly 4900 in the succession, so no one can be #726,354! :). - Nunh-huh 22:57, 17 March 2008 (UTC)[reply]
Some royal houses (Denmark?) confine the succession to a very small number, e.g. descendants of the current monarch's grandparent, so their lists are too short to be interesting. In others, the potential heirs are a huge fuzzy swarm, with the actual succession dependent on the decision of a council and thus unpredictable. In contrast to both, I suspect that the British succession attracts the attention of nerds because it is nontrivial but also finite and (in principle) knowable, being confined by legislation in 1701 to the descendants of one woman, in a rigid sequence. —Tamfang (talk) 01:31, 19 March 2008 (UTC)[reply]

How does economics explain the earnings of supermodels?

I'm puzzled why supermodels, for example, earn a lot of money. The work they do does not require a lot of training, its not difficult, and a lot of people have equal skills or ability and could easily replace them. For example this model Gisele Bündchen has only slighlty better than average looks (although I've never approved of body fascism or the cult of celebrity) yet earns millions a year apparantly. Economic theory would seem to predict they would not earn much. 80.2.200.28 (talk) 23:03, 14 March 2008 (UTC)[reply]

There is a flaw in the premise of your question. You say "a lot of people have equal skills or ability and could easily replace them." First, that's not true. A lot of people can not replace them. Second, I don't believe that it is the "skills and abilities" per se that drive these salaries. It is the economics of the model's ability to "draw" / attract the correct demographic crowd (customer) and to represent the face of the product. Let's face it, there are a million actors better at acting than, say, Tom Cruise. But it is indisputable that Tom Cruise can draw in millions of fans to his films (and millions of dollars to the film company). Hence, they will pay Tom Cruise $10 million to act in that film (for example). Same idea with these supermodels, I believe. To rely on a dated example: Perhaps Calvin Klein paid Brooke Shields $10 million dollars to appear in the "Nothing comes between me and my Calvins" ads in the 1980s ... but they did so only because the accountants / financial wizards / execs at Calvin knew that paying her $10 million would generate $100 million in sales for them. So, yes, they are willing to spend $10 million to generate $100 million, thereby keeping the profit of $90 million. That's the basic idea, I believe. And why does/did America love the ads and love Brooke so much back in the 80s? Who knows? That is the vagary / caprice / whim of the consumer public. So, if Calvin can bet on a sure thing, they'd be fools not to go for it. (Joseph A. Spadaro (talk) 23:14, 14 March 2008 (UTC))[reply]
Also, I don't believe that their "work" is as easy as it seems on the surface. We probably don't see the behind the scenes stuff. (Those ignorant of film-making might say "Geez, how long can it possibly take to make a 2 hour film? ... how can it possibly take 7 months?") Those "top" supermodels (and remember, they represent 0.000000000001% of all models) need to be on top of their game 24/7 ... they probably have to watch what they eat, their sleep, diet, exercise, nutrition, etc., etc., etc., non-stop 24/7. Could you live like that? I can't. Also, don't forget ... they are "in and out" of the industry quicker than a flash. They can only earn these gazillion dollars for a few years before the next younger / prettier / trendier thing comes along. Rather analygous to pro athletes. They are in top shape, get used up, and then get spit out when the younger generation athlete comes on the scene. So, for that brief window of opportunity, they need/can command a high salary --- but it's relatively short-lived. I'm thinking. (Joseph A. Spadaro (talk) 23:28, 14 March 2008 (UTC))[reply]
I was, and to a certain extent still am, a freelance photographer and filmmaker. I can assure you that there are literaly thousands of attractive would-be models; also that there is little effort or skill involved. And staying fit, slim, and healthy is the norm for intelligent young people in at least Europe. I admit it helps to be young, but when I was a student nearly all the female students were good-looking enough to be models if they had wished - with just the few fat ones excluded. 80.2.196.223 (talk) 23:38, 14 March 2008 (UTC)[reply]
For some reason, I was thinking along the lines of runway models --- not print models --- when I read the original question. Nonetheless, I am sure it takes great effort to "look beautiful" each and every day. They probably have to live in the gym 24/7 and most normal everyday people can barely drag themselves to a gym 1 day a week or probably don't even belong to a gym. Right? There is hard work on the part of a model. It's just different hard work than, say, a construction worker or a coal miner or a day care teacher. With the beautiful model, you don't see that "hard work" during the course of the photo shoot. It's how the model lives / eats / breathes the other 23 hours of the day that enable her to look beautiful for that one hour of photo shooting. Also -- the proof is in the pudding. These top business execs are in business ... so we can presume that they indeed know what they are doing (business-wise). They are paying Model X multi-million dollars only because there is a good sound financial business reason to do so --- and for no other reason. No two ways around that. After all, they are in business, not charity. If "anyone" would do (as their model), and the Company could drastically reduce their expense yet still maintain / generate the exorbitant revenues, you can bet your a$$ that their pencil-pushing accountants would go out and hire that cheaper "anyone" model. But, they don't. Proof is in the pudding. (Joseph A. Spadaro (talk) 23:55, 14 March 2008 (UTC))[reply]
I both dated and met a number of models in my youth, and none of them did any work-outs as you speculate, or did anything involving much effort. The make-up skills they had were nothing more than most young women could do: remember that professional make-up artists are used in photographic shoots or fashion shows. While I note your comments about "business execs" paying them a lot of money because they think that doing so will be a good investment, this does not explain how supermodels come to be worth such large amounts of money. 80.2.196.223 (talk) 00:17, 15 March 2008 (UTC)[reply]
Hmm. Well, perhaps it is circular to this extent. Supermodels are worth a lot of money because top business execs say so. And, because top business execs say so, it is. And, knowing that "it is the case", the supermodels can therefore command the high salaries. And, yes, that's circular. No offense ... but you keep referring back to the work, skills, abilities of the model -- as if the model is fully in control of these. I think it's more nebulous -- meaning that the public / consumer / customer is drawn to that "face" which is the representative of the company. In other words, hypothetically ... if we (as a society) were all drawn to / attracted to fat old overweight women in their 70's, then that's who would be the supermodels generating our consumer dollars and commanding high supermodel salaries. In fact, we actually do have that, of sorts. I don't know if you recall when that actor Mr. Whipple died (Please don't squeeze the Charmin.) He was an aging, old, unattractive "model" / spokesman in his 60's or 70's for the product. He said he worked like 2-3 weeks a year and made millions of dollars. So, for that market (toilet paper), he was the "supermodel" and commanded that high fee. And the supermodels you refer to are doing the same thing ... just in a different product market. Either way, the person (Mr. Whipple or gorgeous supermodel or Tom Cruise) has the ability to generate zillions of dollars for the product ... thus, the company is willing to pay accordingly. Why these individuals have this ability (to generate this revenue base for the company) is not a question of economics --- but of sociology / psychology ... I would think? (Joseph A. Spadaro (talk) 00:35, 15 March 2008 (UTC))[reply]
Another point: the original poster suggested that Ms. Bundchen has only 'slightly better than average looks'. Granting that is true, perhaps Ms. Bundchen has much higher-than-average confidence levels, which I'm sure is critical for selling fashion. Vranak (talk) 23:31, 14 March 2008 (UTC)[reply]

Some of the arguement put forward is of the form "supermodels get paid a lot because people pay them a lot" or "supermodels get paid a lot because they are at the top of their profession", which is like saying "some people are very tall because they are at the top of the height range" - it is circular thinking that does not explain anything. Are there any economists who could comment please? 80.2.196.223 (talk) 23:46, 14 March 2008 (UTC)[reply]

Perhaps you are unaware that all statements are either tautological (circular, as you said), or a bald attempt to convince. If you haven't been convinced then you disregard an argument as circular. But it is only circular to one who remains unconvinced. Let's see, I just went in three circles I believe. Vranak (talk) 03:20, 15 March 2008 (UTC)[reply]
I wrote circular thinking - what I meant was paraphrasing. 80.0.110.219 (talk) 12:52, 15 March 2008 (UTC)[reply]
No one above stated anything close to your circular "they are at the top because they are at the top" argument. The economics has been explained. In dollars and sense, Model X costs our Company $10 million and simultaneously generates $100 million ... so we get $90 million profit if we hire Model X. (Model X is called a supermodel who commands a high price.) On the other hand, Model Y costs our Company only $1 million but simultaneously generates only $5 million ... so we get only $4 million profit if we hire Model Y. (Model Y is called a "regular anybody" who commands a low price.) Therefore, let's hire Model X. Yes, it will cost us more --- but the benefits far outweigh the costs. Pound for pound, it is "cheaper" to hire Model X than Model Y, after you consider the net effect (profit) of the ad / spot. That is the economics / financial thinking behind it. I more or less explained all this in prior posts, or words to this effect. (Joseph A. Spadaro (talk) 00:06, 15 March 2008 (UTC))[reply]
I'll second Mr. Spadaro's point: a worker who (for whatever reason) is linked to the success of a project gains in reputation, and is often held in higher regard than other, less successful competitors. That's how I justified fame, anyway, until recently...T-T-Teeth (t-t-talk) 00:29, 15 March 2008 (UTC)[reply]
I'm not an economist, but I would say that supermodels are a vital part of the presentation and marketing of high end, niche products. As such, they might influence sales figures, and as such their earnings would be proportionate to the revenues of the companies. It's probably similar to football (soccer) players. By playing the sport they make money for the club in prize money, sponsorship money, tv rights, etcetera, and as such they get a more or less proportionate share of the money they amass for their respective clubs. AecisBrievenbus 00:05, 15 March 2008 (UTC)[reply]
How do you explain them getting high incomes? Accountants and (in a sporting context) groundmen do an equally vital job, yet get paid far less, yet are more highly skilled and are less replacemable. And in the 1960s or 70s footballers used to get paid very little, yet now they get paid millions. Why the change? How is it explained by economic theory? 80.2.196.223 (talk) 00:33, 15 March 2008 (UTC)[reply]
You keep getting bogged down with concepts like "vital job or not", "highly skilled or not", "easily replaceable or not". You are focusing on the labor market model. I think that's the wrong lens to view this phenomenon. This is the economics of it: these people called supermodels -- for whatever reason (good reasons, bad reasons, or indifferent reasons) -- can and do generate great revenue for the business ... thus, the business pays them accordingly. That's the economics of it --- like it or not. The question you want answered, really, is: Why do these supermodels generate such revenue from the consumer public? I do not believe that is a question of economics --- but perhaps sociology, psychology, marketing, etc. The economics are clear. It's these other nebulous "x factors" that really drive your question. It's like saying why does The Simpsons TV cartoon show stay on air for 20 years and those voice actors get paid a zillion dollars? (Probably "any one" can voice those cartoons, in all reality.) Why does a TV show like Freinds attract zillions of viewers and thus zillions of advertisers and thus account for the actor's zillion dollar salaries? Why do people love these TV shows? Who knows? That is the psychology / sociology behind it ... not the economics that the original poster wants to know more about. I think. (Joseph A. Spadaro (talk) 00:49, 15 March 2008 (UTC))[reply]
By the way, I am not a model and am probably ignorant of the field ... but I would have to guess that being a model (and a good one, at that) must require more than just "looking pretty". Otherwise, as the original post states, anyone can do it ... and thus anyone would do it. But, that's not the reality. (Joseph A. Spadaro (talk) 00:21, 15 March 2008 (UTC))[reply]
Here's another idea: supermodels are essentially whores for products. They lend their face, voice, and false enthusiasm in the service of some soulless company's bottom line. The more beautiful a girl is, the purer her soul is, and the less likely she'll be a whore for suits. So getting a Giselle Bundchen to model your clothes in a magazine is going to cost you a lot because it costs Ms. Bundchen her integrity. Vranak (talk) 03:26, 15 March 2008 (UTC)[reply]
Interesting and amusing point. 80.0.110.219 (talk) 12:52, 15 March 2008 (UTC)[reply]
From what y'all saying, it does seem to be just an exchange of capital in a mutual brand building exercise – the model exchanges her individual capital (sometimes social capital) in a way that makes other people want it for their market advantage and therefore monetary gain and ultimately his/her financial reward and there's evidently a scale of sellers and buyers from down to up market. See also Endorsement (advertising). Interestingly the trend has been to replace models with celebrities to sell cosmetics, as catwalk participants, etcetera. Julia Rossi (talk) 04:08, 15 March 2008 (UTC)[reply]

Do supermodels earn a lot of money, in the sense of being a supernormal profit? I know that they would earn quite a bit from advertising. In that case, the client is paying for the goodwill generated by the association of the product with the supermodel.

Assuming that they get paid a lot - why then are not people flocking to be supermodels? I think the answer is: they are. However, the barrier to entry of being a supermodel occurs at the brand establishment stage. It costs a great deal for a supermodel to be marketed successfully as a supermodel. The market has a limited appetite for such brand names.

In terms of the relation between supermodel and advertiser, I think of "supermodel brand" as the goods supplied by the supermodel, and demanded by the advertiser.

However, an "ingredient" of the "supermodel brand" is "public goodwill", supplied by the public and demanded by the supermodel/promoter. The supply of public goodwill is limited, because there is a limited number of celebrities whom the public will love - an analogy perhaps is the limited amount of space in the entertainment section of a newspaper.

So only a limited number of supermodels will attain sufficient public goodwill to pass a threshold level. I guess this can be conceived as a maximum level of risk for large brands. Once a model is established enough that the risk of associating with the model is below that maximal level, they enter the pool that shares in the aggregated advertising budget of major brands - which, as has been said above, is determined by the value added to the client's brand by association with the super model. This will seem high, but is an equilibrium position given the initial constraint (the public will love only so many models). --PalaceGuard008 (Talk) 05:23, 15 March 2008 (UTC)[reply]

Thanks for a causal economic-theory type answer. I once dated the sister of a successful magazine cover-model, and she said the other less successful models were very unpleasant to her. When I was a successful hard-working photographic student I also had to endure a lot of unpleasantness and back-stabbing from the machismo students. So it may be common in the arts - they do not attract people with the selfless integrity of true professionals. That is one barrier to entry that people overcome by luck or whatever. I suppose it is like actors - hundreds of people may apply for a job, the person that gets it gets audience recognition which appears to be worth money, despite many other actors being able to do just as good a job - an example of a winner takes all phenomonen. 80.0.110.219 (talk) 12:52, 15 March 2008 (UTC)[reply]


March 15

How religious was Benjamin Franklin?

What were his views, beliefs, etc? Did he attend church, mosque? 66.239.186.98 (talk) 13:11, 18 March 2008 (UTC)[reply]

Please read our article on Benjamin Franklin. As a Deist, he wouldn't have been attending either a church or a mosque. He lived with Deborah Read, mother of some of his children, without benefit of clergy. - Nunh-huh 08:59, 19 March 2008 (UTC)[reply]

Obligatory rights

Is there a name for rights that are obligatory? Like a minimal wage, life or other rights that may not be waived, not even if the person who holds these rights wants to. Mr.K. (talk) 00:03, 15 March 2008 (UTC)[reply]

What do you mean by "waived"? How do you "waive" your life? The closest thing you are referring to is entering into a contract, where a party to the contract attempts to "waive" something that is non-waivable. Or, in other words, the party is attempting to agree to something that is illegal to agree to, to begin with. That is, I cannot enter into a contract for less than minimum wage. Because that would be an illegal agreement. I cannot enter into a contract where I agree to give up my life in exchange for $1 million. Because that would be an illegal agreement. Such contracts are called invalid or void. They are illegal contracts and, thus, not "contracts" at all. The public policy is to not get in a sticky predicament where the government is enforcing illegal bargains and essentially condoning / supporting / assisting illegal activity. Also, there are issues of fraud / duress / unequal bargaining power / coercion to deal with --- which these restrictions help to eliminate or avoid. Does this help? (Joseph A. Spadaro (talk) 00:16, 15 March 2008 (UTC))[reply]
I meant 'waive your right to life'. And yes, your answer do help.Mr.K. (talk) 00:46, 15 March 2008 (UTC)[reply]
Inalienable. See Inalienable rights. 4.234.99.243 (talk) 00:36, 15 March 2008 (UTC)[reply]
That is. Mr.K. (talk) 00:46, 15 March 2008 (UTC)[reply]
A right is merely something that is not denied you by the society in which you live. E.g. in Myanmar you have no right to vote. - Kittybrewster 11:34, 15 March 2008 (UTC)[reply]
Maybe not a legal right, or perhaps a legal incapacity, but many would argue that the moral right to vote is the hallmark of a free society and is inalienable, regardless of what actual restrictions may apply in particular cases. -- JackofOz (talk) 22:00, 15 March 2008 (UTC)[reply]
I would call it rather the right not to be governed without the opportunity to vote. —Tamfang (talk) 01:45, 19 March 2008 (UTC)[reply]

I think inalienable rights means that other people can't take them from you. AllenHansen (talk) 09:28, 16 March 2008 (UTC)[reply]

Yes, inalienable essentially means 'non-transferable'; it's from aliẽnum which means 'belonging to another'. You can buy my chattels, and thereafter enjoy property rights in them exactly as I had; but it is impossible, not merely illegal, to add my life or liberty to your own. —Tamfang (talk) 01:45, 19 March 2008 (UTC)[reply]

white, black and hispanics community

Is there a place in U.S. that where white people, black people and Hispanic people live together? —Preceding unsigned comment added by Don Mustafa (talkcontribs) 01:01, 15 March 2008 (UTC)[reply]

Yeah, almost all of them. Corvus cornixtalk 01:41, 15 March 2008 (UTC)[reply]
Um, that's what makes us the United States. --~~MusicalConnoisseur~~ Got Classical? 06:28, 15 March 2008 (UTC)[reply]
Think OP was after a specific example of a harmonious community, and not the ghettos and almost-all-black/white/hispanic cities that we see on TV so often.
Which reminds me of an imported American TV series -- ten years ago?? -- that had all black characters. At the time I had no idea if that was reflecting reality, or if it was deliberate. And I still don't. --PalaceGuard008 (Talk) 11:50, 15 March 2008 (UTC)[reply]
Birds of a feather, sure, but Corvus was right. Go to any, and I mean any, Little League game in the United States and see interracial harmony in the wild. The fistfights are over the plate umpire's calls, not race. Foreigners have a mistaken idea about this place, which isn't surprising since we Americans also do. There are ghettos and gangs, and Klan rallies, but outside of the hot spots we couldn't care less what color you are. Foreigners see Obama's candidacy as a sign of change being wrought, but it's actually not a sign of anything. The change has already happened. Not every American ideal is a forlorn dream; some of it works. --Milkbreath (talk) 15:15, 15 March 2008 (UTC)[reply]
Well, yes and no. Little league games are one thing, but most races live near other members of their own race. Much of this is socioeconomic as well. It's not just hot spots where clashes happen—there are more subtle ways in which disharmony is all over the place, internalized, unchallenged by any. Blacks aren't forced to ride at the back of the bus, to use a very literal example, but in my city, they always do. It's self segregation; not as vile or crude as enforced segregation, but it's still there. The "hot spots" are just the places where people end up challenging the passive order, for better or worse. The lines drawn, though, are rarely among purely racial grounds—economics seems even more important, but disentangling that from race can be pretty tough. Maybe I'm remembering wrong, but little league games where I came from were a largely middle class phenomena—if you take them as an example of how races interact in the United States, you're leaving out a lot. --98.217.18.109 (talk) 20:12, 15 March 2008 (UTC)[reply]
There are three metro areas that appear on the list of 10 least-segregated cities for blacks and Hispanics from the 2000 census: Tampa Bay; Oakland/East Bay, Calif.; and Seattle. -- Mwalcoff (talk) 04:25, 16 March 2008 (UTC)[reply]
Having grown up in the East Bay, I can see unequivocally that the East Bay is as integrated as any community in the country. Corvus cornixtalk 04:44, 16 March 2008 (UTC)[reply]
That's where I live. I'm blond; my immediate neighbors speak Spanish; I look outside and see the Black kids giving the Irani(??) kids a ride in their toy car. Ethnic friction? We've got better things to worry about, like traffic and earthquakes. —Tamfang (talk) 01:54, 19 March 2008 (UTC)[reply]

Why did the Government do this? (Australian Government vetoing Oliphant's US Medal)

Reading the article on Australian physicist Mark Oliphant who worked on the atomic bomb, there's this: "His wartime work would have earned him a Congressional Medal of Freedom with Gold Palm, but the Australian government vetoed the honour." I was wondering why the Government took this step. Julia Rossi (talk) 03:16, 15 March 2008 (UTC)[reply]

His obituary in The Independent (found here) suggests that the Australian government declined the award on his behalf on the grouns that Australian citizens should not accept foreign honors. Carom (talk) 04:28, 15 March 2008 (UTC)[reply]
Thanks for the link Carom – I wasn't aware the Australian government had such a policy and I can't find on what grounds they held it. Will pop the cite into the article though. Julia Rossi (talk) 05:59, 15 March 2008 (UTC)[reply]
I see he was awarded the Hughes Medal by the Brits, without a hitch. Julia Rossi (talk) 06:12, 15 March 2008 (UTC)[reply]
He was awarded the Hughes Medal by the Royal Society, not the British government, and since the society's patron is the Queen of Australia that is not a "foreign honour". Malcolm XIV (talk) 10:49, 15 March 2008 (UTC)[reply]
There is not now, nor to my knowledge has there ever been, any Australian government policy that Australians "should not accept" foreign honours. Many foreign awards and honours have been bestowed on Australians over a long time, including the past few prime ministers. But the government can, effectively, veto the awarding of a foreign honour; in other words, acceptance of such an honour is subject to the government's agreement. Why they disagreed in Oliphant's case is anyone's guess, particularly given Australia's emerging strong bonds with the US during WW2. Maybe they didn't want to be seen to be condoning the nuclear attacks on Japan. -- JackofOz (talk) 21:53, 15 March 2008 (UTC)[reply]
In the Biographical Memoirs piece about Oliphant's death, it says that it was the "Australian government then in power" that said citizens couldn't accept foreign metals. Maybe that has some clue to it to people who know more about Australian politics than I.
The author that first discovered this was Oliphant's biographer Stewart Cockburn, in 1980. One could, I imagine, take a look at his biography; it might have more. --98.217.18.109 (talk) 23:55, 15 March 2008 (UTC)[reply]
Just on the Hughes Medal. Elizabeth II wears 16 different crowns. She's the patron of the Royal Society solely in her capacity as Queen Elizabeth II of the United Kingdom of Great Britain and Northern Ireland, not in any of the other 15 capacities. When she visits Australia; appoints the Australian Governor-General; performs acts overseas on behalf of the Australian government etc, she does so solely in her capacity as Queen of Australia, not in any of the other 15 capacities. And so on. So I would argue the Hughes Medal is a foreign honour, but one that the Australian government would have less of a problem with (well, no problem at all, actually) than, say, the North Korean Star of Perpetual Freedom and Democracy (I just made that up for the sake of argument, btw, so don't bother googling it). But the even more germane point is that, despite having royal patronage and the word "Royal" in its title, the Royal Society is still a private organisation, not an agency of the British government. Whatever policies past Australian governments may have had about the acceptance of foreign awards, they related to awards made by foreign governments, not by foreign private organisations. The Australian government would have no more power to prevent an Australian citizen being given an award from the Royal Society than it would to prevent them being awarded a Nobel prize, an Olympic medal or an Oscar. Governments can physically prevent their people from participating in the Olympics, in which case the question of a medal never arises. They can also physically prevent them from going to Sweden or Norway to personally receive their Nobel prize, but the award is still made in their name. For example, Boris Pasternak was effectively prevented by the USSR; he chose to decline it before it ever got to that point, but only because of the probable consequences on his return if he had gone. As far as the Nobel committee were concerned, though, the award was made to him by them, and it remained in his name despite his refusal of it, and it was ultimately presented to his family after his death. -- JackofOz (talk) 01:38, 16 March 2008 (UTC)[reply]
I take it Oliphant was ambivalent about the success of the bomb and distanced himself from military projects subsequently, but the "veto" suggested the government stepped in rather than acted on his behalf exactly, so it looks like I'll be thumbing through that biography – thanks for the reference. Thanks to everyone for building the picture, Julia Rossi (talk) 11:00, 16 March 2008 (UTC)[reply]

Did Milošević plan to attack Montenegro?

As you know the 9/11 Commission held hearings about that attack in 2004. During the hearings process, loads of Clinton administration officials were called. One of the officials called to testify was William Cohen, Clinton's Defense Secretary from 1997-2001. During his testimony he talked about the other challenges facing him, such as the Balkans. He made an interesting statement about a planned attack by Slobodan Milošević on Montenegro after the 1999 NATO air strikes. If the attack had succeeded, according to Secy. Cohen, he would used this attack to restart the war in Bosnia. This is what he said:

Following the war against Serbia over Kosovo, Milosevic prepared for a possible

blitzgrieg military action against Montenegro, which while federated with Serbia in a rump Yugoslavia was exercising increasing independence from Belgrade. The US European Command developed plans to defeat a Serb military move against Montenegro, which Milosevic would have used to reignite conflict in Bosnia. Concurrent with this, Milosevic sought to stage manage an election process to bolster his political position after his failure in Kosovo. But the process became a real contest, and effective support to the democratic opposition led to Milosevic’s ouster and then to his imprisonment in The Hague. This action prevented the fifth Balkan war of the decade, bringing to an end a series of wars that had killed hundreds of thousands, flooded Europe with millions of refugees, and threatened European stability and security at the very time that the collapse of the Soviet Union had created the opportunity to build (to quote President Bush Sr.) “a Europe whole and free” – an opportunity that we seized by supporting the enlargement of NATO and the European Union. Principals and Deputies actively guided this closing

phase of the Balkan wars.

To read this in context, go 9-11commission.gov. The section on the Balkans starts on p. 17 of the PDF.

I have wanted to use this as a source in articles on the topics mentioned, but my searches have turned up two articles about fears of a coup, not an attack. this BBC article from April 1999 was written during the air strikes, and falls out of the date range above. This CNN article from August 2000 also has fears of a coup. The third article from July I have found warns Montenegro could become the next Balkan hotspot. Because of the lack of info to back up what the Secy said and with how the Balkans are a hotspot here as well, are there any sources native to the region that could back up this quote? Or did I encounter a bunch of BS? - Thanks, Hoshie 06:16, 15 March 2008 (UTC)[reply]

The latter, I strongly suspect, Hoshie! Clio the Muse (talk) 03:19, 16 March 2008 (UTC)[reply]
Thanks for your help. I'm begining to agree with ya. If what Secy. Cohen did happen, I'm pretty sure we would have known it by now as this would have been great propaganda material for both Serbia and Montenegro in the years since. Also found it hard to believe that something that happened more than 10 years ago could be buried in a PDF file! - Thanks, Hoshie 23:02, 16 March 2008 (UTC)[reply]

Conscripted African soldiers

In discussing Nigeria and the First World war Professor Jide Osuntokun said that the more Africans were forcibly conscripted by the European powers than had beren pressed into slavery. I should like to know if this is true and what impact in general the world wars had on Africa? Thank you. Topseyturvey (talk) 06:18, 15 March 2008 (UTC)[reply]

The total number of Africans, including the Maghreb, who served in the military forces of the colonial powers is said to be around two million. Many, perhaps most, of these were porters and labourers rather than soldiers as such. The widespread use of forced labour must make this no more than a very rough estimate. But there was no single pan-African experience of the First World War, not even if we limit ourselves to the area south of the Sahara.
However, Nigerian experience in the Second World War shows how problematic these numbers can be. In theory only a few hundred Nigerians were conscripted into the military. The thousands of workers on bases on the Takoradi air route were, so I read, recruited voluntarily. But the hundred thousand or so Nigerians sent to work in the tin mining industry on the Jos Plateau from 1942 onwards were surely conscripts by any reasonable definition of the word, see Bevin Boys for example. As far as conscript workers in the First World War are concerned, the two million figure does include people from French Africa sent to work in France, but I suspect that it will not include any equivalents of the tin miners, people moved around within the colonial empires. On the whole, I feel that the professor is likely overstating the case, even if he means only the Atlantic slave trade, but numbers will probably never be knowable with any precision. Angus McLellan (Talk) 16:33, 15 March 2008 (UTC)[reply]

Topseyturvey, if I can just offer a slight correction to the terms of your question. What Professor Osuntokun actually said was the slave trade at its height never involved on an annual basis one-tenth of the men forcibly enlisted in the First World War, an altogether more accurate observation. There were some volunteers but most recruits were effectively obtained on much the same basis as the old slave trade: it was a task delegated to the tribal chiefs. In French West Africa the chiefs were asked to obtain a given quota of 'volunteers', with no questions asked about the manner in which these men were harvested. When conscription was introduced by the colonial authorities it resulted in what the Governor called a chasse â l'homme-a manhunt. Many mutilated themselves rather than go. In the British colony of Northern Rhodesia, where up to a third of the male population was recruited for military labour, the co-operation of the chiefs was to ensure their lasting unpopularity.

As far as the more general part of your question is concerned, the First World War might be said to mark the final stages in the Scramble for Africa, with major territorial changes arising from the peace, new forms of colonialism disguised in the fiction of the League of Nations mandate. The Second World War weakened and finally ended the whole imperial project, giving birth to new forms of African nationalism. Clio the Muse (talk) 03:15, 16 March 2008 (UTC)[reply]

Thank you both for your answers. Thank you also, Clio, for setting me straight on the professor's precise words. Topseyturvey (talk) 04:49, 16 March 2008 (UTC)[reply]

Champange with raw egg

I saw a film where a guy had a glass of champagne and he cracked an egg and added maybe the whole thing or just the yoke. The yoke was whole and the drink went unmixed before consumption. What is that about? --Seans Potato Business 07:41, 15 March 2008 (UTC)[reply]

Sounds like it may be a hangover cure? FiggyBee (talk) 09:51, 15 March 2008 (UTC)[reply]

A champagne flip. Gdr 11:17, 15 March 2008 (UTC)[reply]

Eggs don't have yoke, but yolk.Mr.K. (talk) 04:00, 16 March 2008 (UTC)[reply]

This is a scene from The Quiet Earth(1985)

Cawnpore Massacre

How true is it that the Cawnpore massacre contributed to the speedy defeat of the Indian Mutiny of 1857? 81.152.107.181 (talk) 08:00, 15 March 2008 (UTC)[reply]

Only in that it whipped up British public opinion against the rebels and inspired the brutal retaliatory campagin known as the Whirlwind. AllenHansen (talk) 23:59, 15 March 2008 (UTC)[reply]

It certainly made the troops fight with a more ferocious determination when the news got out. All across Bengal, British soldiers went into battle shouting, "Remember Cawnpore!" On at least two occasions the Sutherland Highlanders are known to have charged at the enemy with cries of primitive rage, not cheering, the usual practice among British soldiers hitherto. It was the memory, above all, of the bloody little handprints, those of the children, on the walls of Cawnpore. One officer said afterwards, "I have spared many a man in a fight, but I will never spare another." When one company at Cawnpore was asked for a volunteer hangman, never a popular duty, to execute the captured rebels, every man stepped forward. Some of the captured Hindus were made to lick an area of floor clean of blood before being hanged, to destroy their cast before their lives were ended. The Muslims were sewn into pigskins. Clio the Muse (talk) 02:44, 16 March 2008 (UTC)[reply]

Bahai

Can Bahai Faith be considered as a right and heavenly religion? —Preceding unsigned comment added by Meyabi (talkcontribs) 10:40, 15 March 2008 (UTC)[reply]

Bahá'í Faith.
Could you define "right and heavenly religion" - it seems a matter of opinion87.102.21.171 (talk) 12:50, 15 March 2008 (UTC)[reply]
The people who follow it certainly think so :) As the user above mentions, you need to define your criteria a little more explicitly. Zahakiel 14:33, 15 March 2008 (UTC)[reply]

I've always considered the inclusivity of the Baha'i rather arrogant. By saying that your religion is the logical synthesis of all other previous religious implies that all other religions are just fractions of their own one true religion. Ninebucks (talk) 21:06, 15 March 2008 (UTC)[reply]

Should you expect any less? Wouldn't it be ridiculous if they went, "well, our religion isn't quite as good as Islam and Christians do have more fun and the Buddhists leave us in the dust..." Even if you are a Deist, you assert that your religion is best, otherwise, why stick with it? AllenHansen (talk) 09:25, 16 March 2008 (UTC)[reply]

Chester U.K. Laws in Tudor(?) times

I understand that there was a law which made the killing of a welshman lawful if he was inside the walls of Chester town after nightfall. I think this was during the times of the Tudors rule in England/Wales. Can this be confirmed and when, if ever, it was repealed? 213.7.108.48 (talk) 10:46, 15 March 2008 (UTC)Brian B[reply]

True. See Chester#Present_day last paragraph. or search for "chester law welshman" for more details87.102.21.171 (talk) 12:56, 15 March 2008 (UTC)[reply]
See http://www.chester.gov.uk/tourism_and_leisure/culture_and_leisure/chester_history_and_heritage/history_facts.aspx for the actual conditions.87.102.21.171 (talk) 13:00, 15 March 2008 (UTC)[reply]
Supposedly, there's a similar law regarding York and Scotsmen. It all sounds a lot like urban legends to me. For example, this article used to mention the York version but it was removed as unverified (which makes me wonder if news.bbc is operating under Wikipedia rules).AlmostReadytoFly (talk) 14:05, 15 March 2008 (UTC)[reply]
New laws superscede old laws, which is why some funny old laws don't get repealed.87.102.21.171 (talk) 14:29, 15 March 2008 (UTC)[reply]
As to the BBC operating under Wikipedia rules, becoming familiar with Wikipedia has opened my eyes to how unreliable and open to errors every source is. Several times I have encountered errors on the BBC news site, and when it's mattered I've emailed them and they've updated it (like when they had David Cameron down as an MP in Oxford :0). It's like a really slow, inefficient version of editing Wikipedia... Once you start poking around, you realise you can find most of the sources most media outlets use for most of their stories, and that news is written by fallible writers who misinterpret, make typos and fail to dig far enough or corroborate facts more often than you'd expect. And trust what self-proclaimed 'experts' tell them too often. So yes, BBC news is operating under Wikipedia rules, it's just that becoming an editor is a bit more tricky. Skittle (talk) 16:29, 17 March 2008 (UTC)[reply]

Facts on the History of the community of Pudur Dravidians

Would anyone be able to help me to know everything about the these "Pudur Dravidians" who are believed to be be belong to the Pudur village in Nellore district of Andhra Pradesh State in India? I am keen to know their background from the begining to the present. —Preceding unsigned comment added by Chandrasekhar73 (talkcontribs) 12:09, 15 March 2008 (UTC)[reply]

"from the beginning"? Where do you stand on the Darwin heresy? :) —Tamfang (talk) 01:59, 19 March 2008 (UTC)[reply]

Opium and Kubla Kahn

Is there any evidence on the extent to which Samuel Taylor Coleridge's opium addiction influenced the composition of Kubla Kahn? —Preceding unsigned comment added by Old Miff (talkcontribs) 12:16, 15 March 2008 (UTC)[reply]

Kubla_Khan#Structure_and_theme "Coleridge claimed that the poem was inspired by an opium-induced dream (implicit in the poem's subtitle A Vision in a Dream) but that the composition was interrupted by a person from Porlock"
http://etext.lib.virginia.edu/stc/Coleridge/poems/notes.html#KublaKhan "..In consequence of a slight indisposition, an anodyne had been prescribed.." from notes by the author "This fragment with a good deal more, not recoverable, composed, in a sort of Reverie brought on by two grains of Opium taken to check a dysentery"
It's in the article.87.102.21.171 (talk) 12:47, 15 March 2008 (UTC)[reply]
While Coleridge was a heavy opium user I think we must treat his claim that Kubla Khan was inspired by an opium-induced dream with a pinch of salt. While the idea may have come to him in a dream it was probably inspired by the books he had read on the mongol empire. Also his story that he was interrupted (hence the poem is called 'a fragment') seems unlikely when you realise how technically complex the poem is. It is a very musical poem using inclosed assonance, rhyming of inclosed syllables and oscillation to create minor echoes and pattern. It is intricately built and not a fragment at all but 'a vision of creation and destruction, each complete' (Harold Bloom, The Visionary Company p.212) - I would be quite skeptical of Coleridge's claims... Lord Foppington (talk) 17:09, 15 March 2008 (UTC)[reply]
I don't think it's that unlikely - though the idea of coleridge 'spicing up' the tale of the creation of the poem is quite attractive, having no insight into his personality.
However the opiates (morphine at least) do cause vivid dreams of a fantastical nature - so the idea that use of opiates may have at least inspired him is not without merit. Maybve we have someone here with experience of opium who could comment on whether his tale of creation is at least plausible in terms of its drug use???87.102.2.103 (talk) 18:12, 15 March 2008 (UTC)[reply]
Opium? Good heavens, no; of course not-honestly! Oh, the memories of that Chinaman! Clio the Muse (talk) 01:53, 16 March 2008 (UTC)[reply]
As others have said, it will never be settled and we have nothing to go on but his word. I don't find it too unlikely however, particularly because of the musicality of the poem. In my experience, music and dreams go together. I would find it harder to believe if the poem was complex in terms of ideas, but clunky in terms of the sound. Skittle (talk) 16:21, 17 March 2008 (UTC)[reply]

It's really a matter that will never be resolved. The idea, perhaps; the composition, almost certainly not. It's as well always to distrust statements by drug addicts, and Coleridge is no exception here. And as for the Person from Porlock, well, it took another poet to nail that particular fancy to the floor, in words of sympathy from one artist to another;

As the truth is I think he was already stuck

With Kubla Khan...

I am hungry to be interrupted

For ever and ever amen

O Person from Porlock come quickly

And bring my thoughts to an end.

I know how she feels! Clio the Muse (talk) 01:53, 16 March 2008 (UTC)[reply]

abuse of women, forced fat

I read in a book called, "wake up, I'm fat", a small amount of information about people who prey on women who are slightly overweight and have a low self-esteem. They take them home and fatten them up using things like baby formula and other high calorie foods, then once they are too big to be independent, they dump them and move on starting the whole process again with someone else. I haven't been able to find any other info on it. Can you help? Bonny75.80.86.181 (talk) 13:49, 15 March 2008 (UTC)[reply]

There was a documentary on British TV a few years ago called Fat girls and their feeders. You could try googling that. 86.133.55.238 (talk) 19:13, 15 March 2008 (UTC)[reply]
See also Force-feeding, also known as gavage, which is used on girls in Mauritania before marriage - if they won't co-operate, they are locked up as prisoners and fed until they are plump enough to be wed. BrainyBabe (talk) 15:19, 17 March 2008 (UTC)[reply]

Stalingrad

How come the Germans were caught so completely at the Battle of Stalingrad? Edward Conan (talk) 14:16, 15 March 2008 (UTC)[reply]

Partly because of Hitler's "never retreat" fetish. AnonMoos (talk) 14:40, 15 March 2008 (UTC)[reply]
Fremde Heer OstFremde Heere Ost, the branch of German intelligence that dealt with Soviet capabilities and plans, was not always very good at its job. Its biggest failing was that it repeatedly underestimated the size of the Red Army. On 6 November 1942, not long before the beginning of Operation Uranus, FHO confidently announced that the Soviets had the capability to launch just one major operation in the coming winter of 1942–1943, and that the main target would be the 2nd Panzer Army3rd Panzer Army of Army Group Centre in the region of Rzhev. Now in part this was accurate. The Soviets did plan a major operation, Operation Mars, against Army Group Centre. This opened in early 1943 and was a total failure as the Germans had accurately predicted what was planned.
The problem, however, was that in concentrating on the Rzhev sector, the Germans ignored the warning signs in the south. Although the build-up for Operation Uranus was carried out in secrecy, it involved moving very large numbers of men, and tanks, and aeroplanes, and huge mountains of supplies. Although the Germans had no clear idea of the scale, they did know something was happening around Stalingrad. But because all eyes were on the city of Stalingrad, where the 6th Army was inching its way towards final victory, and on Rzhev, nothing much happened.
And it's not as if the Germans had many reserves anyway. Even if they had thought something like Operation Uranus was planned, what could they do? Army Group A was bogged down in the North Caucasus, and a retreat to a defensible position would be difficult. Withdrawing Army Group B from Stalingrad and the River Volga to the River Don couldn't happen until Army Group A had moved a long way north of its positions in early November and would have been very difficult to do without heavy loss. As AnonMoos said, Hitler was not likely to agree to a rapid withdrawal - it could easily have become a humiliating rout - to the Don, and certainly not when FHO has said that there is no real danger in the south. Angus McLellan (Talk) 17:27, 15 March 2008 (UTC)[reply]
You can't see something coming if you refuse to open up your eyes. Actually the article explains the issue quite nicely; the decisive factor was Hitler, and to a lesser degree Goering. The danger of the German Sixth army being trapped and isolated at Stalingrad had been seen by many German generals who advised not to send too many troops into Stalingrad in the 1st place, to reinforce the southern and northern flanks with plenty of troops and equipment, and to make a strategic retreat if necessary. The positions were being guarded by 2nd-rate Romanian, Italian, and Hungarian troops which were low on morale, badly equipped, and ill-trained. However Hitler (aka Marshal Shickelgrueber) refused to listen to their warnings and to make any kind of preparations. As the Soviet troops managed to to overrun the Southern and Northern shores there was still a reasonable chance of pulling off an orderly retreat. However fat drug-addict Goering loudly promised that the Luftwaffe was capable of supplying the encircled troops with everything necessary. Many officers of the Luftwaffe protested and pointed out that this was an impossible task but the Groesster Feldherr Aller Zeiten Adolf Hitler choose rather to listen to his fat friend. The result was that the encircled German troops were slowly starving, running low in ammunition, and freezing to death. Hitler didn't even give Paulus the order to retreat during Operation Wintergewitter. Flamarande (talk) 00:50, 16 March 2008 (UTC)[reply]

Oh, the answer is simple enough, Edward: the bridgeheads on the western bank of the Don had not been fully cleared, and the flanks of the Sixth Army were held by the weaker Romanian Third Army and Romanian Fourth Army. The Soviets pushed through these hinges with easy, cutting the Germans off in a huge pocket around Stalingrad in a matter of days. A break-out and strategic withdrawal might have been possible at an early stage, when the trap was still soft, so to speak, though it would have been an immensely difficult operation, requiring also the withdrawl of Army Group A from the southern front. It is almost certain that the Germans would have suffered a serious defeat, though not as serious as they suffered by remaining in place.

I note with interest that the old canard that Hitler was known as Schickelgruber has made a fresh appearance. He never was! Clio the Muse (talk) 01:24, 16 March 2008 (UTC)[reply]

When Herr Hitler escaped the bomb on July 21 he described his survival as providential. I think from a purely military point of view we can all agree with him. Certainly it would be most unfortunate if the allies were to be deprived in the closing phases of the struggle of that form of warlike genius by which Corporal Schickelgruber has so notably contributed to our victory. Winston Churchil, Speech in the House, Setember 1944 (inside of Maxims and Reflections of the Rt. Hon. Winston S. Churchill) Flamarande (talk) 01:48, 16 March 2008 (UTC) PS: I fully agree that several of Hitler's decisions helped the Allies immensely.[reply]
Propaganda is a little like Plutonium: it seems to have a virtually endless Half-life; in some minds at least! Clio the Muse (talk) 01:57, 16 March 2008 (UTC)[reply]
Using the surname "Shickelgruber" when speaking about Hitler is a joke, and was meant as such (and as a little hassling insult) by Churchill. One can always use it in the same fashion as I did. I never seriously said that Hitler was known as Schickelgruber. Flamarande (talk) 02:13, 16 March 2008 (UTC)[reply]
I'm sure it was, Flamarande. My comment was aimed at minds perhaps a little less subtle than yours! Clio the Muse (talk) 02:28, 16 March 2008 (UTC)[reply]

Term for stuff characters do that we don't see, during other parts of their lives

I placed this here because I was uncertain if it belonged in literature or entertainment (TV, movies, etc.) - but it got no real help in the Entertainment section.

Is there a term for the span of time and/or activities that it is normally presumed (by rational readers/viewers, anyway) a character does which are not seen by the reader, or observed by the viewer? It's not out of character - on the contrary, the person is assumed to still behave in character. I don't *think* it's off screen, though it could be; it just sounds weird because this applies to books as well as television - it's just more pronounced on TV becasue we see so little of people's lives. A sitcom, for instance, shows us only 30 minutes out of 168 hours each week that the people lived. A book shows us more, but still not all their waking moments. (Though tightly woven fiction can cover much of a person's day.)

For instance, in "A Christmas Carol," even as meticulous a writer as Charles Dickens doesn't mention a character going to the bathroom, but it's much more obvious that there are no bathrooms visible on board the USS Enterprise in the original Star Trek.

Suspention of disbelief is what you need to presume that people use the facilities in both universes, but that's what readers/viewers use, and I'm talking about the actual acts of going to the bathroom, eating, paying bills, and so on.4.68.248.130 (talk) 14:09, 15 March 2008 (UTC)[reply]

4.68.248.130 (talk) 14:18, 15 March 2008 (UTC)[reply]

I'm not sure that what you speak of has a technical term. Your mentioning of A Christmas Carol and Star Trek, however, implies that the reader or viewer believes that what they are reading/watching is real. Also if I were telling a verbal story about a specific event in my life I wouldn't mention what I ate or when I went to the toilet unless it was essential to the plot. Streamlining of the story is important otherwise we would lose interest quickly; the pieces missed out are either accepted or ignored. I hope this helps Lord Foppington (talk) 17:25, 15 March 2008 (UTC)[reply]
You might call it an ellipsis.  --Lambiam 17:29, 15 March 2008 (UTC)[reply]
I've never quite got around to reading it, but I'm told that Marcel Proust used something like 30 pages to describe merely turning over in bed, in À la recherche du temps perdu. Just imagine if he described every moment of his waking day in such glorious and interminable detail. It would expand from 7 volumes to 7,000! Then there's the next day, and another 7,000, and so on. Writers have to be vicious cherry pickers, and when it comes down to it they leave a HUGE amount of information out - but that's important because it's information that isn't relevant to their story. I recall the first time (and, indeed, one of the very few times) that I saw a character in a movie sitting on the toilet and engaged in the act of wiping their backside. It was Jane Fonda in the original (1977) version of Fun with Dick and Jane - and I thought, "Why the heck are they showing us this?". It was, in fact, utterly irrelevant to the story being depicted, and was just put in there for titillative effect, I suppose. Something that people would tend to notice (as I did) and tell their friends about (as I'm finally doing now, over 30 years after the event). I remember next to nothing about the movie now, except for this bit. So I suppose, for me, Jane wiping her ass has become the story. Just imagine if Proust had gone into such intimate detail about such matters. (Maybe not). -- JackofOz (talk) 21:29, 15 March 2008 (UTC)[reply]
James Joyce in Ulysses famously goes into the intimate detail you describe JackofOz - Leopold Bloom going to the toilet in the morning, Bloom and Stephen urinating in the back garden etc. I'm not sure such details are relevant to the story but I guess they are relevant to the novel as a whole? Lord Foppington (talk) 14:47, 16 March 2008 (UTC)[reply]
Have you noticed how people on TV or in films never have to go to the toilet? Only very rarely do they do any work. They virtually never watch TV. Clearly, they are not human. 80.0.108.245 (talk) 19:07, 16 March 2008 (UTC)[reply]
Thanks, Lambiam. Hmmm, since an ellipsis is the narrative device of omitting it, perhaps the time omitted would best be termed the charcter's "elliptic life." Hey, if this gets around, wikipedia may just get credit for a new word someday. :-) 63.3.19.1 (talk) 23:36, 16 March 2008 (UTC)[reply]
Actors are told their characters' backstory, the previous history that makes the person who they are, before we knew them. As a parallel construction, how about "sidestory" for all that life they live offscreen, simultaneous to our knowledge of them? BrainyBabe (talk) 15:28, 17 March 2008 (UTC)[reply]
Thanks for that, too - a "sidestory" sounds good, too. Of course, the backstory may include mroe thant he actor is told, too - he may be told certain things for not others about his childhood, for instance. But, that could still be part of the backstory.
I wonder what the post-fiction life would be called, then - "front story" sounds like something for a coverup. :-) Although, that is where an adjective that goes with "epilogue" would fit well. "Epilogic?"4.68.248.130 (talk) 01:52, 20 March 2008 (UTC)[reply]

Can I edit?

Hi everyone, I'm Iranian living in Argentina, I have seen the article of Age of majority and saw Iran=18. It's wrong, in Iran, the age of majority is 15. Can I edit this? 190.49.116.210 (talk) 17:16, 15 March 2008 (UTC)[reply]

Yes, you are allowed to edit anything on here, but if you do not have a valid screen name you cannot log in. It is free to set up an account, though. I only mention this because, for reasons beyond my comprehension, anonymous computers go by different IP addresses, and the IP address changes, seemingly at random. So, you may be blocked from editing only if you are anonymous and the system somehow thinks your IP address is one that did something bad earlier.
Then again, maybe that's only a problem with America Online users like myself.209.244.30.221 (talk) 17:25, 15 March 2008 (UTC)[reply]
If you change the information in the article, please explain the edit briefly in the edit summary window, and please note a reference or reliable source for the information. It would be great if you would establish a screen name ((could be a nickname or other screen name other than your own). Thanks for contributing. Edison (talk) 19:32, 15 March 2008 (UTC)[reply]

British throne taken by monarch from elsewhere - what happens?

I read the query about the British royal family from Nov. 2, 2007, so I'm somewhat educated about how Parliament might act, etc.; my question deals with the specifics between countries if the monarch of one should happen to become monarch of another - using Great Britain as a sort of guide. Harald V of Norway is only 62nd in line to the throne of Britain, as noted in the article.

So, trying to stay off the hypothetical, is there a plan in place? Would Parliament get involved and decide if Harald should remain king? Parliament would have to act if he were to then abdicate, I know from the earlier query. But, it does seem like that's what would be most prudent - since Harald's children are next, he would probably just let one of them become reigning monarch of Great Britain, and the other reigning monarch of Norway. I mean, I doubt that Britain and Norway wound engage in an act of union at this point. (Though it would be an interesting way to seal Britain's total submission to the European Union, if they were to combine with Norway.)209.244.30.221 (talk) 17:52, 15 March 2008 (UTC)[reply]

Norway is not a member state of the EU. Malcolm XIV (talk) 20:09, 15 March 2008 (UTC)[reply]

209.244, I think you should understand that a monarch of one country becoming the monarch of another country does not necessarily entail political union between the two. Between 1603 and 1649 and then again between 1660 and 1707 Scotland and England had the same monarchs but still continued to have a separate political existence. Similarly, between 1714 and 1837 the kings of Great Britain were also the Dukes of Hanover, though the two were entirely separate political entities. It might also be of interest to you that Norway was joined with Sweden for almost a hundred years in what was purely a personal union.

On the wider point Parliament would have to give its consent to any constitutional changes. Clio the Muse (talk) 00:57, 16 March 2008 (UTC)[reply]

How can there be constitutional changes when there is no written constitution? Edison (talk) 04:10, 16 March 2008 (UTC)[reply]
Edison, the UK has a constitution, however its an unwritten one. I'm guessing that the monarchy falls under this. Furthermore the Commonwealth realms would have to approve any personal union between the UK and Norway. - Thanks, Hoshie 06:37, 16 March 2008 (UTC)[reply]
Are you sure about that last comment, Hoshie? If all the others dropped dead and Harald V became the legitimate heir of QE2, then surely the law says he becomes King Harald of the UK at the moment of her death. The realms have no say about that, just as they have no say about Charles being the current heir. If it just so happens that the next UK monarch is already the monarch of another country, then a personal union is simply an effect of the law and can't be undone except by an amending law. -- JackofOz (talk) 06:57, 16 March 2008 (UTC)[reply]
My last comment comes from two articles that I remember reading, namely Statute of Westminster 1931 and Queen's Privy Council for Canada. If I read the germane sections correctly, it seems in my mind that a foreign monarch ascending to the British Throne would be such a moment. This is my own speculation, so if I made a logical error, I'm sorry for the mix up. - Thanks, Hoshie 08:50, 16 March 2008 (UTC)[reply]
It's easy to get confused. The Statute of Westminster said that "any alteration in the law touching the Succession to the Throne or the Royal Style and Titles shall hereafter require the assent as well of the Parliaments of all the Dominions as of the Parliament of the United Kingdom". The abdication of Edward VIII was such a case because kings cannot abdicate unilaterally but require the law to be changed to permit them. However the legitimate accession of Harald V of Norway as King Harald III (?) of the UK and Northern Ireland, not to mention King of Australia, New Zealand, Jamaica, Canada and all the others, is not a case of an alteration in the law, but of its culmination. Now, if it were considered just too damn odd to have a Norwegian reigning over Jamaica, and the Jamaicans wanted to choose somebody else closer to home, they'd have to either first withdraw from the Commonwealth, or get the agreement of all the other realms to changing the Law of Succession. -- JackofOz (talk) 17:16, 16 March 2008 (UTC)[reply]
Heh, Edward the Confessor isn't counted in the numbering, so I don't suppose either of the previous Harolds would be. Here's another funny thought – The name under which a monarch names may be new, e.g. the grandfather of the chap in question changed his name from Karl (of Denmark) to Håkon. Is there any precedent, in crown unions, for a monarch reigning under two different names, thus "Harald V of Norway alias Alexander IV of GBNI"? —Tamfang (talk) 02:12, 19 March 2008 (UTC)[reply]
To say the British constitution is "unwritten" is an exaggeration. Many written Acts are constitutional in substance, in the sense of defining how the state is organized and run – though they can be changed by a new Act of Parliament, unlike the US Constitution, whose amendment requires extra-legislative procedure, such as appointing judges to rationalize any violations. —Tamfang (talk) 02:22, 19 March 2008 (UTC)[reply]
Tamfang, I stand corrected on the above. However you are a bit off on amendments are made to the US Constitution. They can ether be proposed by Congress or by a convention called by the states. They are ratified by the state legislatures or by conventions called the states. For more see Article Five of the United States Constitution or here. In short, adding an amendment to the US Constitution does not require a judge's input. - Thanks, Hoshie 05:10, 20 March 2008 (UTC)[reply]
You describe the official procedure. (To pick a nit: the amending convention is not "called by the States", but by Congress when N states so demand.) I described the more usual practice, with an attempt at humor. I could have said: the US Constitution is amended by declaration that "The Constitution is not a suicide pact" (with which I'd agree, though with implications opposite to what is customarily meant) or "The Founders could not have foreseen this or that" (which is why they wrote Article V). —Tamfang (talk) 19:43, 20 March 2008 (UTC)[reply]
Thanks, Clio, I had thought that only shortly before 1707 did England and Sctoland have the same monarch. Interesting stuff. 63.3.19.1 (talk) 23:43, 16 March 2008 (UTC)[reply]
Yes, it is! Clio the Muse (talk) 02:31, 17 March 2008 (UTC)[reply]

The UK already shares a monarch with several other countries, including Canada, Australia and New Zealand, and it doesn't make much difference to the political situation. DJ Clayworth (talk) 14:44, 17 March 2008 (UTC)[reply]

I talked this over with a friend who knows such matters, and he said that in essence, a personal union would arise normally; the countries would share a king but would be separate countries as were England and Scotland before they were united (and similar to the UK and Canada now, although they used to be more closely linked). The sovereignty should in theory pass instantly (or at the speed of light, if you prefer your technicalities to make physical sense) at the moment of death to the next heir. However, there is a problem in this particular case, which is that apparently Sweden's constitution forbids the monarch to be sovereign of another country without parliamentary permission. He wasn't quite sure what would practically happen in such a case; presumably he'd be king of both illegally until the Swedish parliament allowed it or told him to abdicate. Daniel (‽) 18:39, 17 March 2008 (UTC)[reply]

Women in the 18th century

Where can I find literature or information about womens status in the 18th century? - Kittybrewster 19:19, 15 March 2008 (UTC)[reply]

Vivien Jones (ed.), Women in the Eighteenth Century: Constructions of Femininity (Routledge, 1990) is an excellent collection of essays on the subject Lord Foppington (talk) 19:45, 15 March 2008 (UTC)[reply]

My goodness, there is so much! On the assumption, Kittybrewster, it is the status of women in England at the time that you are interested in, I would add the following to Lord Foppington's recommendations, in just the order they come into my head;

  • Women in England, 1760-1914 by Susie Steinbach.
  • Wives and Daughters: Women and Children in the Georgian Country House by Joanna Martin
  • Women in England, 1500-1760:A Social History by Anne Lawrence
  • Disorderly Women in Eighteenth-Century London: Prostitution and the Metropolis by Tony Henderson
  • Living by the Pen: Women Writers in the Eighteenth Century by Cheryl Turner
  • Women, Work and Sexual Politics in Eighteenth-Century England by Bridget Hill
  • The Autobiographical Subject: Gender and Ideology in Eighteenth Century England by Martha Nussbaum
  • Women and Urban Life in Eighteenth-Century England: On the Town by Rosemary Sweet and Penelope Lane
  • Consuming Subjects: British Women and Consumer Culture in the Eighteenth Century by Elizabeth Kowaleski-Wallace
  • The Invisible Woman: Aspects of Women's Work in Eighteenth Century Britain edited by J Carre, I Baudino and C Revauger
  • Gender in Eighteenth Century England: Roles, Representations and Responsibilities edited by H Barker and E Chalus

Phew! I could go on but I think that is quite enough. If you have any energy left you might wish to dip into some contemporary literature; and here I can think of no better guide than Jane Austen. There is also Alexander Pope's satirical Epistle to Miss Blount, published in 1760, which laments the plight of a country lady;

She went, to plain-work, and to purling brooks,

Old-fashioned halls, dull aunts, and croaking rooks:

She went from opera, park, assembly, play,

To morning-walks, and prayers three hours a day,

To pass her time 'twixt reading and Bohea,

To muse, and spill her solitary tea,

Or o'er cold coffee trifle with a spoon;

Count the slow clock, and dine exact at noon;

Divert her eyes with pictures in the fire,

Hum half a tune, tell stories to the squire;

Up to her godly garret after seven,

There starve and pray, for that's the wav to heaven.

Clio the Muse (talk) 00:40, 16 March 2008 (UTC)[reply]

Very nice, Clio. The line "Divert her eyes with pictures in the Are" intrigues me. It seems to be out of place in the rhyming sequence. And what's "the Are"? -- JackofOz (talk) 01:01, 16 March 2008 (UTC)[reply]
For Are read fire (is this an OCR error?). Algebraist 02:17, 16 March 2008 (UTC)[reply]
Ha! Ha! Yes, it is. Well spotted, Jack! It was a pure and simple typo, now corrected! Clio the Muse (talk)

Churchill and Russia

When and under what circumstances did Winston Churchill discover that Hitler planned to invade Russia? What action did he take? Sal Coats (talk) 21:17, 15 March 2008 (UTC)[reply]

The internet says April 1941 Churchill warned Stalin of an invasion threat. The information came from decoded german military intelligence transmissions.87.102.2.103 (talk) 22:14, 15 March 2008 (UTC)[reply]

Over the winter of 1940 and into the spring of 1941 Hitler had been building up his forces on the Soviet border in preparation for Operation Barbarossa. Before any attack could be mounted he had to secure his southern flank in the Balkans, which meant, amongst other things, drawing Yugoslavia into the Tripartite Pact. To exert as much pressure as possible three German panzer divisions in Poland were moved to the Balkans. After the Yugoslavs gave way on March 25 they were ordered back to their original positions. But following the Belgrade coup of 27 March, Hitler immediately countermanded these instructions in preparation for the Invasion of Yugoslavia. Hitler's order had been sent by a top-secret radio message, intercepted by British Intelligence at Bletchley Park. As soon as it was decrypted it was sent to Churchill, who at once understood the significance of the movements, not just for the attack on Yugoslavia but for the solid indications it provided that Russia was becoming a clear target. He at once sent the information to Stalin, saying, "Your Excellency will readily appreciate the significance of these facts." Fully aware of the dire prospects for England of a Soviet collapse he gave further instructions that all future intelligence of this nature be immediately forwarded to Stalin. Clio the Muse (talk) 23:55, 15 March 2008 (UTC)[reply]

Who demanded 1914 borders in east of Germany in German political scene after World War 2

I just found this election poster and I wondered if SPD demanded 1914 post-war ? I did heard that organisations of transfered Germans did make such claims but this poster of SPD clearly shows Germany in 1914 borders. [6]

The poster is from a webpage showing election posters in 1949-65 in Germany.[7]

Did SPD called for 1914 border in the East looking at the poster ? What other organisations called for such border ?--Molobo (talk) 21:32, 15 March 2008 (UTC)[reply]

For many years the SPD, along with other political parties in the Federal Republic of Germany, refused to recognise Russian and Polish occupation of the lands to the east of the Oder-Neisse Line, pushing for a return to the pre-war border of 1937-the Versailles border. So far as I am aware they never promoted a return to the border of 1914, a rather extreme revanchist position. There are two possibilities with that map. First, and most unlikely, it is simply an over-liberal application of the printer's yellow ink! Second, the poster has been misdated and was actually published in the old Kaiserreich. The fact that the print is partially in Gothic script would, on the face of it, seem to support such a conjecture, as this type-face was abandoned after the Second World War. I was slightly reluctant to suggest this on the assumption that the people who organised this display presumably had some degree of expertise in the matter. But I can think of nothing else. Clio the Muse (talk) 23:21, 15 March 2008 (UTC)[reply]
Didn't Hitler get rid of Gothic script in 1940-41? [8] , [9] , [10] , [11]. Edison (talk) 04:02, 16 March 2008 (UTC)[reply]
That's a Socialist poster. Perhaps they were using the Fraktur script to prove their distance from Nazism. -- Mwalcoff (talk) 04:17, 16 March 2008 (UTC)[reply]
The Gothic script (Fraktur / Schwabacher) was, though infrequently, used after WWII. Hermann Hesse insisted on his books to be printed using the font and a major paper,the FAZ, used a sort of Fraktur typeface for headers until quite recently.
I suggest that the usage of this particular font was indeed a subtle attempt to imply the continuance of pre-war Germany. Bear in mind that the division of Germany at that time was considered to be a temporary situation, as, for instance, was the case in Austria where the occupied sectors were reunited in 1955. --Cookatoo.ergo.ZooM (talk) 11:15, 16 March 2008 (UTC)[reply]
Cookatoo, you seem to be missing the point. What you have written, moreover, is only partially true. Yes, the division of Germany into three-subsequently four- occupation zones was considered to be a temporary arrangement, just like the Austrian divisions. However, the movement of the Polish border westwards to the Oder-Neisse Line was to be permanent, a decision the leading Allied powers had taken at the Yalta Conference, though there was to be much subsequent disagreement on the precise line of the border. Stalin made recognition of the Oder-Neisse Line a condition of allowing the Soviet Zone of Occupation, now the German Democratic Republic, to re-enter a fully unified Germany, a condition turned down by Konrad Adenauer, first Chancellor of the Federal Republic. For years after, all of the German political parties in the west were united in calling for a return of the territories lost to Poland and the Soviet Union, all of those areas within the 1937 borders; namely East Prussia, Pomerania, Silesia and parts of eastern Brandenburg. And so it continued, until Willy Brandt began his Ostpolitik. In 1970 he concluded the Treaty of Moscow and the Treaty of Warsaw, giving final recognition to the Oder-Neisse Line.
However, this is getting far away from the essential point at issue. The map in question shows the borders of the old Second Reich, the Germany of Kaiser Bill, borders that existed in 1914 prior to the First World War, not the Second. I cannot for a moment believe that the Social Democrats, at any stage in their political career, were calling for a restoration of the Kaiserreich, which, as I have said, would imply the deepest forms of revanchism. Clio the Muse (talk) 01:02, 17 March 2008 (UTC)[reply]
It's hard to see, but it looks to me like the yellow map is missing Alsace-Lorraine, so it is not exactly the old Kaiserreich, but a chimera composed of bits of the borders of the late Second Reich and bits of the borders of the Weimar Republic. If I'm right, the dating 1949–1965 must be correct. Curious.  --Lambiam 23:24, 18 March 2008 (UTC)[reply]
I'm not so sure, Lambian. West Prussia is certainly included, and that slightly rounded edge to the western border also suggests the inclusion of Alsace-Lorraine. http://en.wikipedia.org/wiki/Image:Deutsches_Reich1.png Clio the Muse (talk) 00:16, 19 March 2008 (UTC)[reply]
First time you are not completely right, Clio. In fact Lambian is right. The map does not include Alsace-Lorraine. It shows the current south-west german rhineborder, exsisting since Versailles. You will find a lot of german afterwar maps that include the lost eastern territories (after Versailles) with the text "under polish administration" implying that this is temporary. The new system for numbers and abreviations on license plates even were planned leaving room for additional ones for these territories. Germany didn´t relinquish it´s claims officialy until 1990. This was an important political question for the wast number of refugees (up to 14 Millions until 1950) of formerly german territories from the east. Those people were organized and did massive lobbying. Having even an own ministry [12]. So parties had to make concessions to that. Strange enough this never caused as much trouble as now (Erika Steinbach) that those people have lost all political power (due to natural causes) and are basicaly no political factor anymore.--Tresckow (talk) 21:42, 19 March 2008 (UTC)[reply]
Thanks, Tresckow; that was very useful. I don't want to belabour the point, but I have to say-and I know the scale does not help to make it absolutely clear-but that slightly rounded edge suggests to me the border of 1914, rather than the more angular post-Versailles border. But let that pass. I'm still puzzled, though. Are you saying that the SPD was, in fact, laying a nominal claim to those territories lost to Poland in 1918-19 as late as the 1950s? Surely the Treaty of Final Settlement and the subsequent German Polish Border Treaty only relates to the territories east of the Oder-Neisse Line, lost in 1945? Clio the Muse (talk) 00:47, 20 March 2008 (UTC)[reply]


March 16

Trial court procedures: documents and their filing?

in civil litigation when a case settles..what are the settlement documents that will be required and where are these filed? —Preceding unsigned comment added by 141.157.185.161 (talk) 00:28, 16 March 2008 (UTC)[reply]

Anonymous poster, are you talking about US federal law, the law of some US state, or some other law? Please note that anything looking like a request for legal advice is not answered here.
¡ɐɔıʇǝoNoetica!T09:33, 16 March 2008 (UTC)[reply]
No Legal Advice - Just A General Fact Outline: But, generally speaking, (in the USA) ... when a case "settles", this simply means that both sides / parties have come to an agreement by themselves (not through the court). And, part of that agreement / settlement is to withdraw the lawsuit that had been filed in the court. In short, the document is simply an agreement / contract agreed / signed by both parties. In some cases, (not all, and not most) a court may have to approve of the settlement. Otherwise, a settlement is tantamount to Party A saying to Party B, "I agree to drop this lawsuit if you agree to do such and such (e.g., pay me $10,000)". Party B says, "OK". Then, Party A withdraws the lawsuit that they had originally filed against Party B to begin with. And the case is now closed. From the judge's (court's) standpoint, he just knows that the Party A dropped their suit and it's off of his (the judge's) plate. He doesn't much care how or why. Parties can withdraw suits for lots of reasons. Settlement is just one. It's really not the judge's concern or interest, for the most part ... the judge / court simply knows that the case is gone. In fact, the term you usually hear is that "they settled out of court" ... meaning that the two parties were able to mutually agree and resolve the case without the court's help / intervention. Hope this was of some general help. (Joseph A. Spadaro (talk) 08:10, 17 March 2008 (UTC))[reply]
Sometimes the settlement forbids the parties to make its terms public. This would obviously be impossible if it were filed in the public record. —Tamfang (talk) 02:29, 19 March 2008 (UTC)[reply]
True. And good point. Sometimes, settlements between the parties are indeed confidential (if both sides agree to this stipulation). Which is fancy legal mumbo-jumbo that essentially allows big shots (large corporations, rich people, etc.) to get away with murder. (Joseph A. Spadaro (talk) 05:59, 19 March 2008 (UTC))[reply]
... over and over again. (Joseph A. Spadaro (talk) 17:01, 19 March 2008 (UTC))[reply]
In the United States, where the parties to a case are private individuals or corporations, generally all that must be filed is a form stating that the case has been resolved and the litigation is discontinued. Where the parties to a case are under a legal disability (minors or mentally disabled individuals) or where the case involves absent parties (class or derivative actions), the procedure is more complicated as court approval is required. In addition, if a party to the case is a public company and the outcome of the litigation may have a material effect on its financial condition, disclosure may be required in the company's SEC filings. Newyorkbrad (talk) 17:05, 19 March 2008 (UTC)[reply]

The Peloponnesian War

hi, i am a year 10 student in need of information about the Peloponnesian War, and i find wikipedia gives the best results however your artical about The Peloponnesian War says that it was a military conflict. but i cant see what the conflict was exactly about. i have read through it and just got confused. sorry about this, but i just cant find the answer

thankyou.

Courtney Stringer —Preceding unsigned comment added by 59.167.79.4 (talk) 00:42, 16 March 2008 (UTC)[reply]

The quote from Thucydides that begins section 1. Prelude of the Peloponnesian Wars is unusually clear, I would have thought. There may be experts here who disagree, of course. ៛ Bielle (talk) 01:09, 16 March 2008 (UTC)[reply]
Courtney, Bielle is absolutely right: the section headed Prelude could not make the causes of the war any clearer. In essence it was brought on by concerns in the rest of Greece over the growth of Athenian power. Have another look, and if it's still not clear come back with a fresh question. Clio the Muse (talk) 02:12, 16 March 2008 (UTC)[reply]
Dear Courtney: As Bielle and Clio suggest: Re-read the Wikipedia article, then ask another question, and you will doubtless get another answer. Edison (talk) 03:49, 16 March 2008 (UTC)[reply]

College football season: US and Canada

When is the next college football season coming for both U.S. and Canada? I am a big football fan. —Preceding unsigned comment added by Don Mustafa (talkcontribs) 02:33, 16 March 2008 (UTC)[reply]

Football like American football? Those seasons generally start in early fall. If you mean soccer, I don't know. Paragon12321 (talk) 03:31, 16 March 2008 (UTC)[reply]
ESPN's first 2008 CFB broadcast is Thursday, Aug. 28: NC State at South Carolina and Oregon State at Stanford. Most teams start their season two days later. -- Mwalcoff (talk) 04:14, 16 March 2008 (UTC)[reply]
The Canadian Interuniversity Sport football season stars on September 1, according to the official website. (With my old school playing the opening game!) Adam Bishop (talk) 05:23, 16 March 2008 (UTC)[reply]

The Canadian Football League season starts around June. If you are looking for the NFL then you've just missed it. Soccer in Canada starts when the snow melts, because it's not a rich enough sport to afford indoor stadiums yet. DJ Clayworth (talk) 14:39, 17 March 2008 (UTC)[reply]

From my textbook: Hitler, racism, and "Anglo-Saxon supremacy"?

"Hitler was driven by a virulent form of racism and Anglo-Saxon supremacy."

Should I be scratching my head?

Lotsofissues 07:50, 16 March 2008 (UTC)

Could we have the name of the textbook, so we know which to avoid? AllenHansen (talk) 09:17, 16 March 2008 (UTC)[reply]

What's the confusion? Okay, sure, we usually use the term Aryan, but Aryan, Nordic, Anglo-Saxon—they're imprecise terms (at best) for racial/linguistic groups that have a shared Germanic origins, and the Nazi racial theorists used them all at different points. I don't think it's too far from the mark, as far as textbook simplifications go. --98.217.8.46 (talk) 15:38, 16 March 2008 (UTC)[reply]
Anglo-Saxon is a historic/political term, not a racial one. There was never a Nordic Kingdom, or an Aryan Empire, but there were several states that were Anglo-Saxon, that possessed laws and beaurocracies and that have modern successors. Ninebucks (talk) 15:52, 16 March 2008 (UTC)[reply]
The central tenet of the sort of racial categorization that the Nazis (and others) used was that the historical and political (and hence the linguistic, even though they were well aware that anyone raised in a given culture will pick up the language) were linked to the racial, the biological. For this reason the terms are often used interchangeably and can't be easily extricated. --98.217.8.46 (talk) 17:53, 16 March 2008 (UTC)[reply]

Perhaps the compiler thought that Anglo-Saxon was simply a synonym for White? Likewise, I have sometimes heard Nelson Mandela refered to as an African American. Ninebucks (talk) 15:49, 16 March 2008 (UTC)[reply]

Hitler did not just believe in "white" supremacy. Check out the links I gave above. He considered many groups of "whites" to be inferior (Poles, Slavs, probably the Irish, etc.) That particular brand of European racial theory subdivides "white" up very finely, unlike racial theories in the modern USA which tend to see things as white v. colored (bi-racialism). "Anglo-Saxon supremacy" is closer to Hitler's view than is "white supremacy," though both are simplifications. --98.217.8.46 (talk) 17:49, 16 March 2008 (UTC)[reply]
But Anglo-Saxon is used to refer to England and people descending from the English. AllenHansen (talk) 18:51, 16 March 2008 (UTC)[reply]
The Anglo-Saxons were a Germanic people (see Anglo-Saxons). Hitler had no racial problem with the Anglo-Saxon British, except towards the way they operated their societies, if they had diluted their "blood", etc. Most of the internal (that is, meant for Germans to read) Nazi racial propaganda at the time argued that the "white" societies of France, UK, and USA had allowed themselves to become diluted from their original strong stock by allowing too much immigration, interbreeding, etc. --98.217.8.46 (talk) 19:26, 16 March 2008 (UTC)[reply]
But he wasn't driven by a virulent form of Anglo-Saxon supremacy!!!! AllenHansen (talk) 23:08, 16 March 2008 (UTC)[reply]
I'm not sure that this is right. A friend of mine recently went to the Saxon Museum in Berlin (I think), where they had a display board with lights to show the historical spread of the Saxons. The first lights are around (modern day) northern Germany, and then on the East coast of England (which is why the Romans built the Forts of the Saxon Shore). When I was at school, I was taught that 'the English' are supposed to be a melting pot of Angles, Jutes, Saxons, Celts, Norman Franks, Romans (whatever that means), etc., although modern genetic research [13] apparently contradicts this happy mythology (which, in its way, is as hopeless as the belief that Brutus fought Gog and Magog). As I understand it, Germans wouldn't see themselves as Anglo-Saxon; Saxon, perhaps, or Prussian, or Bavarian; the Angles came from what we would consider modern-day Denmark. The Angles (or, more correctly, the Anglo-Saxons, ie. used as a synonym for 'the English') were renowned (then as now) for their great physical attractiveness.
The Germans are (or were, until the 20h. century) traditional allies of the British: both nations united in a shared and understandable dislike of their common French neighbour!--Major Bonkers (talk) 10:13, 18 March 2008 (UTC)[reply]
Yes, Allen, you are absolutely right: it's a bizarre and absurd contention. I can just envisage the Führer chewing the carpets of hell in his frustration! Clio the Muse (talk) 21:02, 18 March 2008 (UTC)[reply]
Really tacky synthetic carpets, one hopes, in vile colors. —Tamfang (talk) 02:32, 19 March 2008 (UTC)[reply]
Reminds of something in Mad magazine, I think, where two sweet little old ladies didn't realise that Hitler was dictator over most of Europe, but thought he was the interior decorator. AllenHansen (talk) 12:01, 19 March 2008 (UTC)[reply]

Glorious Revolution

What was so glorious about the Glorious Revolution? Horace Morris (talk) 09:51, 16 March 2008 (UTC)[reply]

Glorious triumph of parliamentarianism and Protestant religion, trouncing of absolute monarchy.--Wetman (talk) 11:28, 16 March 2008 (UTC)[reply]
It was largely bloodless. In England, at any rate, the amount of bloodshed was astonishingly small. You also need to see it in terms of the seesawing Catholic-Protestant-Catholic-Protestant history of the recent centuries since Henry VIII's break with Rome, and the perception that stability at last was assured. --Dweller (talk) 12:01, 16 March 2008 (UTC)[reply]
It was glorious because no one died.--nessup (talk) 18:44, 16 March 2008 (UTC)[reply]
That is quite untrue. Some died in England, more in Scotland, and lots in Ireland.
Horace, it was glorious because the victors said it was glorious: it's really as simple as that! It was an expression first coined by John Hampden, a leading Whig, formerly implicated in the Rye House Plot. In essence, it became a way of justifying what was, in simple terms, an act of treason against the reigning monarch, subsequently enshrined in Whig folklore from Gilbert Burnet onwards. Treason doth never prosper; what's the reason? For if it prosper, none dare call it treason. Clio the Muse (talk) 01:27, 17 March 2008 (UTC)[reply]
You don't approve? I think you must be a Jacobite, Clio. Horace Morris (talk) 09:43, 17 March 2008 (UTC)[reply]
I neither approve nor disapprove, Horace; my opinions are always guided by the facts. Clio, a Jacobite? Of course not! Clio the Muse (talk) 21:00, 18 March 2008 (UTC)[reply]

Syrfia

Syrfia appears on sixteenth-century maps, on the right bank of the Danube at its mouth, between Bulgaria and Moldavia. There's nothing on the Internet under this spelling. What was Syrfia? Does it needs a Wikipedia article?--Wetman (talk) 11:28, 16 March 2008 (UTC)[reply]

Yep, there's Syrfia on Abraham Ortelius maps, between Varna and Danube delta. My guess is that it is a variation of spelling of Servia (Serbia), but it is obviously misplaced quite a bit :) . But this is just a guess. Hope this helps, anyway. --Dr Dima (talk) 16:55, 16 March 2008 (UTC)[reply]
I was thinking that too, or maybe a misplaced and misspelled Sofia? Adam Bishop (talk) 21:03, 16 March 2008 (UTC)[reply]
Okay, so I've added the following at Varna: "In the sixteenth<ref>A map of Ortelius, Romania, Bulgaria, Walachia et Syrfia (Antwerp 1602) shows the small territory of Syrfia south of the mouths of the Danube.</ref> and seventeenth century<ref>In a document of 1630 the Greek Orthodox metropolitan of Ternovo was in charge of numerous bishoprics, including Varna in Syrfia (A. d'Avril, "Bulgarie chrétienne", Revue de l'Orient chrétien 2 (1897:39).</ref> Varna was situated in a region called Syrfia."

Practice for Lawyers, Doctors, Etc. - Derivation of Term

I am curious as to why lawyers, doctors and others refer to their practice instead of work. I do not believe it is the same as practice makes perfect.75Janice (talk) 12:46, 16 March 2008 (UTC)75Janice75Janice (talk) 12:46, 16 March 2008 (UTC)[reply]

A practicing lawyer or doctor is one who is currently putting their training to practical use. The distinction is needed because people with professional qualifications are politely referred to as a lawyer, doctor, etc, even if they're retired or not currently working. FiggyBee (talk) 13:23, 16 March 2008 (UTC)[reply]
Architects, too, have a practice whereas painters and sculptors have patrons. --Wetman (talk) 05:29, 17 March 2008 (UTC)[reply]
Although one could also argue that, because law and medicine are always changing, they are continuing to get better at their trade and therefore practicing new ideas; however, that sort of breaks down with architecture, which I didn't realize was also considered a "ractice."4.68.248.130 (talk) 01:43, 20 March 2008 (UTC)[reply]

Fragrence ın Sultan Ahmed Mosque, Istanbul

Can anyone help me fınd out what the ıncense or fragrence used ın the Blue mosque ( Sultan Ahmed Mosque, Istanbul), as ıt ıs really famılıar but I couldn,t fınd anyone who could tell me what ıncense\fragrence ıs used. Thanks 85.105.17.138 (talk) 14:39, 16 March 2008 (UTC)[reply]

Frankincense? myrrh? Chanel No. 5? --Wetman (talk) 05:30, 17 March 2008 (UTC)[reply]
If you find it, please add it to the so beautiful article. Julia Rossi (talk) 22:13, 18 March 2008 (UTC)[reply]
I got a chuckle from the use of the Turkish 'ı' throughout. Thought at first it was a flaw on my monitor! —Tamfang (talk) 02:59, 19 March 2008 (UTC)[reply]

Atheist pope

Is it possible that the pope (present or any previous pope) does not believe in the existence of a God (which of course would be inadmissible), but does everything that he does, because he believes that it's in the overall interest of humanity, or is that just crazy-talk? ----Seans Potato Business 17:40, 16 March 2008 (UTC)[reply]

Is it possible? Of course. One's private beliefs might have nothing to do with one's public actions. I have known a least one atheist priest, who saw his job as being about community primarily. There are other reasons, mind you, that one might act as a Pope other than believing in the "overall interest of humanity"—I tend to regard the positions of the Catholic Church as often being more about enforcement of power than much else, but I am rather cynical about that particular organization. (My suspicions come not from any doctrinal aspect of it, but the way in which is centralizes power. In my experience any organization which centralizes power too rigidly in a hierarchy ends up becoming an organization whose primary purpose is to consolidate and maintain that power, whatever the original or ostensible "true" purpose is. I speak here about the organization, not the followers.)
Is it likely is another question—that would require a very in-depth knowledge of any given pope to make sense of. --98.217.8.46 (talk) 17:46, 16 March 2008 (UTC)[reply]
The Grand Inquisitor is an excellent "short story" discussing such a person. GeeJo (t)(c) • 18:23, 16 March 2008 (UTC)[reply]
Three popes met by chance in a pub and they got to discussing athy-ism. They all freely acknowledged that they were athy, but to different degrees, and they couldn't decide who was the athiest, who was merely athier, and who was just athy. Then in walked Sisters Mary Benedicta, Perpetua and Vaticana for their daily rum and coke. They espied the popes and came over to join them; fortunately they were sitting in the smoking section, so they could all pull out their Havanas with impunity. Sister Benedicta pulled out her personal brass cuspidor from her generous bag and placed it on the floor near her feet. One of the popes turned to the nuns, explained their quandary, and asked them to help them decide. Sister Perpetua, the youngest and most highly-strung of the three, said "Oh, Your Holiness, it's not for the likes of one such as I to comment". Sister Vaticana said "Well, since you ask, Your Holiness, I have a view on the matter, but it's best left unspoken". The popes then turned as one, resembling a very small group of tennis spectators, to Sister Benedicta, the eldest. She pulled herself up to her full height, had 2 puffs on her cigar, and downed the remaining three-quarters of her drink in one gulp. She appeared to be about to speak, but she removed her spectacles, breathed on them, slowly rubbed them with her cassock, and carefully replaced them on her head. She took another deep breath, then opened her mouth, and said "
(To Be Continued). -- JackofOz (talk) 21:04, 16 March 2008 (UTC)[reply]

Sean, you might very well be interested in San Manuel Bueno, Mártir by Miguel de Unamuno, which offers a fictitious treatment of the kind of situation you describe, with a possible outcome, though on a much humbler scale. I would hope that your atheist Pope would have the courage to act in the same manner as Don Manuel! Clio the Muse (talk) 01:56, 17 March 2008 (UTC)[reply]

But -- in short -- yes, that's just crazy talk. (Joseph A. Spadaro (talk) 08:19, 17 March 2008 (UTC))[reply]
It is unlikely because of the amount of dedication and sacrifice (cost of entry) to becoming a high ranking member of the church and eventually the Pope. In addition, as the Pope is, through all stages of the process, chosen by his peers, he would have to do a damn good job of convincing his fellows of his religious conviction.
I would say it might have been more likely had the Pope retatined more temporal power: cast in the present day, it is not hard to conceive that an atheist Medici might be pushed to the Papacy simply in order to cement his uncle's legacy and his family's influence in the papal territories.
I say "cast in the present day", because back in the 15th century everyone was Catholic, which reduces the probability of the Pope being atheist. --PalaceGuard008 (Talk) 09:31, 17 March 2008 (UTC)[reply]

Titanic

I'm sortof watching Raise the Titanic (OMG is a crap film!), and they think the Titanit in one piece and therefore "raisable". But in reality it split in two. How come they didn't realise that it split in two until they found the two bits? Kate Winslet was hanging onto one end as the bow went down then got in the water and hanging onto some flotsam until found by a lifeboat, - didn't she survive tell them, or if anyone didn't actually do that, then surely someone must have seen it go under? Bobble hobble dobble (talk) 18:48, 16 March 2008 (UTC)[reply]

zou do realise that raise the titanic predates the titanic movie by decades...? —Preceding unsigned comment added by 79.122.41.220 (talk) 19:05, 16 March 2008 (UTC)[reply]

And whatever you're watching, if Kate Winslet is in it, it sure ain't "Raise the Titanic". She was in "Titanic", a somewhat different movie. -- JackofOz (talk) 20:35, 16 March 2008 (UTC)[reply]

Now it makes perfect sense. The makers of "raise the titanic" couldn't know that the ship was in two pieces because they hadn't seen the 1997 Titanic film yet! -- (OR MAYBE NOT). Read the question. Now can I have the real answer pls, thx. —Preceding unsigned comment added by Bobble hobble dobble (talkcontribs) 20:36, 16 March 2008 (UTC)[reply]

If your reference to Kate Winslet is actually meant to be a reference to her appearance in "Titanic", a movie of which you make no mention in your question, then can I suggest the question is very poorly formulated and ambiguous. We're not mind readers. -- JackofOz (talk) 22:46, 16 March 2008 (UTC)[reply]

Why didn't the makers of raise the titanit know the ship broke its back and sunk in two bits? —Preceding unsigned comment added by Bobble hobble dobble (talkcontribs) 20:38, 16 March 2008 (UTC)[reply]

The straightforward answer is that, at the time of the publication of the novel (1975) and the production of the movie (1980), it was widely held that the wreck of the Titanic was relatively intact (or at least, that the hull was in one piece). Although some passengers reported that the ship had broken in two as it sank, the official inquest determined that the evidence provided by others (namely surviving officers and some first-class passengers) more believable, and consequently concluded that the ship had sunk in one piece. It was not until 1985, when Robert Ballard and others discovered the wreck (in two pieces) that this belief was upended. In short, the makers of the movie simply didn't know the condition of the wreck; any research that they may have done would have led them to the conclusion that the ship was probably in one piece. Carom (talk) 21:00, 16 March 2008 (UTC)[reply]

Yes, basically, everyone always thought that it went down as one piece --- not broken in two. No one knew the truth until the wreck was discovered many years later (over 70 years later) in 1985. As far as the survivors witnessing the event, there were probably many factors. There would be lots of confusion and panic and unclear thinking. Some observers were (in life boats) quite a distance away. It was pitch black in the middle of the night, if I recall. The sinking of a huge ship would cause a great splash, obscuring much of the view. Ultimately, a lot of conditions made it hard to see perfectly clearly that night, I'd imagine. Also, perhaps from an engineering perspective, the Titanic should not have cracked in two (just like it should not have sunk). Maybe from an engineering / physics point of view, the ship was deemed that it would have / should have / could have withstood this pressure and the material was strong enough to survive and not crack in these conditions. But, they were wrong. (Joseph A. Spadaro (talk) 08:31, 17 March 2008 (UTC))[reply]
Also, see this section of the Wikipedia article on the Titanic ---> RMS Titanic#Rediscovery ... it goes into detail about why people thought it sunk as one piece or broke in two. (Joseph A. Spadaro (talk) 08:43, 17 March 2008 (UTC))[reply]

Do religious israelis in israel think god gave them the land they're settling?

Is my understanding accurate that the Israelis think God gave them the land they have been settling for the past few decades? -- Thank you!79.122.42.52 (talk) 23:04, 16 March 2008 (UTC)[reply]

update: I have been told by a roommate that no, it's just because israel is a place they were historically. the story in the bible that I'm thinking of, she says, is in modern-day iran. Could I get a reference for this? Is there any movement of jewish people who want to go to the actual place god gave them (in modern-day iran? Is that true??)? where in the bible is the whole god giving them thing anyway? Refernces, please :)79.122.42.52 (talk) 23:04, 16 March 2008 (UTC)[reply]
If you'd look at that article link I posted in the answer which you removed — do not, if you please, remove the responses of others, even if they are informing you that your etiquette is off, and being told that your approach is "rude" is not, under Wikipedia definitions, an "attack" — you'd see that 1. your roommate is wrong and 2. the precise Old Testament citations regarding this issue. --98.217.8.46 (talk) 19:31, 16 March 2008 (UTC)[reply]

I understand your position but I don't appreciate being given the run-around. You linked a page which, as short as it is, is gibberish to me, and you carefully refrained from answering my question. So what's the answer, please, is my understanding that Israelis think God gave them the land they have been settling correct or incorrect?79.122.42.52 (talk) 23:04, 16 March 2008 (UTC)[reply]

Look, here's a map, to make it extra simple. The red and the blue borders correspond to dimensions specified in the Bible; the specific locations are those citations on the right in the legend (Numbers 32:1-2, Ezekiel 47:13-20). The answer is clearly "Yes, the Old Testament says that God gave the Jews some land, and much of that land corresponds to the boundaries of modern Israel." I hardly think linking you to a concise article with both references (which you requested) and images (to make it super-clear) is a "run-around." The first line of the article I linked to answered your question: "In Jewish belief, The Land of Israel is the region of land given to the Jewish people, according to the Hebrew Bible," and the illustrations clearly show that this is in the area of the modern day Israeli state, not in Iran. --98.217.8.46 (talk) 19:41, 16 March 2008 (UTC)[reply]
You're STILL carefully refraining from answering my question!!! It is my IMPRESSION that your answer "Yes, the Old Testament says that God gave the Jews some land, and much of that land corresponds to the boundaries of modern Israel" means "Yes, the religious Israelis in Israel think God gave them the land they have been settling", but if you carefully compare the previous two quotations in this sentence you will see that, in fact, they don't mean the same thing at all!!! So does that make the answer to my actual question "no" instead? I asked about what religious israelis think, and you answered about what the Old Testament says. Is it one and the same? Could I read your answer ("Yes, the old testament...") for my proposed answer ("Yes, the religious Israelis..."), or is there some important difference? If not, could you answer my question in the same terms, rather than different, confusingly similar, but perhaps in reality quite different, terms? Thank you!! And sorry to be trying :) —Preceding unsigned comment added by 79.122.41.220 (talk) 19:45, 16 March 2008 (UTC)[reply]

Yes. --Dweller (talk) 19:49, 16 March 2008 (UTC)[reply]

Thank you. Could you add it to our Land of Israel article?79.122.42.52 (talk) 23:06, 16 March 2008 (UTC)[reply]
Okay, I've added it myself, we'll see if it's reverted, and with what explanation... —Preceding unsigned comment added by 79.122.42.52 (talk) 20:10, 16 March 2008 (UTC)[reply]

No. --Dweller (talk) 19:49, 16 March 2008 (UTC)[reply]

This seems to contradict the above. Can you explain (BRIEFLY) why not??? Please be very very brief, I can already read the linked articles, but I don't see HOW they contradict my statement above. Thank you!!79.122.42.52 (talk) 23:04, 16 March 2008 (UTC)[reply]
Sorry if the above pair of posts seem contradictory, but to explain the apparent contradiction will take more than one word. Sadly, in this world, complex issues rarely can be boiled down to one or two words without losing more than could possibly be gained. --Dweller (talk) 19:49, 16 March 2008 (UTC)[reply]
Modern Israel is full of many people of many different religious beliefs. If you are asking, "Do the practicing Jews in Israel believe that God gave them the Promised Land, a territory which includes the borders of modern Israel and also some additional land on all sides?" the answer is "Yes." If you mean "Do all citizens of Israel believe this," the answer is clearly no. If you are asking, "Do most citizens of Israel believe this," the answer is probably yes, if you are asking "Does the current government of Israel believe this," the answer is, in that case, "I'm not sure, but probably." All around your problem here has not been with the answering, but with the asking. Going to a forum where people take their time to answer your questions and then berating them, before they've answered, about how they answer, is, in a word, rude. And it certainly won't get you useful results unless they have incredible patience. Had you just been a bit more deferent at the beginning, and simply asked for help clarifying the specific points you were having confusion with, we could have sped this along much quicker, couldn't we? --98.217.8.46 (talk) 20:07, 16 March 2008 (UTC)[reply]
Thank you for your answer! It confirms what I thought -- of course, I meant your first interpretation "Do the practicing Jews in Israel believe...", that's why I phrased it as "do religious israelis" in israel... As for the rest of your points, I think it's sufficient to say that I don't think you would have given these wonderful answers if i hadn't asked for this brief answer. Even as it is, until your post, the other ones had been handwaving and links to complete articles!79.122.42.52 (talk) 23:04, 16 March 2008 (UTC)[reply]

Is it fair characterize one of your answers as: "The Israelis that practice Judaism in Israel believe that God gave them the territory which includes the borders of modern Israel and then some" ?79.122.42.52 (talk) 23:04, 16 March 2008 (UTC)[reply]

Is this part of why practicing Jews in israel have been settling the land, or are the two things independent? —Preceding unsigned comment added by 79.122.42.52 (talk) 20:39, 16 March 2008 (UTC)[reply]

This is part of the reason why Jews have been trying to re-settle that area (and not, say, Alaska) for a very long time. There are more reasons than just that, though—historical, political, etc. To characterize all of that activity as being solely because of a few lines in the Old Testament would be an oversimplification. --98.217.8.46 (talk) 20:42, 16 March 2008 (UTC)[reply]
Was there a movement to settle a different area, for example Alaska as you mention, a part of Africa, etc, or from the beginning did it focus on modern-day Israel?79.122.42.52 (talk) 23:04, 16 March 2008 (UTC)[reply]
See British_Uganda_Program. -- Mwalcoff (talk) 22:31, 16 March 2008 (UTC)[reply]

you say it's part of the reason why that area was chosen. I'd like to read more analysis for why that area was chosen to settle, can you point me to a resource? Thank you.79.122.42.52 (talk) 23:04, 16 March 2008 (UTC)[reply]

The concept is discussed in Judaism 101, Level 1, here: [14] ៛ Bielle (talk) 21:39, 16 March 2008 (UTC)[reply]

okay, I'm reading it. Can other people here read it too and tell me if there are any glaring NPOV issues I shoudl know about since, it's a jewish site... —Preceding unsigned comment added by 79.122.42.52 (talk) 22:01, 16 March 2008 (UTC)[reply]

What did you jews do to this guy? he seems to have some 'issues'

87.102.124.155 (talk) 22:09, 16 March 2008 (UTC)[reply]

I don't know about "you jews" but I can explain what "you reference desk people" did. I asked for a simple summary of the history of israel/palestine, and you blasted me with links and detailed instructions of what to read while avoiding answering my question. In fact, the same thing is happening here with the notable exception of 98.217.8.46, who is the only person to actually answer my question (and did so with "If you are asking, "Do the practicing Jews in Israel believe that God gave them the Promised Land, a territory which includes the borders of modern Israel and also some additional land on all sides?" the answer is "Yes.", though a reference that states just that would be nice) 79.122.42.52 (talk) 23:04, 16 March 2008 (UTC)[reply]
Is someone not signing with four of these things ~? It's getting hard to follow first thing in the morning here. Thanks, Julia Rossi (talk) 22:13, 16 March 2008 (UTC)[reply]
I went back and signed. 79.122.42.52 (talk) 23:04, 16 March 2008 (UTC)[reply]


Dear Julia Rossi, The original question was whether RELIGIOUS Israelis in Israel (not all people, not the Israeli government, etc) believe that the land they have been settling for the past few decades has been given to them by God. What is your answer (about the religious israelis' beliefs). To me, it seems like a pretty clear "yes, they believe that" but as you can see the only person to give a clear "yes" (Dweller) as a response followed it with a clear "no" as a response! Can you weigh in as well? —Preceding unsigned comment added by 79.122.42.52 (talk) 23:10, 16 March 2008 (UTC)[reply]

Religious Jews think that God promised a land to the Jewish people, a promise the Bible has God making directly to Abraham, and reiterated to Isaac, Jacob and Moses. Religious Jews consider themselves the inheriters of that line. However, how that simple promise manifests itself is profoundly complicated and cannot be simplified by a "yes, no" question. Is it promised to them for them to receive now? Is 100% of modern day Israel "promised land"? Is Israel 100% of the Promised Land? These and a multiplicity of other questions are inherent in your question and the answers to every one will vary from individual to individual. Judaism is notorious for the climate it creates of a lack of uniformity of approach (2 Jews, 3 opinions is the joke). Chuck in the political overtones of the term "settling" in your question (a vast preponderance of religious Israelis are not "settlers", but live within the undisputed pre-1967 borders of Israel) and it becomes harder still to take your question with the simplicity you wish us to answer it. --Dweller (talk) 23:35, 16 March 2008 (UTC)[reply]

All right, now I seriously need some education (BRIEFLY) because I thought the whole country of israel didn't exist before 1945 or whenever, and now you're saying it had an undisputed old part? Since when? Thank you! (please please please don't link to dense text I'm barely literate). —Preceding unsigned comment added by 79.122.42.52 (talk) 23:46, 16 March 2008 (UTC)[reply]
<excuse inaccuracies caused by trying to simplify> After WWI, the Brits (who owned that bit of the world at the time) said the Jews should have a country in that bit of the world. Shortly afterwards, they chopped off 2/3 of it and called it Jordan. Then, after WWII, the United Nations voted to grant the Jews a hunk of land and the Palestinians a hunk of land. The motion was carried by the necessary margin. Consequently, when the British "mandate" expired in 1948, the Jews declared an independent state, called Israel. The surrounding Arab nations invaded both the Jewish bit and the Palestinian bit. They lost the fight against the Israelis, but took the Palestinians' land. Then, in 1967, Israel won an overwhelming victory known as the Six Day War. Israel conquered chunks of land - Gaza, the Sinai desert, the Golan heights, Jerusalem and the West Bank. Since then, the various territories have had different fates. Gaza is now under Palestinian control. Sinai was returned to Egypt as part of the 1979 peace deal. Golan was annexed by Israel. Jerusalem and the West Bank? Well, they're an awful mess and let's avoid the bluelink plague you say you don't want. But in essence the "pre 1967 borders" are uncontroversial in all but the most extreme circles (Hamas, for example, by documentation, deny Israel's right to exist at all... similarly, you'll find if you look hard enough, Jewish extremists who wistfully make comments about territory east of the Jordan being parts of biblical "Israel"). Hope that helps. It's a horrible tangly mess that some of the brightest minds of the last 50 years haven't come close to solving. And I strongly advise you that any explanations that take a paragraph or two are probably misleading in one way or another... the more you delve into this one, the more you'll understand. --Dweller (talk) 00:09, 17 March 2008 (UTC)[reply]
Wow, thank you. Much better than your "yes. no." above :) Thank you for not bluelining it to death either. I read your paragraph carefully, jeez, it's like a soap opera. But I don't see why you say one or two paragraph summaries are misleading in some way or another. Granted, they won't include all the nitty-gritty, but that's hardly misleading. —Preceding unsigned comment added by 79.122.42.52 (talk) 01:36, 17 March 2008 (UTC)[reply]
Dweller's answer above is an OK simplification except for two major errors. The first is the story that Jordan was chopped off from the country (the mandate) the Brits said the Jews should have. Many books says this, but it is just a very successful fabrication; Jordan was added to the smaller original mandate in the 20s, not chopped off from it. See the long note in Transjordan. The second is that it wasn't just the Arab states that took the land of the Palestinian bit in 1948. There was a 50-50 split of the Palestinian bit between Israel and the Arab states (Egypt and Jordan).John Z (talk) 04:03, 17 March 2008 (UTC)[reply]
And thank you guys because I didn't expect you to go to so much trouble. I know what tl:dr means with some of this stuff. I learned the diff between an Israeli and Jew by ignorantly assuming they were the same thing and the Israel person bluntly put me straight. It is well worth understanding if you can be patient. Cheers, Julia Rossi (talk) 02:18, 17 March 2008 (UTC)[reply]
and what is that difference please? Love you Julia. 79.122.42.52 (talk) 02:33, 17 March 2008 (UTC)[reply]
At the risk of showing even more of my ignorance, the Israeli is a citizen of Israel (which includes Arabs) and the like. This can include someone Jewish by birth yet not religious in the least (added: as in ethnic Jew).
Also living in Israel is the religious Jew who upholds the common values and sacred texts of Judaism, claims the lineage and keeps observance in the Judaic faith. This can be someone who ranges from a liberal "open" type that believes and keeps the calendar and goes to Temple, to more fundamentalist "closed" sects such as Hasidic Jews (pious ones) and all the little splinter groups within that. Getting clunky here, so I'll stop. Hope this helps Julia Rossi (talk) 03:49, 17 March 2008 (UTC)[reply]
This is not directed to the questioner who has the guts to admit to semi-literacy, but to anyone else who is interested in further reading: there is an OK-ish article at Proposals for a Jewish state which includes plans (some no more than wishful thinking, some fairly evolved proposals) for Jewish settlement in Alaska (Slattery Report), Australia (Kimberley Plan), Japanese Manchuria (Fugu Plan), Madagascar (Madagascar Plan). There was a settlement in C17 in Suriname (Jodensavanne). And wasn't there an early C20 one in southern South America? BrainyBabe (talk) 16:19, 17 March 2008 (UTC)[reply]


And on a separate note, "religious Israelis" are citizens of Israel who take their religion as a defining part of their identity: that could be Jewish, Muslim, Christian or any other religion. Jewish Israelis (as non-Jewish Israelis) may be secular, somewhat religious, or very religious -- I have heard them explain this tripartite division as "serves in the army, girls don't serve in the army, neither sex serves in the army". BrainyBabe (talk) 16:26, 17 March 2008 (UTC)[reply]
Secular, Hiloni, means that they aren't religious, observant or believing. By somewhat religious, you mean shomrei masoret, followers of the tradition. They are fairly religious, believing and observe most of Judaism, especially holidays, but not as strictly as the very religious, or Orthodox.

Hilonim are just as likely to avoid serving as Orthodox Jews, but for different reasons. I know many Shomrei Masoret girls who serve. AllenHansen (talk) 23:42, 17 March 2008 (UTC)[reply]

E-mail and the law

Moved to Computing

request for summary of suicide bombings by palestinians.

when did they start and why? why don't israelis, the other side in the arab-israeli conflict, also suicide bomb? (or do they, please correct me if I'm wrong).

i'm looking for a neutral explanation, ie one that doesn't assume one of the faiths is right. —Preceding unsigned comment added by 79.122.42.52 (talk) 20:47, 16 March 2008 (UTC)[reply]

Leaving aside the rights and wrongs of the Israel/Palestine conflict, there is a conflict, and each side inflicts strategic damage on the other in the hope of achieving their objectives. The Palestinians use suicide bombing as a tactic because they have few other weapons, and because their religion encourages them to see themselves as martyrs in a righteous cause. The Israelis don't because they have more and better weapons and the power of a state to deliver them. The Israelis can, if they wish, send helicopter gunships into Gaza, but the Palestinians have no helicopter gunships to send into Israel. Both sides are using the weapons available to them. --Nicknack009 (talk) 21:05, 16 March 2008 (UTC)[reply]
Ironically (or perhaps appropriately) it was Zionist extremists who started the bombing trend over there in the 1940s, first by bombing the British (who were occupying the area at the time), and then bombing the Palestinians (see Stern gang, Irgun, etc.). The radical Palestinians responded in kind after awhile of this medicine. But this was "just" car bombing—the use of vehicles to transport massive amounts of explosives to a target and detonating them—not suicide bombing. Suicide bombing didn't really take off in the Middle East until the late-1980s, when it was shown to be very effective in Beirut when used by Hezbollah in attacks against Israelis and Americans.
As for why the Israeli state doesn't use suicide bombings, a few reasons could be given: 1. they don't need to (suicide bombings are used because it is difficult for the Palestinians to otherwise penetrate "soft" targets with their weapons; the Israelis have much more advanced weaponry and can do so quite easily), 2. there is less of any sort of tradition of martyrdom in Judaism, at least to that degree, and you'd have a much harder time finding volunteers amongst the more affluent Israelis, and 3. suicide attacks, while they have a strong propaganda effect in terrorizing the enemy, also rob those who use them of moral legitimacy. While it could be debated whether the Israelis have a lot of moral legitimacy at the moment, their use of conventional arms makes them, at least in world affairs, taken seriously as a conventional player. The use of unconventional arms would make this more difficult, in the way that many countries will not deal with the current Palestinian government after the suicide-bombing group Hamas took over. It would be impossible for Israel to maintain its current relations with the USA if it started using suicide attacks, for example, as the USA has centered almost all of its criticism of the Palestinians on the tactics they have used. --98.217.8.46 (talk) 21:08, 16 March 2008 (UTC)[reply]
The Israelis and the Palestinians are two different cultures with two different senses of morality. Many Palestinians think it is a good thing to strap bombs to yourself and blow up a shopping mall. Palestinians actually celebrate when these things happen. You'll be hard-pressed to find Israelis dancing for joy should someone blow up a Palestinian school. Israelis don't [www.freerepublic.com/focus/f-news/1423506/posts dress their kids up like terrorists and suicide bombers] or use TV kids shows to indoctrinate children into terrorism. There are, of course, practical considerations, as 98.217 mentions. But I can't see the Israelis ever developing a suicide-bomber culture like that of the Palestinians under any considerations. There weren't any American kamikaze pilots, either. -- Mwalcoff (talk) 22:25, 16 March 2008 (UTC)[reply]
They celebrate not because they think blowing up a shopping mall is inherently good but because they think it is accomplishing a great goal, attacking a greater evil. The Israelis blow up things and sometimes people celebrate; when the United States atomic bombed two cities their citizens celebrated. It is not a case of having "different senses of morality" so much as it is "in the given context they saw this particular action as being necessary to win the particular war they were fighting." It's hard to find nations that have not, at various times in their history, targeted "soft" targets (malls, markets, etc.) because they felt that instrumentally that would help them at the time. I do agree, however, that the use of self-sacrificing tactics is relatively rare and requires a strong cultural component that would allow such a thing. But I think this is separate from the issue of targeting civilians in general, which is not at all unique. (By the the way, those children pictures are primarily just dressed up like warriors. Lots of countries dress their children up like warriors. I think it's gross, but it's hardly a sign of a culture of death. Palestinians hardly have a monopoly on violence in this world. It might be worth noting that I am not, in particular, pro-Palestinian, so much as I am generally unwilling to divide the world up into "good guys" and "bad guys" simply because one side is, at the moment, not targeting civilians as directly as they did before when they felt it was their best way to victory.) --98.217.8.46 (talk) 22:32, 16 March 2008 (UTC)[reply]
One other note: that Islamic fanatics used specifically suicide attacks is actually somewhat unexpected. They were not common before the 1980s. They were not used against the Soviets in Afghanistan, for example. It is very interesting that as a tactic it has made remarkable truck with Muslim extremists, but it's not like Muslims (of any nationality) have been doing that for hundreds of years, like it is some permanent part of their religion and culture. (Making war, sure. Like most people. But not suicide attacks.) It is also very interesting how something that has only been really used for the last 20 years or so (really making its first real show with the 1983 Beirut barracks bombing) is now taken to be a major part of their culture. --98.217.8.46 (talk) 23:45, 16 March 2008 (UTC)[reply]
There clearly are practical considerations to suicide terrorism (good for dedalus for mentioning Pape's book). However, when asking why one group does it and another doesn't, we shouldn't discount cultural considerations. To do so is to assume a deterministic view of history that doesn't square with reality. There are differences between cultures, just as there are differences between individuals. -- Mwalcoff (talk) 02:28, 17 March 2008 (UTC)[reply]
Right, but we shouldn't assume cultural statics. It is easy (as even S.ded does) to say things like "X is almost never" as if this were a very common thing throughout history, as if the cultures themselves had allowed for this from the beginning. There's a lot of historical contingency there, much of this is much more recent than it is made to look. The final assessment is going to be a mix of practical and cultural—if it becomes excessively practical, the cultural (if it has the slack) will bend around it, they will mutually reinforce. Again, the atomic bomb example: quizzed on whether it is a moral act to slaughter hundreds of thousands of non-combatants most Americans would probably say "no", but in the context of "what is moral to have dropped the atomic bomb in World War II?" most would probably say "yes". Without being too cynical about that, it is clear that the culture has come up with many ways to support this "practical" action, some of which are what I would call "legitimate", some of which are less so. --Captain Ref Desk (talk) 15:35, 17 March 2008 (UTC)[reply]
Suicide bombing is almost never a tactic employed by a state (the kamikazes being the exception). It is a last resort tactic used in asymmetric warfare. Suicide bombing is also used almost exclusively against democracies. A suicide resistance depends on the understanding that the target nation will use at least some restraint. A dictatorship might simply exterminate the ethnic or religus group carrying out the bombings. Read Dying to Win: The Strategic Logic of Suicide Terrorism by Robert Pape for a more detailed explanation. --S.dedalus (talk) 00:22, 17 March 2008 (UTC)[reply]

It may be useful to remember that Palestinians were not the first to use suicide attacks. The Tamil Tigers in Sri Lanka predated them. The Tigers are a secular Marxist group, drawn from a Hindu culture -- not Arab, not Muslim at all. BrainyBabe (talk) 15:03, 17 March 2008 (UTC)[reply]

I would like to ask someone on the Chinese Reference desk/Humanities (section) whether the curse/blessing May you live in interesting times is of Chinese origin. I do not know Chinese, so I can not do this. Could anyone here, who can write Chinese, do this and tell us/me if there was any reply? DanielDemaret (talk) 20:53, 16 March 2008 (UTC)[reply]

The article you linked to above makes it pretty clear that it's somewhat unlikely to really be of Chinese origin. You might try the Language Desk though if you'd like it rendered into modern Chinese. --98.217.8.46 (talk) 20:54, 16 March 2008 (UTC)[reply]
I don't have time to dig through my Buddhist history books, but this sounds very much like a poor representation of a common Buddhist "curse" from a story that was popular when Buddhism spread through China. The full curse was (from memory) "May you live in interesting times and die in peaceful times." I'm simply having a great deal of trouble placing the comment with the specific story. My mind keeps trying to place it in Journey to the West, but I know that is incorrect. What I remember is a young guard for a Buddhist master being told this phrase by the Buddhist master. It isn't really a curse or insult. It is really a "I hope you get what you want" type of comment since the young guard wants to fight and become famous in battle. Perhaps sleep will help me remember more. -- kainaw 02:52, 17 March 2008 (UTC)[reply]
Have never heard of it in Chinese, as with a number of other "an ancient Chinese proverb is..." expressions.
As for your original request... I wouldn't bother, given that the Chinese Wikipedia is blocked in mainland China and is these days frequented only by users from Hong Kong, Taiwan, and overseas communities. I don't hold out much hope of you receiving a credible answer, judging from the general standard of Chinese expression (very poor) in Chinese Wikipedia articles these days. --PalaceGuard008 (Talk) 03:48, 17 March 2008 (UTC)[reply]
Despite my cynicism, I've posted the question in Chinese at: zh:Wikipedia:詢問處#英语中的所谓中国谚语"May you live in interesting times". Let me know if you notice a response before anyone here does and need help reading it. --PalaceGuard008 (Talk) 04:02, 17 March 2008 (UTC)[reply]
That is very kind of you, PalaceGuard008. Thank you. :) I was sad to hear that the standard of chinese expression is poor in chinese wikipedia due to being blocked. Perhaps the question could be put with better luck at Baidu Baike? I am aware that the current article states that nobody has heard of the quote/curse, but sometimes one gets lucky. The suggestion by Kainaw looks like it thereal thing. DanielDemaret (talk) 11:55, 17 March 2008 (UTC)[reply]

Pictorial difference between hieroglyphs and cuneiform scripts?

What I understand is that they both represent objects through pictures, but that would mean that the languages would essentially be the same, when clearly a distinction has been drawn between the two. What would that distinction be?--nessup (talk) 20:55, 16 March 2008 (UTC)[reply]

Did you mean cuneiform from mesopotamia and Egyptian hieroglyphs, or more generally?
The languages they describe are different.
Cuneiform means roughly "wedge shaped" the script is indeed wedge shaped - of the type obtained by pressing a knife blade tip into clay, the heiroglyphs are more pictoral.
Were you asking whether the two scripts were used for both languages or what.?87.102.124.155 (talk) 21:09, 16 March 2008 (UTC)[reply]
I was asking what differences there were at all, because initially I was under the impression that they were both pictogram-ical languages.--nessup (talk) 21:22, 16 March 2008 (UTC)[reply]
The article says that cuneiform started out as 'pictographs' which has essentially the same meaning as 'heiroglyphs', unfortunately wikipedia doesn't seem to have any examples.
But here are some http://karenswhimsy.com/cuneiform.shtm http://www.britannica.com/eb/art-66036/Examples-illustrating-the-evolution-of-cuneiform-writing that show the evolution of the script.
Hang around for another expert - hopefully they'll be able to tell you if the original cuneiform 'pictographs' are the same as the heiroglyphs (or maybe you can do this yourself.)
"cuneiform" + developement or history or evolution etc seem to be good search terms, also try a picture search.87.102.124.155 (talk) 21:30, 16 March 2008 (UTC)[reply]
Neither of the two are true ideographic scripts. Hieroglyphs had an ideographic origin, but quickly became more logographic, where concepts are represented by pictures, but the pictures aren't necessarily representative of the idea. That is, you can't see a picture and know exactly what it means. The changes were based on things like the rebus principle, which in English would use a picture of an eye to represent 'I', because they sound the same. Then they started using elements of alphabetic script, which added more complexity to the writing, but also a lot more flexibility, because they could write things that are difficult to show with just a picture. The same thing happened, more or less with cuneiform, but because they spoke different languages, the ideographic to logographic progression went differently, and they ended up with different analogies between words. Steewi (talk) 01:07, 17 March 2008 (UTC)[reply]
This question might have been better asked at the language reference desk. The systems have a somewhat similar internal logic and development -- an early origin as pictographs, the use of some pictographs to write similar-sounding words whose meanings are hard to draw (the "rebus principle"), finally resulting in a mixed system of word-signs, sound-signs, and "determinatives". However, there are some historical differeences between the two systems, such as:
1) Egyptian hieroglyphs only wrote consonant sounds, and basically completely ignored vowels (with only a few very limited partial exceptions), while cuneiform sound signs generally indicated vowels.
2) The Egyptians preserved the original picture-drawing appearance of their writing system until the end (though they did also develop less cumbersome variants for less formal use), while once the Sumerians started to change their original picture drawings to simpler shapes easily formed with stylus-wedges, they quickly abandoned most iconic aspects of their script.
Both scripts were developed to represent their respective languages, and neither attempted to write pure ideas independent of language. There have been some such attempts in modern times, like Blissymbols, but neither hieroglyphic nor cuneiform are really specific precursors to Blissymbols. AnonMoos (talk) 08:56, 17 March 2008 (UTC)[reply]
AnonMoos (talk) 08:51, 17 March 2008 (UTC)[reply]

Jewmasters

I'm translating the article about Norwich to Norwegian, and then I came to the word Jewmasters here. The sentence is as follows: In 1144, the Jewmasters of Norwich were accused of ritual murder. Does it mean Rabbi, like the 'masters' of the Judaism, or was it just some Jews living in Norwich during the Middle Ages? Or can I translate it with 'some Jews'? Kasschei (talk) 22:18, 16 March 2008 (UTC)[reply]

This might be related to Blood libel against Jews. Edison (talk) 23:52, 16 March 2008 (UTC)[reply]
It is indeed an example of the blood libel, Edison, relating specifically to the murder of the 12-year-old William of Norwich in March, 1144. Kasschei, I have never come across the term 'Jewmaster'. I can only assume that it refers to prominent members of the community. Perhaps the most apt translation would be 'some leading Jews' Clio the Muse (talk) 00:19, 17 March 2008 (UTC)[reply]
It was actually vandalism from back in November (http://en.wikipedia.org/w/index.php?title=Norwich&diff=174179110&oldid=173308785). I've changed it back to just "Jews". Adam Bishop (talk) 07:29, 17 March 2008 (UTC)[reply]

Thank you for helping. Kasschei (talk) 09:58, 17 March 2008 (UTC)[reply]

Worst military defeat

I am looking for some good examples of the positively worst defeat in a nation's history in a single battle. What I mean is the kind of defeat that extinguished the country's liberty, or its very existence, for several generations or more. Thank you. —Preceding unsigned comment added by Turnvater (talkcontribs) 23:25, 16 March 2008 (UTC)[reply]

Battle of Hastings, 1066. Battle of France 1940, although it was more of a 6 week series of defeats and retreats than a battle. Edison (talk) 23:56, 16 March 2008 (UTC)[reply]
The Battle of Hattin in 1187 pretty much destroyed the Kingdom of Jerusalem as an effective state. The Battle of Baghdad (1258) destroyed the Abbasid caliphate. Adam Bishop (talk) 00:49, 17 March 2008 (UTC)[reply]

Well, there is the Battle of Alcácer Quibir,-the Battle of the Three Kings-in which Portugal suffered arguably the greatest defeat in its history. The body of King Sebastian was never found, allowing Sebastianism to enter the Portuguese imagination, with the once and future king occupying the same place as King Arthur or Frederick Barbarossa, returning through the mist in the hour of greatest need. In the short term his death was to lead directly to a succession crisis and the long union of Portugal with Spain.

There is also the Battle of Kosovo, so important in Serb national memory, which still resonates today. But my personal 'favourite' has to be the 1526 Battle of Mohacs, in which Suleiman the Magnificent defeated and killed Louis II of Hungary and Bohemia. It was such a disaster for Hungary that for all future generations the response to bad luck or misfortune of any kind was "Never mind; more was lost at Mohacs field." Clio the Muse (talk) 02:21, 17 March 2008 (UTC)[reply]

Check out the following articles:
Twas Now ( talkcontribse-mail ) 18:27, 17 March 2008 (UTC)[reply]
The Battle of Carthage destroyed the state and the civilization of Carthage forever. Marco polo (talk) 18:49, 17 March 2008 (UTC)[reply]

The Battle of Fallen Timbers, 1794. The Fall of Constantinople, 1453. The fall of Babylon, 539 BCE. The fall of Jerusalem in 586 BCE (which History of Jerusalem strangely states happened in 597 BCE???????) Edison (talk) 19:05, 17 March 2008 (UTC)[reply]

The Battle of Singapore was probably the British Empire's worst ever military defeat. Although the British ended the Second World War on the winning side, the Empire never really recovered from the fallout of what happened at Singapore. Xn4 19:49, 17 March 2008 (UTC)[reply]

did early europeans (colonial times) in America believe America was given to them by God?

So I understnd that the Puritans fled persecution by going to America, did they believe that this land was given to them by God? —Preceding unsigned comment added by 79.122.42.52 (talk) 23:49, 16 March 2008 (UTC)[reply]

Yes.--droptone (talk) 01:40, 17 March 2008 (UTC)[reply]
Note that that only refers to Puritans. There were a lot of non-Puritans among Europeans in Colonial times. I doubt there's one easy generalization you could make about it. --Captain Ref Desk (talk) 15:29, 17 March 2008 (UTC)[reply]
Not all the English in North America shared the Puritan sense of being specially favored by God, but when they compared how the land was actually being used by the Indians with a vision of the prosperous English-settled farms and cities that might arise there in the future, only a few of them (such as John Eliot) had true serious regrets... AnonMoos (talk) 07:27, 18 March 2008 (UTC)[reply]

March 17

6 month old news article

Hi there, I want to know if there any news relating to law such as laws suits, changes to legislation, controversial issues in the law and current events that happened 6 months ago like it happened in February, January, December, November, October or September of 2007? The news have to deal with either in Canada or outside Canada. —Preceding unsigned comment added by Don Mustafa (talkcontribs) 00:56, 17 March 2008 (UTC)[reply]

Ask your friendly librarian to familiarize you with Lexis-Nexis... AnonMoos (talk)

No, I don't want the lexis thing but I want is that if CBC have any archives from September 2007 through February 2008 dealing with the question of mine. —Preceding unsigned comment added by Don Mustafa (talkcontribs) 14:59, 17 March 2008 (UTC)[reply]

I'm having trouble making sense of what you want here, but Lexis-Nexis is a news and legal archive. It is pretty useful for finding out exactly the sort of thing you are asking about. --Captain Ref Desk (talk) 15:16, 17 March 2008 (UTC)[reply]
Do other countries have Lexis-Nexis equivalents? Julia Rossi (talk) 23:28, 17 March 2008 (UTC)[reply]

Precedents

Is there any precedents that to deal with the case that happened in 2007 where in Yorktown, somewhere in Canada, a principal of local high school called the police to do the search for drugs and illegal weapons and the boy was arrested but he said that he was innocent and this law suit had to do deal with Canadian Charters of Rights and Freedoms, Section 8. So any precedents relating to this case?

American artist - PEAKE, CHANNING (1910-1989)

I am searching for information about American artist Channing Peake.He created many murals, and was the West Coasts equivelant to Jackson Pollack. I understand he was born in Marshall, Colorado in 1910, then moved to the San Fernando Valley, California as a young boy. Later he owned a working cattle ranch/horse breeding operation in the Santa Ynez Valley, in Santa Barbara County, California. There is a county gallery space named for him and dedicated in his honour in Santa Barbara, California. I am curious to learn more about his years abroad spent with Picasso, Braque, Hemmingway, and his art dealer, Frank Perls. Oh, and also about his works created via the W.P.A. and where they are currently located, and the condition they may be in. May I THANK YOU! THANK YOU! THANK YOU! in advance for any help you are able to provide. Ejr26 (talk) 02:10, 17 March 2008 (UTC)[reply]

Here is an exhibition in San Francisco (More information here). When you have put all this together, will you create an article? Please? SaundersW (talk) 10:20, 17 March 2008 (UTC)[reply]

is this woman for real?

OPRAH: So what kinds of things? You don't have to give us the gory details, but what kinds of things went on in the family?

"RACHEL": Well, there would be rituals in which babies would be sacrificed, and you would have to, you know…

OPRAH: Whose babies?

"RACHEL": There were people who bred babies in our family. No one would know about it. A lot of people were overweight, so you couldn't tell if they were pregnant or not, or they would supposedly go away for awhile and then come back…

Why would anyone do that???? She's just making it up, right????

link —Preceding unsigned comment added by 79.122.42.52 (talk) 02:26, 17 March 2008 (UTC)[reply]

As to why someone would conceivably do that, see: Human sacrifice. --PalaceGuard008 (Talk) 04:07, 17 March 2008 (UTC)[reply]
Perhaps see also Satanic ritual abuse. I would wager on it being crap, especially if it was on Oprah. Adam Bishop (talk) 04:22, 17 March 2008 (UTC)[reply]
As I understand it, despite the hype, there has never been a single substantiated claim of Satanic ritual abuse. There's a good page on it at religioustolerance.org. --Nicknack009 (talk) 09:09, 17 March 2008 (UTC)[reply]
Jair-ri Jair-ri... Julia Rossi (talk) 23:20, 17 March 2008 (UTC)[reply]

Nazi questions

I have some questions on the Nazi party, state and system of justice. 1)To what extent were women involved in the Nazi party? 2)Did Jewish people offer any protest or resistance to the boycott of April 1933? 3)Concentration camps were initially set up as a way of dealing with political enemies. How did the Nazis justify retaining them as a permanent feature of their penal policy? Did they even attempt such a justification? 4)Was the Gestapo always intended as a permanent part of the police apparatus? 5)In what way did the role of the Criminal Police change after 1933? 6)What was the Nazi approach to dealing with ordinary crime, like robbery and so on, and how was this received by the German people. 7)Was there such a thing as a specifically Fascist theory of justice? Sorry so many. Tee Pot (talk) 09:38, 17 March 2008 (UTC)[reply]

Amazing. You have heaps of homework to do. Mr.K. (talk) 13:10, 17 March 2008 (UTC)[reply]
On your question (1), membership of the Nazi party was open to women, but there was a separate organization for them, the Frauenschaft, led by Gertrud Scholtz-Klink. And the League of German Girls, led for eight years by Jutta Rüdiger, was separate from the Hitler Youth for boys. One of Scholtz-Klink's tasks was to sell the message of male leadership. Our article on her quotes this from one of her speeches: "The mission of woman is to minister in the home and in her profession to the needs of life from the first to last moment of man's existence." See also Kinder, Küche, Kirche, for a little on Hitler's policy of squeezing women out of the employment market, until the Second World War led to a labour shortage. Despite discrimination, a few women, like Scholtz-Klink and Leni Riefenstahl made successful careers under the Nazis. Sadly, a disproportionately high number of the Nazi women who made names for themselves did so in the concentration camps. Xn4 18:56, 17 March 2008 (UTC)[reply]
In fact, although she made propaganda films, Leni Riefenstahl was never a member of the Nazi Party. Malcolm XIV (talk) 19:37, 17 March 2008 (UTC)[reply]
Sure, but she made no secret of her support for Hitler and the party. Of Mein Kampf she said in the 1930s "The book made a tremendous impression on me. I became a confirmed National Socialist after reading the first page. I felt a man who could write such a book would undoubtedly lead Germany. I felt very happy that such a man had come." Xn4 20:01, 17 March 2008 (UTC)[reply]
And in any case, she was enjoyed the favor of the party to make her films, which is the salient point here. She had close connections to top party brass, though in later years she would try to underplay that (for obvious reasons). --Captain Ref Desk (talk) 20:29, 17 March 2008 (UTC)[reply]
Some ten years ago I watched - by chance - an interview with Ms Riefenstahl on German TV. It was a chilling and disturbing experience as it was obvious that the woman had not lost her fascination with some aspects and personae of the Third Reich. Even more chilling was her blatant arrogance and ignorance, a sickening caricature of the spirit of the master race. --Cookatoo.ergo.ZooM (talk) 21:36, 17 March 2008 (UTC)[reply]
|Good for you. Don't forget that the reference desk is not a soapbox. And nor are you supposed to post diatribes. Flamarande (talk) 22:23, 17 March 2008 (UTC)[reply]
All fair enough; I did not defend Riefenstahl. But the OP was specifically asking about women in the Nazi Party. Malcolm XIV (talk) 23:57, 17 March 2008 (UTC)[reply]
Well, I, for one, am prepared to defend Leni Riefenstahl as one of the great film makers of the twentieth century, a position for which I make no apology. Clio the Muse (talk) 20:48, 18 March 2008 (UTC)[reply]
Hi Xn4, this intrigues me: "a disproportionately high number of the Nazi women who made names for themselves did so in the concentration camps" – any links or refs I can go to? Julia Rossi (talk) 23:16, 17 March 2008 (UTC)[reply]

Elisabeth Lupka, Margarete Rabe, Ruth Elfriede Hildner, Herta Ehlert, Elfriede Mohnecke, Jenny-Wanda Barkmann, Helga Hegel, Luise Brunner, Erna Petermann, Wilma Fath, Wanda Klaff, Lotte Johanna Radtke, Margarete Bisaecke, Ewa Paradies, Gerda Steinhoff, Erika Bergmann, Elisabeth Becker, Elisabeth Volkenrath, Emma Zimmer, Elsa Erich, Kaethe Becker, Jane Bernigau... Xn4 23:52, 17 March 2008 (UTC)[reply]

Thanks for that, now it's off I go... Julia Rossi (talk) 00:02, 18 March 2008 (UTC)[reply]
What about Ilse Koch, the Bitch of Buchenwald? Surely she's one of the most infamous. - Nunh-huh 01:20, 18 March 2008 (UTC)[reply]

Tee Pot, I’ve now had a chance to consider each of your questions and would answer as follows:

  • 1) Towards the end of the Weimar Republic women became involved in the NSDAP in ever greater numbers. By the end of 1932 the NS-Frauenschaft had a membership of 110,000, which leapt to 850,000 by the end of the following year, and to 1.5 million by the end of the year after that. I addition to that, the mass-orientated Deutches Fraunenwerk (German Women's Enterprise) was set up in September 1933, as an umbrella organisation to replace all those that had been banned by the Nazis for one reason or another. By 1935 it had a membership of 2.7 million, leaping to four million by the close of 1938, by which time it had become the largest non-compulsory organisation in the country. Tim Mason, a British Marxist historian, went so far as to argue that, in terms of the sheer numbers involved, the Nazi state enjoyed a higher degree of passive and active support among women than it did among men.
  • 2) There was very little Jewish people could do to resist the April boycott, but some offered protests in the most direct and meaningful way they could. One Edwin Landau, a decorated war veteran and shop owner, donned his medals and visited as many Jewish stores as he could, as well as confronting the SA men posted on his own doorstep. In Berlin, some Jewish shopkeepers posted notice of their distinguished war records on their front windows.
  • 3) After the war many Germans claimed they knew nothing of the concentration camps, a highly disingenuous position. In fact, from the very outset, they were part of the Nazi system of 'popular justice', whose existence was widely known and approved of by the community at large. It was Heinrich Himmler who offered a justification for need for 'protective custody' during the early days of the regime:

The state protects the life of all citizens. Unfortunately, it is only possible to provide such protection for certain individuals, and those involved have to be taken into protective custody under the direct protection of the police. The individuals involved, who are often of the Jewish faith, have through behaviour towards the national Germany, such as through offending nationalist feelings, and so on, made themselves so unloved among the people, that they would be exposed to the anger of the people unless the police stepped in.

Less than a week later he gave instructions to open a new camp at Dachau to 'protect' these social outcasts.

  • 4) To begin with the Gestapo was set up on the initiative of Herman Göring, in his capacity as Prussian Minister of the Interior, as part of the ongoing struggle against the Communists. By September 1933, by which time what was left of the Communist movement had been driven underground, brief consideration was given the disbandment of the new organisation, a measure even pressed for in sections of the press. However, by that time, it had become far too important part of the whole police apparatus.
  • 5) The Criminal Police-the Kripo-acquired a much more direct political role, with a strong degree of overlap with the Gestapo. They were released from all of the restraint under which they operated in the Weimar years, acting more and more as they saw fit.
  • 6) First and foremost, they responded to 'ordinary crime' by enforcing existing laws with ever greater vigour. It was all conducted in the kind of populist language designed to appeal to the wider German community. In 1933 Admiral von Levetzow, head of the Berlin police, called on his force to fight for "law and order, for decency, for discipline and for morality." It all went down very well by people tired of crime and disorder. Years later when one woman fondly recalled the years of the Third Reich she claimed that even thieves were shot, so that thereafter "nobody took anything that belonged to anyone else."
  • 7) Yes, there was, one that reversed the precepts of traditional legal systems. Equality before the law, the very heart of a liberal legal system, was effectively abolished. The Nazi system favoured speedy trials with minimal protection for the accused. In place of an old system of law that favoured, in their view, the 'security of the criminal', the chief thrust of the new law was aimed at 'securing the community of the people.' Citizens were told that the liberal principle of 'no crime without a law' had been replaced by 'no crime without a punishment.'

For all of these issues, and others along the same lines, I would strongly recommend that you read Backing Hitler by Robert Gellately. Clio the Muse (talk) 20:48, 18 March 2008 (UTC)[reply]

The Founding Fathers and religion

While driving home yesterday, I found myself listening to Speaking of Faith. They were talking to Steven Waldman and some of the stuff was interesting. What made me get all bubbly was when they talked about the full interview. Apparently parts had been edited out for the show which talked about how the Founding Fathers of the U.S. objected to Roman Catholicism. I'm paraphrasing here since I can't remember the exact wording they used. I'd like to download the whole interview and listen to it but won't have time for a few days to do so. So are there any articles which might reference what they might be referring to? I can read WP at work but listening to an MP3 of the interview can't be done until the weekend. Thanks, Dismas|(talk) 10:15, 17 March 2008 (UTC)[reply]

At the bottom of this page is a link to listen to the full interview in Quicktime. I don't know if it will still be there at the weekend, though. SaundersW (talk) 11:21, 17 March 2008 (UTC)[reply]
Anti-Catholicism in the United States--droptone (talk) 12:00, 17 March 2008 (UTC)[reply]
The Founding Fathers were largely Deists—not only Protestants, but Protestants who were pretty unorthodox to begin with, who had purposefully rejected much of the structure of traditional organized religion. That they would have found the Catholic Church suspicious at the very least is not surprising—the Catholic Church was a standard foe of Enlightenment thinkers, as it stood for pretty much the opposite of everything they did (at least, that's how they saw it). It was as much a political power as well as an intellectual and religious one; it was a centralized, hierarchical organization that traditionally aligned itself with monarchies and preached such anti-Enlightenment ideals as faith, dogma, miracles, etc. No big surprise that they weren't fans of it, to say the least. --Captain Ref Desk (talk) 15:14, 17 March 2008 (UTC)[reply]
Thanks, all! I'll check out the article in the next couple nights. The Quicktime link is also no use to me at work. Basically there isn't any easy way to get 'net audio, MP3 or Quicktime, in the cleanroom. Dismas|(talk) 01:47, 18 March 2008 (UTC)[reply]

I researched this last year. The records of the ratifying conventions in the individual states shed light on the Establishment Clause, religious oaths, and no religious test for public office clauses. Speakers address their fear of Roman Catholicism openly. Reading the debates, I concluded that in their world-view Roman Catholics were not truly fellow Christians. No condemnation is uttered.Catholics are considered alien along with Jews and Moslems. It was not expected in most states that Roman Catholics would run for public office. The religious clauses were not discussed and debated as much as we would imagine. 75Janice (talk) 03:47, 18 March 2008 (UTC)75Janice[reply]

what's the history of this place? what are the borders? what is the population? where are the census figures? when was it founded? who runs it? what is it? where can i find information about it?W-i-k-i-l-o-v-e-r-1-7 (talk) 10:48, 17 March 2008 (UTC)[reply]

Your best bet might be to contact the Contra Costa County Historical Society. Corvus cornixtalk 16:43, 17 March 2008 (UTC)[reply]
Or call the number on this page: http://www.oldmarshcreeksprings.net/ . To see where it is, click the globe icon on the wiki page. --169.230.94.28 (talk) 16:45, 18 March 2008 (UTC)[reply]

What if the south had won?

What if the south had won the American Civil War? I know this was probably impossible by military means. But what if, say, George McClellan had won the 1864 presidential election and concluded a peace because of general war weariness in the north? What is likely to have happened in the long term if the CSA and the USA became entirely separate republics? —Preceding unsigned comment added by Yes, I believe it (talkcontribs) 11:39, 17 March 2008 (UTC)[reply]

It says at the top of the page: Do not start debates. This question does not fall within the remit of a Reference Desk.
That said, you might like to brush up on what Saint Augustine said on the folly of speculating about what did not happen. 80.254.147.52 (talk) 13:55, 17 March 2008 (UTC)[reply]
This is a pretty common subject of alternate history fiction. The article mentions a few novels that use the South winning the war as a jumping off point. Recury (talk) 14:23, 17 March 2008 (UTC)[reply]
  • Augustine of Hippo´s De Civitate Dei is a compendium of mystical speculation.
  • Speculation is the essence of human intellectual development.
  • As to your question relating to an alternative history I hope it will be answered - in the context of some hypothetical reality - by qualified historians. --Cookatoo.ergo.ZooM (talk) 20:27, 17 March 2008 (UTC)[reply]
Your question could be partly answered by looking at what the South hoped to retain and/or gain by fighting in the first place. Origins of the American Civil War outlines the culture and economy of the South with its investment in secession, slavery and white racism, high agriculture and export economy with low or slow industrialisation, a share in federal taxes for development maybe, a repeal of the Tariff of Abominations say, sustained kinship and racist culture. But when you get to that point it looks like things would have changed from the inside pretty well inevitably by outside intellectuals, economic pressure, consciousness raising and one of the things they feared, slave rebellions. Julia Rossi (talk) 22:36, 17 March 2008 (UTC)[reply]
Have to laugh about St Augustine who would have had plenty to speculate about on the paths taken and not taken – shelving his mistress and child was one of them. Oouch. Clever to blame the pagans, turn celibate, get promoted and get published. Julia Rossi (talk) 23:10, 17 March 2008 (UTC)[reply]
Ask Harry Turtledove#The Southern Victory or Timeline-191 Series. Clarityfiend (talk) 23:11, 17 March 2008 (UTC)[reply]

The answer is simple: Harry Turtledove would have written novels about what would have happened if the North had won the war. --Anonymous, 01:51 UTC, March 18, 2008.

Winston Churchill wrote an essay called "If Lee Had Not Won the Battle of Gettysburg". See Alternate history#Early 20th century and the era of the pulps. Corvus cornixtalk 17:31, 18 March 2008 (UTC)[reply]

An interesting question, which really does not have to be answered by a complete 'alternative reality', the usual problem with counter-factual history. The simplest thing is to project forward already existing features of the economic and political system of the North and South.

The North was in the early stages of an industrial revolution, most likely carried forward with ever greater momentum, freed from the problems and economic costs of Reconstruction. The South would have maintained its peculiar institution, which would have acted as an even greater drag on an economy ruined by war. Huge amounts of credit would have been required to restore a ruined infrastructure and build a new shipping fleet to carry cotton to Europe. Could this have been achieved in the short term? Possibly, but only with substantial foreign help for a country with virtually no capital.

For the black people of the South the North, now free in absolute, not just relative terms, would have been a greater attraction than it was before, with no more Fugitive Slave Act or the possibility of a new Dred Scot decision. Presumably the states of the upper South would have to have maintained a strong militia presence to try to prevent the exodus. Would the states of the lower South have been prepared to meet a share of the financial burden involved? Well, we know that even during the war there had been strong centrifugal forces in the Confederacy, with people like Joseph E. Brown of Georgia taking an extreme view of States Rights. These pressures are likely to have increased with peace.

We also have to assume that all of the communities of the South would have settled down to independence based on secession, when over large parts of the Confederacy many were fighting against the whole process, and just as many in the Union Army. It is possible, then, that the new CSA would have faced its own prolonged internal struggle. At the very least, it is likely to have faced a guerrilla war, with Southern dissidents supported by Northern abolitionists, unhappy with the outcome of the war. This would have weakened its tired economy still further.

Touching on abolitionism, the end of the war would have left many in the north unsatisfied, people who are likely to have made the most of border clashes that are likely to have followed if the black exodus had achieved high, possibly unsupportable levels. Once the North had gathered its breath, and greatly strengthened its industrial base, a new war might have followed.

Would the South have carried slavery into the twentieth century? It seems inconceivable, and the institution is likely to have been abolished before the end of the nineteenth, possibly by 1889, the same time it went in Brazil. But how was the South, backward and weak, to afford the huge levels of compensation that would have been necessary without a strong federal reserve? Possibly by introducing the kind of redemption payments which allowed the Tsars to 'free' the serfs. In other words, by mortgaging the future and placing an intolerable financial burden on the poorest sections of the community. Hence more migration; hence an even weaker economy

Could the Confederacy have made it to the twentieth century? No, probably not; or, if it did, as a weak and economically dependant nation. Clio the Muse (talk) 21:56, 18 March 2008 (UTC)[reply]

C.S.A.: The Confederate States of America --Nricardo (talk) 01:55, 19 March 2008 (UTC)[reply]
Oh, how I abhor alternative history fiction. Better to understand the events that created our present situation than wondering what else they could have begotten. bibliomaniac15 Midway upon life's journey... 02:01, 19 March 2008 (UTC)[reply]
I love intellectual sports and challenges of all kinds; and this one is as good as any other. Besides, there is a world of difference between reasoned speculation-which forms part of most historical writing-and 'alternate history fictions.' Clio the Muse (talk) 02:14, 19 March 2008 (UTC)[reply]

Edward I and the Edict of Expulsion

In your page on Edward I of England there is a section on the 1290 expulsion of the Jews. Some possible reasons are given for King Edward's decision, though nothing very specific. I would be grateful for any info on the reasons behind the precise timing of this measure. Thanks. Dora Kaplan (talk) 13:07, 17 March 2008 (UTC)[reply]

In truth, no-one really knows, although plenty speculate. Our article dismisses the usury angle because there was little for the crown to gain, but I think there's a good chance it may be relevant, particularly from the angle of baronial pressure for expulsion of those who were owed substantial moneys. Edward needed to keep his barons on-side and he'd rather they paid him any available cash to finance his ambitious wars, than any Jewish money-lenders. But, I'm speculating. The timing is odd, as it doesn't really coincide with any time of political weakness, but perhaps it's in an unusual lull in foreign martial exploits, so Edward was able to turn his attention to domestic affairs for once? --Dweller (talk) 15:50, 17 March 2008 (UTC)[reply]
Actually, we do know quite a lot about the precise timing of the Edict of Expulsion: the Wikipedia article here has clearly not yet caught up with the latest scholarship on the subject. Certainly Edward's decision was based on a long tradition of anti-Semitism, and the precedent of more limited expulsion, including one initiated by Simon de Montfort, who had expelled the Jews from Leicester early in his career. But in 1290 Edward needed money, which could only be obtained by a new Parliamentary grant of taxation. When Parliament was assembled, the knights of the shires demanded the expulsion of the Jews as a condition of such a grant. And so it followed. The measure was so popular that Edward received the biggest tax grant of the Middle Ages. Clio the Muse (talk) 21:16, 18 March 2008 (UTC)[reply]

US and PLA Marines

on the page QBZ-95, there are two pictures showing chinese and american Marines conducting joint exercises/meeting up. I was not aware that the american and chinese militaries had a relationship. Where can I find out more about this? --AtTheAbyss (talk) 13:12, 17 March 2008 (UTC)[reply]

Try this search http://www.google.co.uk/search?hl=en&q=us+sino+military+coop&meta= first two links look directly related to what you ask.87.102.13.144 (talk) 15:01, 17 March 2008 (UTC)[reply]
I was hoping for something with less rhetoric, but they were useful nonetheless. Thanks. --AtTheAbyss (talk) 12:15, 18 March 2008 (UTC)[reply]

Bankruptcy of mutual fund company: what happens to the fund?

This question is about U.S. law. Suppose you invest in a mutual fund that is administered by some large mutual fund company. Then the company goes bankrupt. Will you get your investments back, or are they treated as assets of the company and distributed among all the company's creditors in the normal course of the bankruptcy proceedings?

A different way to phrase the question: is the money you give to a mutual fund still your money (and therefore immediately falls back to you if the company folds) or is it considered the company's money (and therefore you are just one in a long line of creditors in case of bankruptcy)?

And a third way to get at the same thing: is the mutual fund legally separate from the mutual fund company? In other words, is it possible for the company to go bankrupt (e.g. because of an accounting scandal, or because they speculated unwisely with some of their money [not with the mutual fund money], or because they built too expensive a headquarters building etc.) even though the fund itself is still perfectly healthy (since the underlying securities haven't lost any value)?

[I already know that mutual fund companies are required to carry insurance for the case that an employee runs away with the mutual fund money, but that's not the case I'm interested in. I'm also not interested in the case of the mutual fund shares losing value because the underlying securities lose value.]

Thanks, AxelBoldt (talk) 19:11, 17 March 2008 (UTC)[reply]

The assets of the mutual fund are owned by a trust company, which administers the fund on behalf of its investors. The trust company appoints a fund manager to manage those assets and a custodian to hold the assets. The fund's assets are quite separate from the assets of the fund manager, and will not be lost if the fund manager goes into liquidation. However, there is, in general, no guarantee that investors will get back as much as they invested in the fund, because the value of the fund's assets may fall due to poor fund management or general market conditions. In an extreme case it is possible that the fund itself may become insolvent, and would have to be wound up. Gandalf61 (talk) 14:27, 18 March 2008 (UTC)[reply]
Thanks! In the case of a mutual fund of Vanguard or Fidelity, say, what role do Fidelity and Vanguard play? Are they the trust company, the fund manager or the custodian? And who plays the other roles? AxelBoldt (talk) 17:42, 18 March 2008 (UTC)[reply]

Greatest Knight

I've seen it in the article on Gawain refer to him as the Greatest Knight, saying that he was one of few. Who are the others? Thank you in advance. 99.226.26.154 (talk) 20:59, 17 March 2008 (UTC)[reply]

The Black Knight obviously БοņёŠɓɤĭĠ₳₯є 21:35, 17 March 2008 (UTC)[reply]
Less obviously, there is the Mourning after the Knight Before. --Cookatoo.ergo.ZooM (talk) 21:49, 17 March 2008 (UTC)[reply]
Sir Alec Guinness, a knight twice over. ObiterDicta ( pleadingserrataappeals ) 22:11, 17 March 2008 (UTC)[reply]
Sir Ninian Stephen is a knight 5 times over. Other mere mortals may have more than 5. Of royalty, the Duke of Windsor had 10 knighthoods (the article lists 8 of them, but I believe he was also a GCMG and a GBE). -- JackofOz (talk) 02:15, 18 March 2008 (UTC)[reply]
Welcome OP, the other knights are Galahad, Gawain, Lancelot and Percival. Bit like the best 007 contenders with a bit of Jesus thrown in. Julia Rossi (talk) 23:01, 17 March 2008 (UTC)[reply]
From Sir Ector de Maris's lament for his brother Lancelot in Malory's Morte d'Arthur:
Xn4 23:32, 17 March 2008 (UTC)[reply]
Basically what happened was this: Gawain was the original "greatest knight" of the Round Table (besides Arthur). Over the centuries, the story developed and characters were added to it. Many characters were established as "great knights" because they defeated Gawain in combat. He was like a right of passage. Lancelot, Lamorak, Galahad, and Percy beat him. It was like whenever the new guy showed up and bragged he was good he had to fight Gawain to prove it. Wrad (talk) 02:22, 18 March 2008 (UTC)[reply]
See also the Nine Worthies. Or people like Enguerrand VII of Coucy, or Jean Boucicaut. Adam Bishop (talk) 03:31, 18 March 2008 (UTC)[reply]
Or even Pierre Terrail, seigneur de Bayard, "the knight without fear and without reproach". (But this is all as admittedly subjective as arguing over who the "greatest general" was.) Kirill 04:34, 18 March 2008 (UTC)[reply]

99.226, I assume, from the form of your question, that it is only fictitious knights that you are interested in, though others have offered you some real examples. On the basis of that assumption I will hold off from mentioning the greatest of all the Medieval knights! Clio the Muse (talk) 23:02, 18 March 2008 (UTC)[reply]

Who "was blessed with a brain too small to impede the natural vigour of a big, powerful and tireless physique"?[15]eric 23:50, 18 March 2008 (UTC)[reply]

Gold ownership

Is there any information about the amount of gold available in each country? I'm talking about actual gold owned by people or the government, not ore. Regards, deeptrivia (talk) 21:44, 17 March 2008 (UTC)[reply]

Official gold reserves gives a list of bank holdings. I doubt if numbers on private ownerships are available, as it would negate the purpose. --Cookatoo.ergo.ZooM (talk) 22:12, 17 March 2008 (UTC)[reply]
What purpose? deeptrivia (talk) 00:16, 20 March 2008 (UTC)[reply]

Any National banks selling gold bullions?

Is there any new information if national banks are taking advantage of the current high price of gold to sell gold reserves? I mean with Nixon the link between gold and currency was severed (in the major world economies at least). It would make a lot sense to sell gold reserves right now as the price is reaching record levels. Flamarande (talk) 22:30, 17 March 2008 (UTC)[reply]
  • In what currency would they sell it ?
  • Who would buy it ?
--Cookatoo.ergo.ZooM (talk) 22:48, 17 March 2008 (UTC)[reply]
Any, except the US Dollar. I'm speaking of national banks who have little interest in keeping gold at all.
If the price is getting higher it is a signal that the demand for gold is increasing a lot in the first place (I hope you know of the the theory of "Supply and demand" and that prices usually rise when the demand increases). I suppose that plenty of ppl are buying and investing in gold because of the fear of the economic recession. Flamarande (talk) 22:59, 17 March 2008 (UTC)[reply]
Yours is a very good question indeed, Flamarande. Bear in mind the following: national banks, though exhibit budgets and administer financial goods, have other goals than making profits out of circumstancial situations. What is more, gold reserves are meant to be stocked in order to back the financial system (e.g. to somehow act as a means of sustain of money, though not completely as in the times of the gold standard). In such a particular state of affairs like the current one, it may be not a good idea to sell an important amount of gold reserves. Pallida  Mors 03:36, 18 March 2008 (UTC)[reply]

national banks would be crazy not to hoard gold, because the us economy is the engine for the world's -- if the us had a depression, the whole world would feel it. Possibly EVERY currency would plummet in value, in the worst case. On the other hand, if banks have a big suprplus of gold then maybe now is a good time to sell off a little of it, if they need liquidity for whatever reason. IANAE —Preceding unsigned comment added by 79.122.9.122 (talk) 22:03, 18 March 2008 (UTC)[reply]

The thing is, gold isn't stable either. It's just as volatile as any other currency, because it's based on demand and supply. Hoarding gold artificially inflates the price, but the minute you try to use it as a primary currency, the gold enters the global economy and devalues as it's traded more and more. -- Kesh (talk) 01:10, 20 March 2008 (UTC)[reply]

March 18

Tahitian surfer with same last name

Is Tahitian surfer Manoa Drollet in any way related to the unfortunate Dag Drollet?72.229.136.18 (talk) 01:52, 18 March 2008 (UTC)[reply]

Ah the Dag Droll-ay connection... Manoa doesn't mention any names here[16] but can be contacted via Tahiti Surf Club. Julia Rossi (talk) 02:29, 18 March 2008 (UTC)[reply]

Laundering money

People who want to launder money or just want to avoid paying income or sales tax do something quite interesting I never thought of. Lets say you owe a person $950,000 for drug/prostitution deals to keep your clients happy and they expect payment. Instead of paying them in cash you build a house or office or buy a diamond broach or acquire land or whatever appraised at $950,000. If you "sell" the property to them no one can require you to show that the buyer actually paid. Transfer of title can be made without a corresponding deposit in your bank account that can be tracked and the property becomes a payment made only to appear as a sale. Does everyone except bureaucrats think that no one is going to transfer title without being paid know that this is how to work the system or how the system works? —Preceding unsigned comment added by 71.100.15.115 (talk) 02:25, 18 March 2008 (UTC)[reply]

It would be trivial to see if the person receiving the title paid anything close to market value for the item by checking their bank accounts, which may be why you do not hear about this sort of transaction that much. A much more common method is to buy something at market value and sell to the person you want to benefit at well-below market rates.--droptone (talk) 11:44, 18 March 2008 (UTC)[reply]
Unfortunately the law does not require that the trail leading up to any particular transaction be verified. In fact doing so under current law is considered an act of privacy invasion even possibly under a court order. The reason you do not hear about this all the time is because it is done all the time and is very easy to do and to get away with doing. Designating a transaction as a gift is by far the more common means in use to cover a discrepancy in price. —Preceding unsigned comment added by 71.100.15.115 (talk) 20:24, 18 March 2008 (UTC)[reply]
I have examined many deeds for houses or land in the U.S. which described the property and said it was sold for "$1 and other considerations" without anywhere stating the total price. The "other consideration" could have been an unrecorded lease on some other property, or it could have been personal property, or it could have been art or bearer bonds. There is more to "consideration" than money in bank accounts. Edison (talk) 14:04, 18 March 2008 (UTC)[reply]

An explanation of the Jews

Recently I overheard someone use the movie 2001: A Space Odyssey to explain what happened to the Jews. The idea goes like this: The Jews having escaped Egypt and taking with them their belief of being God’s chosen people and incapable of sin or imperfection (some Jews at least) took on the personal attitude of self-perfection about themselves identical to the computer in the movie named HAL. Although believing themselves to be perfect or to have reached a state of perfection they failed to see anything they did wrong not as being the result of their own error but rather as the result of an error made by someone else. The problem being that other reasonable or logical beings faced with the absolute facts reached the opposite conclusion. Eventually this awareness of self deception was conveyed to a Jew (Jesus Christ) who was one of their own who realized the need to convey this knowledge to his fellow Jews with the idea that the only way he might convince them of his sincerity was to sacrifice his life.. The comparison deviates here somewhat in that the self sacrifice of Jesus’ life did not succeed and the Jews continued to see themselves as incapable of wrong doing or sin (error). The Holocaust then was the attempt to shut the Jews down long enough for them to go through a reset and perhaps then come to realize that they are in fact capable of error and need to question themselves based on the idea that they are not incapable of error and are not perfect despite the conflicting expectation of anyone told they are on a special mission as a child of God. Having heard this I am now curious as to whether the Jews actually believe themselves to be perfect or more perfect than or superior to others in the sense that they are in fact acting as the children of God under God’s direction? —Preceding unsigned comment added by 71.100.15.115 (talk) 05:43, 18 March 2008 (UTC)[reply]

I can tell you that your friend is completely ignorant of the Bible and of Jewish history. If you read the Hebrew Bible (Old Testament), you'll find it's full of sins that the Jews committed, from the Golden Calf to the idolotry of Solomon's wives to the rejection of the prophets like Jeremiah. Clearly, by adopting the books of the Hebrew Bible as their holy text, the Jews were acknowledging their own failings. The Bible also makes it clear that many of the bad things that happened to the Jews, such as the scattering of the Ten Tribes and the Babylonian Captivity, were due to their own sins rather than to "an error made by someone else." A Talmud story says that the Messiah will come only when (the people of) Israel repent for their sins and heed the voice of God. So undoubtedly, Jews are and have always been very attuned to their own sins and think themselves far from perfect. -- Mwalcoff (talk) 08:14, 18 March 2008 (UTC)[reply]
...and is part of the acknowledgment of their own imperfection the believe that God must absolutely conform to their reading of the Scriptures and in the event of God's assessment that His covenant with them had been broken could not become flesh and sacrifice that flesh as the final effort to save them from the consequences of their own undoing? —Preceding unsigned comment added by 71.100.15.115 (talk) 18:24, 18 March 2008 (UTC)[reply]
Your last question appears not to be a question but rather a diatribe or an attempt to start a debate on Jesus. Both diatribes and debates are improper for the reference desk, as the instructions above specify. There are many religion forums on the Internet where you can post such material. -- Mwalcoff (talk) 23:18, 18 March 2008 (UTC)[reply]
Quite the contrary. However, if the Wikipedia reference desk is unable to provide unbiased clarification for the purpose of resolution in support of universal comprehension, enlightenment and understanding then entering into a debate here would be a bit like arguing with HAL. —Preceding unsigned comment added by 71.100.15.115 (talk) 02:35, 19 March 2008 (UTC)[reply]

What are you doing ,Dave?hotclaws 14:23, 19 March 2008 (UTC)[reply]

Easily half of Judaism is about how much sin and imperfection Jews have and how angry God is on account of it. Honestly your friend doesn't know a thing about Judaism. Throwing in misc. junk about Jesus (which shows a total lack of knowledge of the New Testament) plus a dubious literary interpretation of the Holocaust (which seems to make it sound almost necessary or beneficial!!) really tops the cake. Your friend, and probably you too, if you entertained this nonsense, have a great depth of ignorance on these subjects. If you'd like to be pointed to some resources that would give a better understanding, I'm sure we can do that on here, but somehow I suspect you're already self-satisfied with your goofy answer because it plays to some other desires you have. --Captain Ref Desk (talk) 16:51, 19 March 2008 (UTC)[reply]
LOL! Clio the Muse (talk) 02:32, 20 March 2008 (UTC)[reply]

Technical colleges

What are the advantages of attending a technical college? Also what are the career opportunities you may find? —Preceding unsigned comment added by 71.137.236.111 (talk) 06:44, 18 March 2008 (UTC)[reply]

Practical skills? Mo practical less theory. Shorter time scale for graduation. Mo employability through recognised qualifications, trade and other certificates etc, less ...? Talking to a career counsellor at any tech college would help. Julia Rossi (talk) 07:54, 18 March 2008 (UTC)[reply]
Mo? Odd argot. --Tagishsimon (talk) 11:50, 18 March 2008 (UTC)[reply]
Like "ho'" I suppose. ៛ Bielle (talk) 17:06, 18 March 2008 (UTC)[reply]
Do I hear tut-tutting? No Bielle, not like ho. I was rushing at the time. Feel free to answer the OP's question anytime, cheers Julia Rossi (talk) 22:33, 18 March 2008 (UTC)[reply]
"Tut-tutting"? Not from me. Don't have a licence to drive one, for a start. I was just following Tagishsimon's comment with an example of a similar construction. It is similar, and thus not as odd an "argot" as he/she suggested, isn't it? I feel as if I am missing a point here. You have done a fine job in answering the question. Ms Ross; I have nothing more I could possibly add. ៛ Bielle (talk) 23:11, 18 March 2008 (UTC)[reply]
mo is the adjective whose comparative is more, of course. —Tamfang (talk) 03:41, 19 March 2008 (UTC)[reply]

global warming

moved to science desk FiggyBee (talk) 07:35, 18 March 2008 (UTC) what is the meaning of figgybee meanwhile —Preceding unsigned comment added by 59.92.126.151 (talk) 13:54, 18 March 2008 (UTC)[reply]

FiggyBee is a banana bender from Down Under. On her/his user page s/he writes: "My username is a reference to acalolepta vastator, the Fig Longicorn Beetle." --Cookatoo.ergo.ZooM (talk) 01:36, 19 March 2008 (UTC)[reply]

Need help finding melody

Hi, I am looking for a melody that is stuck in my head. I have already gone to Musipedia and used the various searches (keyboard, contour, rhythm, etc.) but have no luck. I am not good at music. I experimented with different things on the keyboard and after a while I put together something that sounds vaguely like it, although it is probably off key and wrong in many places (this is what was entered on the keyboard search field, and click play to hear it):

b''4. a''4. g''4 f''4 d''4 c''2 d''4 f''8 e''8 f''4. d''4 c''8 d''8 c''4 a'2 b''4. a''4. g''4 f''4 d''4 c''2 d''4 f''8 e''8 f''4. d''4 c''8 b'8 c''4 d''4 

If anyone has any ideas I would greatly appreciate it. Thanks, --75.7.61.2 (talk) 08:27, 18 March 2008 (UTC)[reply]


[Irish accent] Before a, for -- gee -- for Ef$@*!idy -- foresee to the four effate afore! Before sea ate the ate sea, for -- aye -- too before! Aye, for! Gee, for afore the foresee to the fore for 88-F, for... . The foresee ate? Be ate, see! -- For the fore!

Makes perfect sense to me. I'm selling translations into English for $1,000 a pop, if anyone's interested.  :) -- JackofOz (talk) 10:17, 19 March 2008 (UTC)[reply]

First World War in Boys' Books (British)

I'm looking for information and leads on how the First World war was depicted in literature aimed at British boys in the early part of the twentieth century. I'm particularly interested in the way in which writers and publishers attempted to maintain a heroic illusion in the face of the realities of total war. Hope someone can help. Captain Wentworth (talk) 09:00, 18 March 2008 (UTC)[reply]

I'm afraid I can't help with the early part of the 20th century, but I can tell you that boys' annuals in the 60s and 70s were maintaining a heroic illusion in their telling of 1st and 2nd world war stories. It's easy really: you just don't mention the pathetic bits and dwell on tales of daring do. All of this was helped by the reluctance of those who came back from the trenches to talk about it. Skittle (talk) 13:31, 18 March 2008 (UTC)[reply]
Your starting point for this is definitely Biggles, though I would imagine you've found him already. He may be a special case as he started in adult books (I mean books aimed at adults, not adult literature) and became a boys character later. But he, and W.E. Johns other characters might get you started. DJ Clayworth (talk) 14:54, 18 March 2008 (UTC)[reply]

I think, Captain Wentworth, that you should begin by trying to understand the general structure of boy's fiction at the time, a genre that was remarkably resistant to change. In essence the parameters had been set even before the Great War, in the kind of fiction offered by the likes of G A Henty, Herbert Strang, Percy F. Westerman and Robert Leighton. You will also find the kind of themes these authors preferred-tales of individual heroism against a late imperial setting-in weeklies like The Boy's Own Paper (which my father remembers with some fondness!), Pluck and The Boy's Friend. The atmosphere and semiotics of these publications, and others in a similar vein, is wonderfully captured by George Orwell in his essay Boy's Weeklies.

So, the generation of 1914 grew up against the kind of plucky and chivalric sentiment expressed by Henry Newbolt in Vitaï Lampada, with war depicted in a uniquely English way as a game of cricket;

The sand of the desert is sodden red-
Red with the wreck of the square that broke
The gatling's jammed and the colonel dead,
And the regiment blind with dust and smoke.
The river of death has brimmed its banks,
And England's far, and Honour a name,
But the voice of a schoolboy rallies the ranks-
"Play up! Play up! And play the game!"

For some the actual experience of war, the unheroic ugliness of the whole thing, did nothing to moderate these fictional depictions. In 1915 Captain F. S. Brereton published With French at the Front, whose hero, Jim Fletcher, could have leaped straight out of the pages of Henty. He is killed in a German attack, calling on his men to fight 'for the sake of old England.' Inspired by his example the Tommies fight on "the thin khaki line of heroes, the cool, calm, cherry sons of Empire", beating back the Hun. It's romantic; it's glorious: it's a lie.

Brereton went on treating the war through a prism of rosy and heroic optimism. In his 1917 novel, Under Haig in Flanders, he paints a nice and cosy picture of life at the front, where the Tommies feed on "frizzling bacon, not to be beaten anywhere, bread that might have graced the table of a Ritz hotel, and jam that would have been the envy of any housewife." He goes on to depict the Battle of the Somme, with the great blood-letting of 1 July, one of the worst days in British military history, described as a "triumph for the Allies and a bitter blow to our ruthless enemy."

This kind of literature, and much more besides, clearly has a propaganda purpose, intended to attract more and more young men to the front with a promise of high adventure. The reality must have been truly shocking. But even after the war, when no further purpose was served by these fairy tales, the genre continued, largely unaffected by the revelations of Robert Graves, Wilfred Owen and Siegfried Sassoon. Brereton's novels remained in print throughout the inter-war period, and were popular as school and Sunday school prizes.

If you want to take this further I would suggest A War Imagined: The First World War and English Culture by Samuel Hynes (Pimlico, 1992). Clio the Muse (talk) 00:07, 19 March 2008 (UTC)[reply]

Are either Michael Keaton or Diane Keaton related to Buster Keaton? I know that both can't be as the article says they are not related to eachother. -- Q Chris (talk) 10:23, 18 March 2008 (UTC)[reply]

Well, Michael Keaton's name is actually Michael Douglas, so no; Diane Keaton doesn't seem to be related to him (she actually changed her name too, but Keaton was her mother's last name, so it's still possible I suppose). Adam Bishop (talk) 10:33, 18 March 2008 (UTC)[reply]
I have found an unverified "quiz answer"[17] saying : "the relationship is the brother to the mother of Diane Keaton". -- Q Chris (talk) 11:06, 18 March 2008 (UTC)[reply]
I can't find that mentioned anywhere else, and Buster Keaton's sister's name was Louise, whereas Diane Keaton's mother's name was Dorothy. FiggyBee (talk) 12:05, 18 March 2008 (UTC)[reply]

Carr, Thompson, and Hobsbawm

This book review quotes the editor of the book as saying, "anything that has been condemned by Carr, Thompson and Hobsbawm must have something to recommend it." I know who Hobsbawm is, but was wondering who the author of the quote might have been referring to by 'Carr' and 'Thompson'. These two, perhaps? --superioridad (discusión) 12:59, 18 March 2008 (UTC)[reply]

according to this essay, "Some professional historians have gone as far as to dismiss counterfactualism as, in the case of E.H. Carr, a “parlour game” and a “red herring.”2 Indeed, E.P. Thompson went as far as to condemn what he called “counterfactual fictions” as “Geschichtswissenschlopff, unhistorical shit.”3" (2 E.H. Carr “What is History?” (1961)3 E.P. Thompson “The Poverty of Theory” (1978)) SaundersW (talk) 13:07, 18 March 2008 (UTC)[reply]
Yes, those are the two. They are two of the best-known British historians (even a lazy American like me is well-acquainted with their works). Carr in particular, in his What is History?, makes quite a show of how silly some forms of counterfactual history are, like the idea that if Trotsky had not had a cold then Stalin would have never taken power, etc. --Captain Ref Desk (talk) 15:53, 18 March 2008 (UTC)[reply]

It might also interest you to know, superioridad, that the author of the book in question, Andrew Roberts, comes from an altogether different tradition from the 'three witches' of British Marxist academic history. His contempt for them is altogether more broadly based, going well-beyond their negative views on counter-factual history! Clio the Muse (talk) 00:30, 19 March 2008 (UTC)[reply]

Russian Jews in London

Years ago, when I was growing up in London's East-End, my great-grandmother told me about the generally hostile reception received by Jewish refugees fleeing the pogroms in Russia towards the end of the nineteenth century. I've been looking around here for any more details on this, though I can find little beyond a brief article on the Aliens Act of 1905. It would seem clear that there was widespread resentment against these Jewish asylum seekers, just as there is often resentment against other asylum seekers today. Does anyone know any more, particularly on how politicians reacted to popular pressure? Dora Kaplan (talk) 13:16, 18 March 2008 (UTC)[reply]

The arrival of large groups of destitute people, concentrated in already poor and overcrowded areas of the East-End, caused a rather unpleasant moral panic, Dora, one focused on fears for the racial purity of the nation. In 1904, Howard Vincent, the MP for Sheffield Central, wrote "While 260,000 people emigrated from the United Kingdom last year, their places were taken by no less than 82,000 of the scum of Europe." Arnold White, a leading eugenicist and racial theorist, considered most of the new arrivals in London to be 'diseased and destitute, a threat to British workers' While admitting that the Russian excesses had been 'regrettable', he said that this was based on a recognition by Russian statesmen that their country was threatened by a Jewish takeover. White was an influential figure, whose views of the Jews as diseased-ridden criminals, on the one hand, and exploitative international financiers, on the other, were widely disseminated. He included none other than Lord Salisbury, leader of the Conservative Party and three-times Prime Minister, among his correspondents.
It was Salisbury, while in opposition, who introduced a Bill in the House of Lords, calling for the expulsion of aliens, who 'threatened the peace and tranquility of the realm.' Destitute aliens, like those who were coming to the East-End, were also to be kept out.
In 1900, during the Khaki election, the anti-immigration East London Observer noted "Surely for Londoners, the election should have one object and that above all party politics. I refer to the presence in their midst of these foreign Jews." The cause was taken up by William Evans-Gordon, soldier, author and Member of Parliament. He co-operated closely with the British Brothers League in having a Royal Commission on immigration set, to which he was appointed chairman. It was as a result of this agitation that the Aliens Act 1905 was finally passed, though, in the event, it was far less restrictive than Evans-Gordon and his supporters would have liked.
It might also interest you to note that it was against this background that Bram Stoker's Dracula made its appearance, carrying themes of pollution by blood and disease, carried to England from Eastern Europe, giving it a particular relevance for the debates of the day. Clio the Muse (talk) 01:14, 19 March 2008 (UTC)[reply]

What's the geographic centre of Chinese politics?

Or, in other words, where does the Politburo meet? What's the Chinese equivalent of 1600 Pennsylvania Ave.? AlmostCrimes (talk) 13:39, 18 March 2008 (UTC)[reply]

Zhongnanhai. — Kpalion(talk) 20:03, 18 March 2008 (UTC)[reply]

Not sure...

...that this is the appropriate place for this question but it is the most likely (i think) to give me a helpful response. I work at a living history museum (pioneer village..though we eschew that title). We need the grass to look like it has been scythed but we don't have the manpower to scythe it all. We have used sickle bar mowers in the past BUT there are 2 problems: 1) the most reliable models/designs don't give us the uneven look that is historically accurate. 2) the brands & models we've used to date, break down too quickly.

Grazing animals might be ideal but would not have been tolerated in the front lawns of the more prominent families.

Any ideas/help would be most appreciated. —Preceding unsigned comment added by 24.226.90.6 (talk) 15:00, 18 March 2008 (UTC)[reply]

Advertise for volunteers - and give them a cookie when they've finished. Maybe put up an advert in a local college - those places are full of people with lots of energy and time.87.102.47.176 (talk) 16:44, 18 March 2008 (UTC)[reply]
Also you could get an uneven finish using strimmers see String trimmer87.102.47.176 (talk) 16:44, 18 March 2008 (UTC)[reply]
Why would grazing animals be a problem for the "more prominent" families? If they were good enough for the President of the United States? — Michael J 21:53, 18 March 2008 (UTC)[reply]
The string trimmer ( I call it a whipper snipper) mentioned above also has the added advantage of giving a scythe like pattern to the grass, as the operator does swing it from side to side in a fairly identical fashion and posture, using the pelvis as a fulcrum.
It is certainly faster (and much cheaper) than using a scythe but presumably slower than the sickle bar mowers. --Cookatoo.ergo.ZooM (talk) 22:20, 18 March 2008 (UTC)[reply]
Grazing animals have the mild disadvantage of depositing the metabolistic left overs of digested grass. Some visitors may turn up their noses at historical verism of such olfactory accuracy. --Cookatoo.ergo.ZooM (talk) 22:32, 18 March 2008 (UTC)[reply]
Expanses of scythed grass with no grazing animals in sight are anachronistic anyway. In what general region is this recreation? --Wetman (talk) 07:19, 19 March 2008 (UTC)[reply]
You could scythe the front lawns of the "prominent families" houses only; acquire sheep/geese/etc. for everywhere else. WikiJedits (talk) 13:25, 19 March 2008 (UTC)[reply]

Iran

I recently read some where, i have forgotten where, but, In Iran you can have gender reasignment surgery, but being homosexual is punishable by death, there fore my question is, before one goes for the op, surely one is gay, and therefore cant get it done because. some one please explain this. Am i stupid, misinformed or is there something I am missing, oh, and I fully support gay rights as a straigh dude. just thought that should be pointed out. Cheers and Beers —Preceding unsigned comment added by 193.115.175.247 (talk) 15:50, 18 March 2008 (UTC)[reply]

I think it depends on definitions. Most gays don't want gender reassignment, and people who want gender reassignment see themselves as "born in the wrong body" rather than gay. Also, I don't know about Iran but in most countries with homosexuality laws the law was against homosexual acts, so a non-practising homosexual would be OK. -- Q Chris (talk) 15:57, 18 March 2008 (UTC)[reply]
The assumption that one who chooses gender reassignment surgery is gay is the problem. There is some discussion of this in the gender terminology section of the transsexualism article. --LarryMac | Talk 16:04, 18 March 2008 (UTC)[reply]
The timing of your question suggests you are referring to the BBC TV program "Transexuals in Iran" which aired on BBC 2 on Monday 25th February 2008 at 9pm, or to stories in other media ancilliary to that, such as this BBC radio 4 Woman's Hour segment. From memory, one interviewee said that as Iran is a conservative society, those in charge like things black-and-white; that men should fulfil a traditional male role, women likewise, and that a man+woman+kids family is the only valid domestic configuration. Homosexuality challenges this structure, as it implies a more complex and varied social unit: the resulting grey-areas vex the conservatives. So the government actively promotes (and pays toward) gender reassignment surgery - it's an effort to make the vexing homosexual conform to that rigid nuclear family template. It also smacks a bit of expediency in the face of rigid religious rule - if the prevailing dogma is that man+man=bad, but doesn't say that men can't become women, then changing one man in the unit into a woman "fixes" the problem in a theologically compliant way. The program also featured an interview with a gay Iranian man who said he was pressured into having the surgery; I believe the program implied that many or most of the gender surgeries done in Iran were really homosexuals seeking to find a livable existence, rather than people who would (in freer circumstances) have chosen gender reassignment. -- Finlay McWalter | Talk 18:03, 18 March 2008 (UTC)[reply]
An older BBC story, in much the same vein, is here. -- Finlay McWalter | Talk 18:08, 18 March 2008 (UTC)[reply]
Is it true that merely being homosexual in nature is punishable in Iran, or is the punishment against homosexual acts? If a man told the authorities he was sexually attracted to men, but truthfully said he had successfully resisted these attractions and had never committed any homosexual acts, would he be in hot water? -- JackofOz (talk) 05:48, 20 March 2008 (UTC)[reply]

Please tell me this is a joke!

So scientologists are saying that this "xenu" took his people to Earth in a DC-8? What the fuck? Can someone verify this isn't some kind of hoax or vandalism? --TV-VCR watch 16:27, 18 March 2008 (UTC)[reply]

Not a DC-8, but a space ship that looks just like one. See Space opera in Scientology. It's all well verified. Friday (talk) 16:30, 18 March 2008 (UTC)[reply]
Well, it wouldn't be in Wikipedia if it were not well verified, would it? For comparitive purposes, Days of Our Lives is also well verified, as are the Bible and the Qoran ៛ Bielle (talk) 16:58, 18 March 2008 (UTC)[reply]
I think he means that it's verified that scientologists actualy believe that. It's worth mentioning, because if you didn't already know, you might think that it was some sort of negative smear/hoax to make Scientology look silly. 72.10.110.107 (talk) 20:23, 18 March 2008 (UTC)[reply]
Of course, people catalog it precisely to make Scientology look silly. Unlike, say, Christianity, which has adherents who catalog its absurd beliefs in earnest -- scientologists don't do the same. —Preceding unsigned comment added by 79.122.9.122 (talk) 21:57, 18 March 2008 (UTC)[reply]
Well, Wikipedia catalogs it because Wikipedia is an encyclopedia. To not do so would be an omission. 72.10.110.107 (talk) 15:22, 19 March 2008 (UTC)[reply]
One reason they do not want us using stem cells to grow new body parts is because as you grow older in most cases you grow wiser too - or at least are not as gullible. Selling odd stuff like off base religions requires an audience which has not yet been there.
Using rationality in arguments about inherently mystic and unverifiable concepts is, by definition, futile.
Freedom of opinion and speech must apply, particularly if one feels the opinion stated is absurdly ludicrous. --Cookatoo.ergo.ZooM (talk) 22:45, 18 March 2008 (UTC)[reply]

Scientologists actually believe this? I mean seriously, just chopping off the engines of a Douglas DC-8, putting it on a space background, and claiming it was what a green martian from some space agency used to fly millions of people to earth to blow them up with H-bombs? My brain is going to melt from how stupid this whole "religion" is. Cockatoo, this is more than absurdly ludicrous... --TV-VCR watch 00:36, 19 March 2008 (UTC)[reply]

As there are those among us who may feel that some of the most cherished beliefs of the world's religions (except our own, of course) are not dissimilar in logic or scientific evidence as to what is believed by Scientologists, it ill behooves the Ref Desk to fall into religious name calling. I strongly suggest this thread stop here before we get into winged humanoids, the raising the of the dead or 72 virgin attendants. ៛ Bielle (talk) 00:49, 19 March 2008 (UTC)[reply]
I agree... If Scientology is about all those things then I'm outahere. —Preceding unsigned comment added by 71.100.15.115 (talk) 01:54, 19 March 2008 (UTC)[reply]
I'm a Christian and not terribly fond of Scientology, but let's have the decency to show them some respect. I agree with Bielle, let's stop this name-calling now. AllenHansen (talk) 11:48, 19 March 2008 (UTC)[reply]
Scientology has all the ridiculous theology of a major religion without the millennia of history to justify it. Imagine if Christianity were invented in the 20th century. Wouldn't that be absurd? Adam Bishop (talk) 12:29, 19 March 2008 (UTC)[reply]
And when things bear signs of their own cultural context, the hand of their creator, it makes it harder to regard them as mystical. As an example, in Paradise Lost Milton has the angels fighting in heaven with muskets and cannons, which is pretty much the same effect as a religion revolving around using a DC-8. When we recognize the creators of a religion as people more like ourselves, we start to suspect them as being just as dubious as anyone we know these days. Personally I find Mormonism suffers from the same problem—it reads to me (a historian) like something could have only come out of the early 19th-century imagination. --Captain Ref Desk (talk) 19:27, 19 March 2008 (UTC)[reply]

Inflation, It's cause

18:05, 18 March 2008 (UTC)18:05, 18 March 2008 (UTC)~~Is failure to establish a constant legal value for the dollar a significant open door to inflation ? (Lack of "Gold standard") (Establishment of a "labor Unit" as measurement, for example ? —Preceding unsigned comment added by 67.164.84.229 (talk) 18:07, 18 March 2008 (UTC)[reply]

Empirically, no; at least in the sense that inflation a problem for gold standard currencies - see Gold standard#Disadvantages. Meanwhile Inflation#Causes of inflation may or may not assist you. --Tagishsimon (talk) 21:16, 18 March 2008 (UTC)[reply]
When the major currencies were directly defined by gold, the explosion (especially after the opening up of the Witwatersrand gold mines) in the quantity of gold in circulation had some big effects, not least in increasing the amount of capital available for economic development. We tend to think of inflation as always bad, but it isn't so simple. Xn4 21:27, 18 March 2008 (UTC)[reply]
hummm... a labor standard for the dollar. I like it. Trouble is that whether or not you extend it to mental labor or not you have the problem posed in the past by horses and today by machines. Even surgery has proven itself to be within the capacity of computer programming. Even computer programming now is within the ability of a neural networks to perform. What about intellect you say certainly human intellect can not be surpassed by a computer. Wrong again. Computers are fully capable of performing optimal classification and reducing multiple state equations to minimum form. Going by the capacity of a computer to play chess. I think we might be living with a dollar worth less than a few cents if we based it on human labor or the ability of human labor to get a job done.

old testament

what is inside the ARK of the Covenant? 24.182.28.106 (talk) 18:15, 18 March 2008 (UTC)[reply]

See Ark of the Covenant. --Sean 18:35, 18 March 2008 (UTC)[reply]

The Ark of the Covenant is described in the Bible as a sacred container, wherein rested the Tablets of stone containing the Ten Commandments as well as other sacred Israelite pieces.

Good isn't it.87.102.47.176 (talk) 18:39, 18 March 2008 (UTC)[reply]

Leadership of Muslim communities/mosques

Who other than the Imam is in the (religious/administrative/...) leadership of a Muslim community or mosque? So, if a Muslim community intends to build a mosque somewhere--who would lead negotiations? Are their official positions, inofficial leaders (i.e. simply those John Does who step forward and do it), etc.? I realize that it may differ between schools of Islam; I'd be most interested in communities most popular at the US East Coast and in Western Europe. Thanks for all help, Ibn Battuta (talk) 18:27, 18 March 2008 (UTC)[reply]

A ra'is can be anything from a community leader to a president. A qadi was/is also a community leader. Adam Bishop (talk) 22:15, 18 March 2008 (UTC)[reply]
This is not an authoritative answer, but in general I'd expect there to be a committee formed of individuals from the community who have taken the initiative, who have the respect and support of the community, and are willing to represent it. Negotiations would be held with the committee, of which a few members may have been more specifically designated to be the primary spokespersons for outside contact, while other members may be more engaged with rallying support and fund raising in the local Muslim community. One of these committee members may happen to be an imam, but in general imams are supposed to be spiritual guides rather than administrators.  --Lambiam 23:32, 19 March 2008 (UTC)[reply]

The page referenced says that the stones therein were made of sapphire.

Question: what is the interpretation of sapphire here - slabs of corundrum seem unlikely, could they be by any chance made of lapis lazuli?

Your expertise, guesses and links appreciated.87.102.47.176 (talk) 18:42, 18 March 2008 (UTC)[reply]

One rather down-to-earth explanation I've seen is that the Hebrew word for book, sefer is related to the word for sapphire, sapir. I gather sapir was used rather generally for things of great value, so when the Talmud mentions a sapphire-like stone that shouldn't be taken literally as meaning a blue stone. But Hebrew isn't my subject. Xn4 22:15, 18 March 2008 (UTC)[reply]
Our article states that sapphire was not known until the emergence of the Roman Empire and does, indeed, suggest lapis lazuli (used by the Egyptians in Mosaic times). As Xn4 above, I am neither a Bible scholar nor a Judaist. --Cookatoo.ergo.ZooM (talk) 23:01, 18 March 2008 (UTC)[reply]
I'm a "Judaist" (I guess) but not a Bible scholar, either. I did look up Tables of the Law in the early-1900s Jewish Encyclopedia, and it says that the "sapphire" of the Tables was "was of a nature that admitted of the tables being rolled up." So clearly we're not talking normal sapphire here. The material was "quarried from the solar disk." The use of the word "sapphire" in various languages to describe the Tables apparently dates to the Early Medieval period or Late Antiquity. The writers of the Midrashim presumably thought it was the best word to describe the literally other-worldly material of the Tables. -- Mwalcoff (talk) 23:35, 18 March 2008 (UTC)[reply]
@ 87.102.47.176, I was in a mini-conundrum about your word "corundrum", assuming it was a mis-spelling of carborundum. But then I discovered you must have been referring to corundum, something I had never heard of. So thanks for the opportunity to expand my vocab. -- JackofOz (talk) 01:20, 19 March 2008 (UTC)[reply]
The word carborundum is a portmanteua formed from carbon + corundum. Let us hope you will never be in a carbonumdrum.  --Lambiam 23:40, 19 March 2008 (UTC)[reply]
As far as I understand it, the tablets themselves are only identified in Biblical texts as being of "stone". The sapphire reference occurs a few verses earlier (in Exodus 24:10), in reference to what Moses and his cronies see as they are brought into the presence of God. Since they are (presumably) prostrate, they only see the ground at the feet of God, which appears "like the working of a pavement of lapis lazuli." I am not a Hebrew scholar, but the relevant phrase is:
כמעשׁה לבנת הספיר
Gwinva (talk) 01:43, 19 March 2008 (UTC)[reply]
In this case, the Hebrew word sappir predates the English word "sapphire" by a couple thousand years or so. According to the Oxford English Dictionary, sappir appeared in the Septuagint as "sapphiros" (with both the "p" and "ph" pronounced). The Greeks seem to have adopted the word "sapphiros" to refer to both the Biblical "sapphire" and to lapus lazuli. When "our" sapphire was discovered, the Greeks probably called it "uakinthos" and the Romans called it "hyacinthus." Latin must have adopted the "sapphirus" to refer to our sapphire before it split into the various Romance languages of today. English takes the term from Old French. Incidentally, the OED says the word "sapphire" is probably not of Semitic origin. "Some scholars have conjectured" that it may ultimately be a Sanskrit word meaning "dear to the planet Saturn" and referred to "some dark gem" -- perhaps even sapphire! -- Mwalcoff (talk) 02:07, 19 March 2008 (UTC)[reply]

ok thanks that was very interesting - I don't know the hebrew alphabet, or hebrew so on any interpretations of translations I rely on you. So... When an english bible says 'sapphire' it means 'lapis lazuli' or maybe lazurite or at least not the sapphire of today, and possibly just means a special material. Hoshen says a similar thing. Another question(s)

"quarried from the solar disk."

Anything you can tell me surrounding this would be great. Are there any other mentions of a 'solar disc' in this literature. Is it anyway similar to the 'vault of the heavens'?83.100.183.180 (talk) 14:13, 19 March 2008 (UTC)[reply]

That's the 1906 Jewish Encyclopedia translation of a word in Canticles Rabbah, also known as the Song of Songs Rabbah, an Early Medieval Midrash. You'll need to go to a really big library (or a Jewish library) to find more information -- unless there's someone out there with quite a home collection of Jewish books. -- Mwalcoff (talk) 23:33, 19 March 2008 (UTC)[reply]
The "solar disc/disk" is simply the sun, as seen from earth. That is, people who believe in a "solar disk" believe the sun is a disk in the sky. So the "solar disk" is hanging from the "vault of the heavens" in this primitive cosmology; a "solar disk" can also be the depiction of this solar disk (as in hieroglyphics). I confess I don't see any reasonable explanation for something being mined from the solar disk, though what unreasonable explanation there is I can't say. - Nunh-huh 06:27, 20 March 2008 (UTC)[reply]
Ok thanks - might have been just a single reference, don't think I'll be able to follow this up right now. Maybe in the future. Thanks.83.100.183.180 (talk) 13:32, 20 March 2008 (UTC)[reply]

First black woman mayor in the United States

Mrs. Dessie Lee Patterson of South Mansfeild, Louisian is the first Black woman mayor in the United States, elected in March of 1971. Is there any documentation of this woman and her accomplishments?

Sincerely,

Vicki Lynn Mayweather (email removed)—Preceding unsigned comment added by 130.13.161.21 (talk) 22:12, 18 March 2008 (UTC)[reply]

Hi Vicki Lynn, your question will get attention without the all caps header, thanks. The protocol is to read the big box at the top of this page, check for your item in the search box, then ask your question as you did, without an email. Best, Julia Rossi (talk) 22:40, 18 March 2008 (UTC)[reply]
According to various sources, e.g. this and this, she was the first black woman mayor in Louisiana, and she wasn't elected in 1971, just appointed (although she did win several mayoral elections later on). Clarityfiend (talk) 00:32, 19 March 2008 (UTC)[reply]
Unfortunately, they're not freely avaiable sources, but maybe your library has one of these publications or can help you get the respective pages. --Ibn Battuta (talk) 00:44, 19 March 2008 (UTC)[reply]


Lutheran Hymns

What were three or four of the most popular Lutheran hymns in the 1760s? AllenHansen (talk) 12:05, 19 March 2008 (UTC)[reply]

These two websites The Lutheran Hymnal, 1941 and The Lutheran Hymnal Online offer versions of the Lutheran hymn book that include the dates the hymns were written. That doesn't tell you which were most popular, but you could at least start making a list of period-correct ones. WikiJedits (talk) 13:51, 19 March 2008 (UTC)[reply]
Ah, hang on, try this one instead: Lutheran Hymnody. It seems "hymnody" is the correct search term. WikiJedits (talk) 14:02, 19 March 2008 (UTC)[reply]
Thanks. Maybe I should rephrase my question. What were some of the most widely sung Lutheran hymns, in German, in the mid-1700s, especially the 1760s. A lot of the hymns in the 1941 Hymnal were obviously not sung by them at the time. AllenHansen (talk) 17:32, 19 March 2008 (UTC)[reply]
Let me take a stab at this ... The second half of the 18th century was a time of great simplification in German, specifically Lutheran, church music, due to the combined effects of the Enlightenment and the stile galant, somewhat as a reaction to perceived excesses of opulence in the preceding period. Extravagant music, such as the cantatas, oratorios, and Passions of J.S. Bach, of was pretty much a thing of the past, -- for a while. The same hymns were often sung as in the first half of the century -- Ein' feste Burg ist unser Gott, Christ lag in Todesbanden, Vater unser im Himmelreich -- but not necessarily with the same settings. Worship services consisted of hymns, readings, prayers, and preaching, with a much diminished importance of music for its own sake. Many, if not most, of the chorales you can find in the 371 Four-Part Chorales of J.S. Bach were still sung, though according to the article in the New Grove, some of the settings were recomposed in accordance with the taste for lighter styles. Antandrus (talk) 00:39, 20 March 2008 (UTC)[reply]
Addendum: a good way to research this would be to take the list of hymns in the 371 by Bach, and then look through one of those Lutheran hymnals for post-Bach harmonizations of the same hymns -- specifically if you can find some dated to the 1750s and 1760s (most good hymnals give the source for the tune, as well as the composer and date of the harmonization). You may find the voice-leading and harmonies somewhat simplified from the versions by Bach. Antandrus (talk) 00:42, 20 March 2008 (UTC)[reply]

March 19

farm animal as pet animal B.C. Canada

I remember there was a news in Canada where in British Columbia where woman has a dwarf horse, which is a farm animal and she wants it as a pet but people and mayor of the town, which I didn't get the name, said it is not right to do that and they bring this matter into the court. Where I can find this news report? I ask this because I think it could change the law of Canada. —Preceding unsigned comment added by Don Mustafa (talkcontribs) 01:12, 19 March 2008 (UTC)[reply]

Could it be http://www.canada.com/victoriatimescolonist/news/story.html?id=1a0bca94-57f0-49e4-9fa5-4f3edff82305? --Bowlhover (talk) 02:06, 19 March 2008 (UTC)[reply]

History assigment-help please

"The political crisis that overtook Scotland and then Britain in the period from 1637 to 1640 was not about Prayer Books; it was not about Bishops; it was about Power". It's a history assignment I have to do over the easter break. I am not asking for an answer. I'm just asking for some clues and some guidance. I know about the Prayer Book crisis and I know about the Bishops Wars. I'm just not sure how to put this information together to get the kind of answer looked for in the question. Help!Donald Paterson (talk)

Well, any political crisis is about power. The question itself is poorly crafted, it's basicaly telling you to come up with the answer they want to hear. Anyway, the Bishop's War was about Bishops, because secular and ecclesiastical power were intertwined. People had to attend church, christenings, marriages and deaths were recorded, invaluable information in pre-census days, so to change the hierarchy to a peer-based system, would hurt the King's authority. The actual church doctrine wasn't as important as who they answered to.

The prayer books, to put it simply, were how God heard the people. Charles was imposing his way on someone with a different set of beliefs. Basicaly, the conflicts were about who the church's power went to. I hope someone more knowledgable will chime in. AllenHansen (talk) 12:31, 19 March 2008 (UTC)[reply]

OK, Donald, yes, it was about power, understood in a number of senses: the loss of power by large sections of the Scottish aristocracy following the Union of the Crowns in 1603; the acquisition of power by the Scottish episcopacy as agents of the crown; the misuse and abuse of prerogative power by a distant king. You should have a look at the page on the Covenanters, which covers these points in some detail.
In your position I would begin by looking briefly at the rule of James VI in Scotland, particularly in relation to the question of church government. The separation of the aristocracy from the more radical elements in the Presbyterian party had been one of James' political successes, enabling him to build a new national church in the north on an Episcopalian basis. When in London James, secure in his power and his peace, was famously to say that he was able govern Scotland 'by pen.' But this form of government was always fragile. It depended on James' understanding of the complexities of Scottish politics; it depended on his familiarity with the leading personalities of the realm; it depended, above all, on his ability to ensure that the nobility had access to office and position. But, in the end, there was only a limited amount of this to give.
Charles' reign started badly in the north. He alienated the nobility by an Act of Revocation, which threatened to claw back all former clerical lands that had been secularised since the Reformation. He did little to win over the Scottish nobility thereafter, preferring to restrict his counsels to a small and Anglicised group of favourites; people like James, Marquess of Hamilton. Scotland was thus left with a large group of suspicious and underemployed aristocrats. It was bad enough that most of these people had little or no access to the king, far removed in London; what made matters far worse was that, from the mid 1630s, Charles began to fill vacancies in the Scottish Privy Council from the panel of bishops, including the post of Chancellor, the most powerful of all. Jealous of the growing power and influence of their Episcopalian colleagues, the nobility only needed a cause to give their immediate and long-term resentments a precise direction. It came in 1637, when Charles insisted on the adoption of a new Anglican-style Prayer Book without taking any soundings at all from the aristocracy.
For the Presbyterian dissidents in an Episcopal Church opposition to the Prayer Book was an ideal cause around which to unite. But it may have come to nothing but from the support they obtained from the nobility: men like James Graham, Earl of Montrose and subsequently Archibald Campbell, Lord Lorne. A new and dangerous political alliance had been created, destroying all the work of James VI. The Presbyterians were able to challenge the power of the Bishops in the Church, just as the Nobility was able to challenge the power of the Bishops in the state. Scottish government was, in effect, completely removed from the crown, as both the Presbyterians and the Nobility went on to challenge the power of the King himself in the Bishops' Wars.
Charles was on a downward spiral. Unable to control events in Scotland he also lost control of events in England. In the end he lost his head, in more ways than one. The best of luck with your assignment. Clio the Muse (talk) 01:39, 20 March 2008 (UTC)[reply]

Purchasing power of US dollar circa 1900

What was the purchasing power of a dollar in 1900 expressed in terms of a dollar today? F Chiles (talk) 11:58, 19 March 2008 (UTC)[reply]

Follow the top links here... WikiJedits (talk) 13:53, 19 March 2008 (UTC)[reply]
Thus: $25.47 in the year 2007 had the same purchasing power as $1.00 in the year 1900. Conversely, $0.04 in the year 1900 had the same purchasing power as $1.00 in the year 2007. (Joseph A. Spadaro (talk) 22:40, 19 March 2008 (UTC))[reply]
There are lots of ways of calculating historical currency rates—it's a case of what indicator you choose to use, and which one you choose to use (ideally) has to do with what you are thinking of purchasing with that dollar. If you're talking about the price of food or rent, the CPI is a good one to go with; if you're comparing government funding of science, the relative share of the GDP makes more sense, etc. Ideally your indicator will be most like the proposed purchasing that you are considering. Additionally, some of the indicators change more over time; food might be less of an expense today (or more of one, I don't know) than it was before, due to improvements in technology, expectations, availability, etc.
I've found this site the best overall conversion site, as it does all of the various types for you and let's you see how much variance there is. Note that you can't reliably do it for 2008, because to calculate these things you need values that won't be published until some time after the year is over, but you can go up to 2006 or so. It also describes the relative strengths and weaknesses of using different indicators, and gives some concrete examples of the sort of things you might use with one or the other. --Captain Ref Desk (talk) 01:24, 20 March 2008 (UTC)[reply]

Most prolific fathers

Could someone provide a list of the most prolific fathers. And a list of the numbers of children they fathered.--Gary123 (talk) 14:06, 19 March 2008 (UTC)[reply]

You should check out the Guiness Book of Records --Dweller (talk) 15:39, 19 March 2008 (UTC)[reply]
Brigham Young had at least 56 children. Osama bin Ladin has about 50 siblings. - Nunh-huh 15:47, 19 March 2008 (UTC)[reply]
Living, it might be this man with 78 and counting. Historically, it's possibly Genghis Khan.[18] It would be very hard to devise a complete, well-referenced list. Marskell (talk) 15:59, 19 March 2008 (UTC)[reply]

Agricultural commodity prices

What is a website where I can find basic commodity prices. I don't want one with a some big fancy graph that I can't understand or lots of big confusing numbers. I just want the prices. Ive been looking for awhile now and haven't found anything. Thankyou. —Preceding unsigned comment added by 64.119.61.7 (talk) 14:12, 19 March 2008 (UTC)[reply]

There is a table listing agricultural commodity prices on the right side of this page when the Chicago Mercantile Exchange is open for trade. Any website that keeps commodity prices current is going to be oriented to serious traders, who will want more information than you may want. Still, you should be able to find the information you want on this site or other sites that publish commodity price quotes. Marco polo (talk) 14:25, 19 March 2008 (UTC)[reply]

bible, talmud etc other books

The old testament etc really seems to be a mine of hisorical information. Are there any other sources that give a similar level of insight into 'bronze/iron age' societies and customs, outside these judaic related texts. (I've already considered egyptian heiroglyphs) - doesn't have to be middle eastern.83.100.183.180 (talk) 14:18, 19 March 2008 (UTC)[reply]

Lots! (You want written documents, rather than archaeological data, right?) The History of Literature page is a great jumping off point. The Vedas are packed with cultural details, and you can look at the Epic of Gilgamesh, Book of the Dead and related items, early Chinese writings, and lots more. WikiJedits (talk) 14:47, 19 March 2008 (UTC)[reply]
Yes thanks.83.100.183.180 (talk) 17:10, 19 March 2008 (UTC)[reply]
The Talmud is a wealth of information, but for the early centuries AD. AllenHansen (talk) 17:25, 19 March 2008 (UTC)[reply]
Don't forget the Amarna letters. -- Mwalcoff (talk) 23:22, 19 March 2008 (UTC)[reply]

The Light

In near death experiences and the like, eg paranormal phenomenon, people are told to eith go or not go into the light. I wish to read our article if there is one on The Light —Preceding unsigned comment added by 193.115.175.247 (talk) 16:41, 19 March 2008 (UTC)[reply]

Take a look at our article on Near-death experience. Personally, I am suspicious that they are anything other than brain misfirings correlated with cultural expectations and experience, but I am without doubt a skeptic. --Captain Ref Desk (talk) 17:00, 19 March 2008 (UTC)[reply]

Was there a gap between the last Roman Emporer and the first Pope, or did they overlap? Was there a transition when Emperors became Popes, or are they two distinctly different and unconnected roles? 80.0.107.56 (talk) 17:00, 19 March 2008 (UTC)[reply]

Emperors and Popes are two different things. Emperors were typically crowned by Popes. Sometimes Emperors even disposed of Popes. To my knowledge no Emperor has been a Pope or vice versa though perhaps someone knows better than I. --Captain Ref Desk (talk) 17:03, 19 March 2008 (UTC)[reply]

80.0.107.56, did you read the two articles that you linked to? Anyway, according to Roman Catholic tradition, the first pope (bishop of Rome) was Saint Peter in AD 33. Traditionally, the last emperor of the Western Roman Empire was Romulus Augustulus. In AD 476, he lost his throne to Odoacer who chose not to use the imperial title and styled himself King of Italy instead. So between 33 and 476, you've got 443 years of overlap. The Roman imperial title was revived by Charlemagne when he was crowned emperor by Pope Leo III in AD 800. During the Middle Ages, the Holy Roman Empire, created in AD 962, and the papacy were the two major politial powers in Europe, often in conflict with each other (see Investiture Controversy). Arguably, the Austrian Empire (later Austria-Hungary), which existed from 1804 until 1918, was a successor state to the Holy Roman Empire, so from 962 to 1918, you've got another 956 years of overlap. Tha papacy had its ups and downs, but it has existed continuously for the last two millenia. — Kpalion(talk) 18:14, 19 March 2008 (UTC)[reply]

And meanwhile, the real Emperor continued to rule in Constantinople until 1453, happily frustrating the Pope for centuries! Adam Bishop (talk) 18:28, 19 March 2008 (UTC)[reply]
And the tradition of the Eastern Roman Empire (and of frustrating the Popes of Rome) was later continued by the Russian Emperors until 1917. — Kpalion(talk) 18:49, 19 March 2008 (UTC)[reply]
At the same time of the Ottoman Sultans also claimed of being the heirs of the Byzantine emperors. There were simply more than one emperor at the same time (sometimes as much as 3), all of them claiming to be the successors of the old Roman emperors (and hence political heirs of Julius Caesar). First there was the emperor of the Byzantine empire (which was called "Roman empire" by its inhabitants). Then in AD 800 the pope crowned Charlemagne as Roman emperor. After a while the Holy Roman Empire of the German Nation was portrayed by its emperors to be the revival of the Roman Empire and backed by the Catholic Pope. This irritated the Byzantine Basileus which were backed by the Orthodox church. After the Fall of Constantinople the Ottoman Sultans and the Russian Czars claimed to be its heirs. The term "real" is very subjective. Flamarande (talk) 19:14, 19 March 2008 (UTC)[reply]
I believe Adam used the word "real" humorously. By the outbreak of the First World War, Europe had at least three, if not more, emperors, all of whom, in some way, claimed to be heirs to the traditions of the ancient Roman Empire. Their imperial titles derived either from the Latin word imperator (French empereur, Russian император) or from the cognomen of Gaius Iulius Ceasar (German Kaiser, Bulgarian цар). And it's not like there was always only one pope at a time. — Kpalion(talk) 20:36, 19 March 2008 (UTC)[reply]
Part of the confusion may arise from the fact that the Roman Catholic pope has appropriated some of the titles and other marks of nobility previously used by the ancient Roman emperors. See, for example, Pontifex Maximus. -- 128.104.112.85 (talk) 22:08, 19 March 2008 (UTC)[reply]

80.0, you might also wish to look at Caesaropapism, though this is more relevant to the Eastern Church.

I am not sure if Adam was serious in referring to the rulers of the Byzantine east as the 'real' emperors, though I suspect any humour intended was draped in the irony common among historians. I, though, am quite serious in insisting that they were the real emperors, intending no irony whatsoever! The occupants of the imperial throne in Constantinople were real in the sense that they had their authority and legitimacy from Constantine the Great, and through him all the way back to Augustus. Odoacer did not 'usurp' the imperial power; in removing Romulus Augustulus he merely ended its division, all nominal authority handed back to Zeno and his successors in Constantinople. One Emperor; one Empire; one God. Charlemagne, by this measure, was a parvenu; a pretender crowned by a western pontiff on no certain authority, simply because the throne of the Roman world was occupied by a woman! Alas, preserve us all from barbarians playing at being Romans! Clio the Muse (talk) 02:14, 20 March 2008 (UTC)[reply]

Clio, is your last sentence a reference to the toga party, perhaps? That's what it made me think of, anyway. I think a case could be made that barbarians playing at being Romans (Theodoric comes to mind in addition to Charlemagne) are preferable to barbarians serious about being barbarians. Deor (talk) 02:33, 20 March 2008 (UTC)[reply]
Toga parties, yes, that's good! Barbarians playing at Romans merely serves to highten their conceit-it does not make them any less barbarous! Clio the Muse (talk) 02:43, 20 March 2008 (UTC)[reply]
I was half-joking, but people do tend to forget that the Roman Empire didn't disappear at all, it just wasn't in Rome anymore. Adam Bishop (talk) 03:35, 20 March 2008 (UTC)[reply]

Enjoyable eastern and other non-western ancient classics?

What eastern classics would people recommend reading for pleasure? Similarly, are there other non-western classics people would suggest? 80.0.107.56 (talk) 17:03, 19 March 2008 (UTC)[reply]

Well, depending on what you define as "eastern", "classic" and "enjoyable", I venture to suggest one of the most "enjoyed" such texts would be the Kama Sutra. 1001 Arabian Nights is also quite entertaining. Book of Esther is also a right riveting read, and topical too, as the anniversary of its denouement is this coming Friday. --Dweller (talk) 17:14, 19 March 2008 (UTC)[reply]
The Mahābhārata and the Ramayana are, without question, great works. Try also The Tale of Genji. Corvus cornixtalk 18:22, 19 March 2008 (UTC)[reply]
On the other hand I found The Tale of Genji to be one of the most boring books I’ve ever read so I guess it’s a matter of taste. . . --S.dedalus (talk) 06:40, 20 March 2008 (UTC)[reply]
I've never read Popol Vuh, but you might want to look at that, too. I don't know anything about African literature, unfortunately, to know what to recommend there. Corvus cornixtalk 18:44, 19 March 2008 (UTC)[reply]
Romance of the Three Kingdoms from china could be compared to if anything shakespeares english war plays - lots of battles.83.100.183.180 (talk) 18:26, 19 March 2008 (UTC) Note I haven't actually recommended it but have heard that many have enjoyed it.[reply]
Genji is a very slow read... I'd recommend the Epic of Gilgamesh which has a lot of story (some of which you may find familiar in other settings). SaundersW (talk) 18:56, 19 March 2008 (UTC)[reply]
Also from China, Dream of the Red Chamber aka Story of the Stone. Family, by Ba Jin, is a modern classic (early 20th century) of Chinese literature, along with Lu Xun's short stories (including Diary of a Madman, Medicine, The True Story of Ah Q and others). Steewi (talk) 00:56, 20 March 2008 (UTC)[reply]

Shahnameh.--Goon Noot (talk) 03:57, 20 March 2008 (UTC)[reply]

The Pillow Book of Sei Shōnagon. -- Julia Rossi (talk) 06:53, 20 March 2008 (UTC)[reply]
Try The Art of War,the Tao Te Ching, The Setting Sun, Dream of the Red Chamber, and Ikite iru Heitai by Tatsuzō Ishikawa. Hope that helps, --S.dedalus (talk) 06:58, 20 March 2008 (UTC)[reply]

grandville sharp rule

I was wanting to ask a question about Grandville Sharps rule on Matt 28:19 when it states that if the article "the" is used more than once it is referring to different persons example: in the name of the Father and the Son and the Holy Ghost does this rule apply also to the scripture Acts 7:32 I am the God of thy fathers, the God of Abraham and the God of Isaac and the God of Jacob. Does this rule apply to this scripture and if not can you tell me why not? Thank you Kennyt77 (talk) 20:18, 19 March 2008 (UTC)[reply]

Granville Sharp was, according to our article, writing about supposed mistranslations of the greek, and expressing his view of a rule pertaining to the greek original. I'm not sure that the rule is applicable to the English translation, for a couple of reasons: the English translation may be suspect (according to GS), and the rule may pertain only in Greek grammar. --Tagishsimon (talk) 21:15, 19 March 2008 (UTC)[reply]

Chinese instrument

I am looking for a Chinese (or perhaps Japanese) instrument that is pronounced "who", but am unsure of the spelling or type of instrument. Unfortunatley these are all the details I have - Google and Wikipedia haven't helped me out much here. Thanks in advance. 92.0.118.76 (talk) 20:38, 19 March 2008 (UTC)[reply]

Have you had a look through Category:Chinese musical instruments, which sports such things as the Zhu (string instrument) ... or List of traditional Chinese musical instruments? Ditto Category:Japanese musical instruments and Traditional Japanese musical instruments. Good luck. You /might/ want to ask this on the language desk, since someone there may have a clue about whjatever chinese word might resemble the English "who". --Tagishsimon (talk) 21:12, 19 March 2008 (UTC)[reply]
Do you perhaps mean the erhu? It's pretty well-known in the west, and is sometimes known as the "Chinese violin". Antandrus (talk) 00:20, 20 March 2008 (UTC)[reply]

Who owns the freehold of London's roads & why

I'm playing around with articles on London roads & their names right now. And it occurs to me to ask, having read history after history of rich knobs building estates on their land ... who owns the roads? Case in point might be Tottenham Court Road, but the same might as easily be asked of Downing Street or Bedford Square or a hundred others. At the time of their development, the land beneath the roads was squarely owned by a Fitzroy or a Duke of Bedford or whoever. Who now owns the freehold? If not the landed estate (e.g. the Bedfords still own an unheathy amount of property in London), then, in general terms, how & when and at what cost was the freehold passed to the borough authority? --Tagishsimon (talk) 21:08, 19 March 2008 (UTC)[reply]

I would suspect they count as a public road, i.e. part of the UK's roads - owned and maintained by the government at a cost to the tax-payer through Road Tax. That is unless they remain private roads which could allow them to introduce a toll (like a bridge near my home) to use it. I have no idea about the history of road-ownership but I suspect that the land-owners will have received some form of compensation for their troubles. There is the case of that weird house in the middle of the Pennines that has a bit of history to it (http://www.guardian.co.uk/notesandqueries/query/0,5753,-2193,00.html) around government offering money for the land but the farmer refusing to sell). ny156uk (talk) 22:44, 19 March 2008 (UTC)[reply]
According to some of the Notes & Queries replies, and also our article, the house in question remained in place because the M62 motorway had to avoid it for engineering reasons. On your earlier point, roads in the UK are maintained out of general taxation: there has been no such thing as 'Road Tax' for many years. AndrewWTaylor (talk) 23:12, 19 March 2008 (UTC)[reply]
It may not be that the crown owns the freehold to all public roads. In English law, there is a concept of right of way whereby the public holds a right of access across privately owned land. To determine the freehold status of a given road, you might need to consult the records at HM Land Registry. Marco polo (talk) 00:44, 20 March 2008 (UTC)[reply]

Why are there no term limits for US Congress, Senators, and Federal Judges? I know the obvious answer is because the US constitution doesn't impose term limits. But my question is why shouldn't there be term limits? What is the argument for keeping this system. There seem to be arguments against them (i.e., the same arguments that give us term limits for other offices), but I don't quite see the argument for keeping it. Should the constitution be amended? Llamabr (talk) 21:46, 19 March 2008 (UTC)[reply]

One of the arguments that I have heard is that "term limits" for congressional representatives happens naturally every other November. That is, if the electorate does not like a particular representative, they are always free to not reelect them. The argument is that it would be improper to countervene the will of the electorate and force a "good" representative out of office, just because he's been reelected a number of times. - Now whether you think that is a good argument is another question. (I'm also not sure of your implication as to the prevalence of term limits in the U.S. Aside from the President, I'm of the impression that term limits are the exception, rather than the rule.) -- 128.104.112.85 (talk) 22:02, 19 March 2008 (UTC)[reply]
According to the article United States federal judge the Federal Judges don't have any kind of terms and aren't elected. They are appointed by the current President. Flamarande (talk) 22:11, 19 March 2008 (UTC)[reply]
Term limits force elected officals out of office at the end of their term. This limits the right of the people to choose to reelect someone who they feel is doing a good job. The people already have the right to impose a term limit themselves by merely not reelecting someone. Thomprod (talk) 01:45, 20 March 2008 (UTC)[reply]
Back in the early to mid-90s, when the term-limit movement was at its height, the group U.S. Term Limits tried to get term limits imposed on members of Congress. They got initiatives passed in some states putting term limits on U.S. representatives from those states, but the U.S. Supreme Court ruled that those limits were unconstitutional. -- Mwalcoff (talk) 04:26, 20 March 2008 (UTC)[reply]
As for "why the President and not Congress"... Congress amends the Constitution. The President just takes the blame. So, Congress is happy to impose a term limit on the President. Don't expect them to impose one on themselves. They are too busy giving themselves raises (with free health care) and blaming whoever the current President is for the cost. -- kainaw 02:03, 20 March 2008 (UTC)[reply]

Vehicle registration and inspections

When a rental agency licenses a vehicle in a particular state which requires emissions testing or safety inspections and then the vehicle is driven out of that state, what happens with the tests or inspections? The vehicle may not be back in that state, possibly, for the rest of its ownership by the company. So does the rental company just submit paperwork on all of its thousands of vehicles to the various states or is there some special article of the laws that these companies fall under saying that the vehicle may not be in the state and thus aren't required to get the vehicle tested? Or is it a matter of policing themselves and they are obligated to have them inspected when the time has come and if the vehicle is in the necessary state? This is concerning the laws in the U.S. by the way.... Dismas|(talk) 22:59, 19 March 2008 (UTC)[reply]

Who's to say it won't be back? Every time I've rented a car, I've been informed that if I don't return it to the point of origin, they'll add a $100 relocation fee to my bill. --Carnildo (talk) 23:53, 19 March 2008 (UTC)[reply]
What? I didn't say anything about the customer not returning it to the place that it was rented from. Dismas|(talk) 00:00, 20 March 2008 (UTC)[reply]
So, if the customer either returns the car to the place where it was rented or the rental company returns the car to its place of origin if the customer doesn't, how would the car end up in a different state? In any case, what matters is where the car is registered. The car can be driven anywhere as long as it meets the registration requirements in the state where it is registered. The car must be registered wherever it is regularly kept by its owner, in this case the rental company. For the car to end up in a different state (other than through an accident), the rental company would have to decide to transfer it. It would then need to meet the registration requirements and be registered in the new state. Marco polo (talk) 00:09, 20 March 2008 (UTC)[reply]
Okay, I read some sort of nefarious purpose to Carnildo's response. I'm not implying anything like that. So if the customer returns it to a previously agreed on location, which is not in the same state, what happens? In the last month I've rented two vehicles, both here in Vermont, which have had out of state plates. We have yearly inspections here. So if a Vermont registered vehicle is dropped off in another state, what happens? Dismas|(talk) 00:19, 20 March 2008 (UTC)[reply]
I think that if the rental company intends to keep a car in a different state from the one where it was registered, it needs to register the car in the new state. I know that in Massachusetts, the car's owner is legally required to register a car in Massachusetts within a certain number of days or weeks of garaging a vehicle here (i.e. regularly parking it here overnight). Of course, plenty of people maintain out-of-state registrations on vehicles kept in Massachusetts in order to save on insurance, but it isn't legal. I don't know whether car rental companies skirt these laws. Marco polo (talk) 01:23, 20 March 2008 (UTC)[reply]

Bharata Janata Party West Bengal and Tripura leaders

Who are the leaders of the Bharata Janata Party of West Bengal and Tripura? —Preceding unsigned comment added by Don Mustafa (talkcontribs) 23:01, 19 March 2008 (UTC)[reply]

The best way to find out would be to email them directly an ask; details can be found on the BJP website under "State Offices" Samilong (talk) 11:50, 20 March 2008 (UTC)samilong[reply]

March 20

H Donald-Frith, portrait painter

Was there a portrait painter with a name like this? - Kittybrewster 00:46, 20 March 2008 (UTC)[reply]

There was a William Powell Frith, an English portraitist. Is that useful? ៛ Bielle (talk) 02:38, 20 March 2008 (UTC)[reply]

I live in an area (40 by 40 blocks) where homelessness is rampant. Area businesses have learned to cope somewhat with people begging for money outside of their establishments. The area was originally developed to provide low cost housing for State university students but for the most part now has been turned into section 8 (low cost government funded) housing. The crux of the problem is drug and alcohol abuse for the hard core homeless and a mental inability or unwillingness to work to earn money for rent, food or clothing on the part of most of the remainder. I'm looking for a private or public agency or project or some other way to address all parts of this socio-economic problem. My thoughts are that some form of mental disability award could be made, but there are many critics to this solution who do not live in or near an area with this problem or stand any chance of ever having this problem themselves. These people need a life jacket or lifeboat and I have no idea where to turn. 71.100.10.177 (talk) 09:50, 20 March 2008 (UTC)[reply]

If you're in Alberta, I suggest you contact your MLA. (Since you didn't say where you live, I have to assume you live where I do. Everyone does, don't they?) --NellieBly (talk) 03:21, 20 March 2008 (UTC)[reply]
Edit conflict: Thanks, no sorry, not Alberta, Canada. I live in the USA, in the State of Florida, in the County of Hillsborough. The City just South of the area in question is not as tolerant (if that is the correct word) of the Homeless as the County, hence one of the reasons that a large population of homeless exists just over the City line in the County. What most people do not really understand including the Deputies who patrol the area that while they are required to pay rent just as I most of these people are unable although it may appear they are only unwilling. It would probably take a psychologist or psychiatrist to comprehend the effect and power of the metal block that stands in the way which can only be removed after the person has be rescued. Its kind of like being overboard with two broken legs and being chastised for not helping to power the ocean liner by kicking when what is needed is a lifeboat and splints until the legs are healed rather than the requirement or demand to start kicking. Surely there must be a world wide homeless organization that knows and understands this and can offer some clear direction. Going to the County Commissioners is a good idea but not without a plan. I need an organization which in fact has a plan. 71.100.10.177 (talk) 09:50, 20 March 2008 (UTC)[reply]
No, Section 8 is a US program. -- Mwalcoff (talk) 04:22, 20 March 2008 (UTC)[reply]
Behind The Choir of Hard Knocks was the recognition of lack of quality of life at this level – something that makes people want to get up in the morning. I googled "homelessness solutions USA" and there are discussions but there might be avenues of funding. Thing about the Choir is it raised people's regard for the participants – and while it began in one city, it caught on in another. I forget where (maybe UK?) that someone implemented a scheme that fostered "responsibility" (or a response at least) by getting people involved in organising something to do with their housing development. It just gave some dynamic to thinking as well as living at that level. Myabe you could get in touch with people who are active in this way. Have you seen our article Street newspapers? There's also The Big Issue as an example of this. Julia Rossi (talk) 07:06, 20 March 2008 (UTC)[reply]
  • Unfortunately the homeless are rarely screened for entertainment talent but that is a very good idea. Theater might become an avenue for the homeless. I can think of any number of plays that reiterate the plight of the down and out. Who better to play the parts? A very good idea.
  • Fostering "responsibility" seems to reflect a common misunderstanding. These people are very responsible. The problem is that they do not want to waste it. They do not want to be responsible for getting a job done, for instance, on behalf of someone else that drains them of even the will to live while making the other person far better off in comparison in exchange for a meal and a place to stay the night. They are people who have been robbed and who do not want to let themselves be robbed again.
  • In America in places where buildings have been all but abandoned and the homeless have asserted squatter's rights, in some cases the government has supported their claims especially through adverse possession laws and an occupancy of longer than 7 years. In the area under discussion, however, many homeless wander the area looking for toilet facilities and have taken up residence across from the County bus terminal on the sidewalk so they can use the facilities as soon as they are open. Public toilets, aside from this, are non-existent in America by design forcing the homeless to search out private facilities. One of the most cherished jobs for the homeless is the job of being the first to arrive in the morning and to clean the bathrooms and police the property for trash before the fast food restaurant opens. The job can lead to things like working the grill when the restaurant opens. Many homeless have personality problems as mental disabilities so such jobs may not last long. 71.100.10.177 (talk) 09:50, 20 March 2008 (UTC)[reply]
Would you mind signing with four of these things ~? It's hard to know which non-signer is speaking. Cheers, Julia Rossi (talk) 09:39, 20 March 2008 (UTC)[reply]
Auto signing should be standard and manual override the exception as well as stable sidebar frames. etc. 71.100.10.177 (talk) 09:50, 20 March 2008 (UTC)[reply]

Divorce and women in the Middle Ages

I've been reading the Wife of Bath's tale from Chaucer. Clearly a woman of the world! This got me thinking of the position of independent minded women in the middle ages. The catholic church allowed divorce for non consumation of marriage. Are there any records of women taking action along these lines and, if so, how did the case proceed?

Medieval crime and punishment

I have another question about life in the middle ages, if I may. We think of the period as one of savage violence, both in crime and in punishment. Was crime peceived to be out of control and is that why punishments were so severe?Alisoun of Bath (talk) 13:32, 20 March 2008 (UTC)[reply]